Cohen Cheat Sheet on “Arising Under” Federal Question Subject Matter Jurisdiction
This is one of three intellectually very difficult parts of the course by my estimation (Erie and preclusion are the others), so do not be discouraged if you find this hard at first (or at twelfth!).
The central question is when a claim “arises under” federal law for the purposes of the statutory grant of FQ SMJ.
The simple answer is given by Justice Holmes in American Well Works v. Layne.
To determine whether an action arises under federal law, look to the law that creates the cause of action (the “Creation Test”). If the cause of action is created by federal law (e.g., Congress passes a statute giving you this cause of action) it “arises under” federal law. If instead the cause of action is created by state law, then the cases “arises under” state and not federal law.
Judge Friendly famously summarized the Holmes’ rule in T.B. Harms Co. v. Eliscu, in which he wrote:
In an endeavor to explain precisely what suits arose under the patent and copyright laws, Mr. Justice Holmes state that “[a] suit arises under the law that creates the cause of action;” in the case sub judice, injury to a business involving slander of a patent, he said, “whether it is a wrong or not depends on the law of the State where the act is done” so that the suit did not arise under the patent laws. American Well Works Co. v. Layne…The Holmes’ “creation” test explains the taking of federal jurisdiction in a great many cases, notably copyright and patent infringement actions, both clearly authorized by the respective federal acts…and thus unquestionably within the scope of 28 U.S.C. § 1338; indeed, in the many infringement suits that depend only on some point of fact and require no construction of federal law, no other explanation may exist…”
For 95% of cases this is a good rule. For example, patent infringement actions arise under federal law because they are created by a Congressional statute. The same is true for Title VII discrimination claims.
But the rule is actually not completely accurate. It is the case that federal claims may “arise under” federal law even where the claim is created by state law. The opposite is also possible, although very rare. We will discuss both exceptions:
First, take the proposition that if the cause of action is created by federal law it “arises under” federal law. That is almost completely true, and in practice you can treat it as true, just keep in your mind the one tiny exception of that is Shoshone Mining Co v. Rutter.
Shoshone Mining Co. v. Rutter, 177 U.S. 505 (1900).
Facts: Congress establishes a system that allows miners to file land patents and to settle conflicting claims, and that the right of possession is determined by “local customs or rules of miners in the several mining districts, so far as the same are applicable and not inconsistent with the laws of the united states.” Is there FQ SMJ for claim under this act?
Holding: No. The “adverse suit” to determine the right of possession may not involve any construction of a federal law or the Constitution. Instead it may just turn on a pure factual matter, like the location of a claim on the ground or the meaning and effect of certain local rules and customs of miners. Therefore does not necessarily arise under the Constitution and laws of the U.S.
Take Away: Not all causes of actions created by federal law confers FQ SMJ, so Holmes test is deficient in this regard too.
That said, this case is a total aardvark! It has not borne much if any fruit. Remember it exists, but don’t worry about it.
The second proposition, that if the cause of action is created by state law, then the cases does NOT “arise[]under” federal law is also not always right, and the deficiency is even more serious/complex. This situation is also much more common.
The issue typically comes up when you have a cause of action created by state law, but part of the cause of action turns on an element of federal law.
Before Merrell Dow the S. Ct. seems to behave somewhat inconsistently in Smith v. Kansas City Title & Trust Co. and Moore v. Chesapeake & Ohio Railway Co.
Smith v. Kansas City Title & Trust Co., 255 U.S. 180 (1921)
Facts Shareholder of Trust Co sues to enjoin the Trust Co from investing in certain federal bonds on the ground that the act of congress authorizing the bonds was unconstitutional. The theory of their case was that under Missouri law, an investment in a bond unauthorized by law was ultra vires and thus could be enjoined. The cause of action was thus state corporate law (the claim the investment was ultra vires). Was there SMJ, given that cause of action was Missouri law?
Holding: There is SMJ, right to relief depends on the construction of a federal statute and such federal claim has a reasonable foundation, that’s enough.
In dissent, Justice Holmes yelled creation test! This is a state law cause of action, it merely incorporates federal law the way you might incorporate a contract]
Compare this with: Moore v. Chesapeake & Ohio Railway Co., 291 U.S. 205 (1934).
Facts: Ky Employer Liability Act says that a plaintiff cannot be held contributorily negligent or to have assumed the risk where his injury results from the violation by his employer of any state or federal statute enacted for the safety of the employee. Plaintiff sued defendant for state law tort under the Kentucky Act. He claimed that his injury was due to the failure of employer to comply with the Federal Safety Appliance Act. Was there SMJ, even though this was a state law cause of action?
Holding: No SMJ, this is just state law negligence claim.
These two seemingly conflicting cases set up the issue of Merrell Dow, what to do in a case involving “the presence of a federal issue in a state-created cause of action”?
Merrell Dow Pharmaceuticals v. Thompson, S. Ct. 1986. Please read Merrell Dow from the playlist after you've finished this portion of the playlist.
Before we can understand this case, you need to understand what is a private right of action? It is a right to sue someone under a statute. That is, can private person A sues private person B for violating a statute. Examples: if universities don’t permit military recruiters on campus the federal government can remove all funding, or defund them. That’s a public right of action, but no corresponding right for a private plaintiff to bring the suit. Without a private right of action, under this scenario an individual cannot bring suit against a university for not allowing military recruiters on campus.
What was the case about? Plaintiffs were mothers of children born with multiple deformities as a result of the mother's ingestion of Bendectin during pregnancy. They sued the drug maker. Most of the counts in the complaint were pure state law claims (e.g., fraud, breach of warranty, etc). In none of those counts is there any federal issue lurking. So clearly would not be able to bring them in Federal Court under FQ SMJ. But in one count, they claimed negligence, a state law cause of action, and said that the plaintiff could prove this state law cause of action by showing that there had been a violation of a federal statute (the FDCA).
Facts: Thompson (Canada) and MacTavish (Scotland) sue Merrell Dow in Ohio state court. Each alleges they had a child with multiple deformities because they used Merrell Dow’s drug Benedictin. They press a bunch of state common law theories (negligence, breach of warranty, strict liability, fraud, gross negligence) and claim that the drug was “misbranded” in violation of the Federal Food, Drug, and Cosmetic Act (FDCA) because the labeling did not provide adequate warning that use of drug was dangerous.
They say in complaint that violation of FDCA in “promotion” of Benedictin creates a rebuttable presumption of negligence and that violation of the statute “directly and proximately caused” the injuries suffered.
Def removes (we’ll discuss this later in this section of course, for now all you need to know is you can only remove an action if it could originally be brought in district court) claiming there is jurisdiction under 1331, FQ. Plaintiffs say no, should be remanded to state court because no SMJ.
Relying on Smith case the district court held that there was 1331 jurisdiction for FDCA claim, but dismissed on Forum non conveniens (we’ll talk about this later too, but notice the interaction of multiple jurisdictional devices).
6th Circuit disagrees and says FDCA does not “create or imply” a private right of action for individuals injured as a result of the Act. Fed SMJ only if Pl’s right to relief “depended necessarily” on a substantial question of federal law. Pl’s complaint treats FDCA as only one possible way of determining that Merrell Dow was negligent. A jury could find negligence without finding a violation of the FDCA, so does not necessarily depend on FDCA. So no SMJ.
Holding: Affirmed. No SMJ. The vast majority of cases where there is 1331 Fed Q SMJ fall under Holmes’ creation test, cases in which federal law creates the cause of action. But we’ve also said that a case may arise under federal law “where the vindication of a right under state law necessarily turned on some construction of federal law.”
This case is definitely not of the “creation test” kind, state law not federal law creates the COA. The question is whether the presence of a federal issue in the state-created cause of action is sufficient to create SMJ?
All parties agree that there is no private right of action under the FDCA here. Whether you have a private right of action = whether a statute gives a private pl a right to sue a private def for actions in violation of the statute.[1]
We have yet to deal with a case that looks like this. In a parallel branch of caselaw relating to implied causes of action, we’ve held that it would flout Congress to intent to create a federal cause of action in this kind of case. We think it would equally flout Congressional intent to give federal SMJ for these actions just because violating a federal statute is said to give rise to a “rebuttable presumption” of negligence under state law.
There are three possible noteworthy objections:
(1) The federal issue here is central to the state law claim.
(2) Powerful federal interest in seeing that the federal statute is given a uniform interpretation and FQ SMJ is best way to do that.
(3) Special factors in this particular case. Not clear if FDCA applies to sales in Scotland or Canada, the extraterritorial meaning of the statute is a special particularly important federal question.
Be prepared to discuss these objections and responses in class. Also make sure to read footnote 12 carefully.
Concl: For the category of cases when state law creates the cause of action, the presence of a federal issue in a state tort law case will not give rise to SMJ when Congress did not give a private right of action for violation of the federal statute. The Congressional determination that there be no federal remedy for violations of this statute is equivalent to a decision by Congress that violations of the federal statute are not “substantial” enough to confer FQ for SMJ.
We will discuss the dissent more in-depth in class. We will also discuss the Supreme Court’s recent decision in Gunn v. Minton developing this jurisprudence. After Merrell Dow, the circuits developed a split on whether § 1331 requires a federal private right of action for federal question jurisdiction to be proper or whether a substantial federal issue embedded in the state cause of action would be sufficient.
When you are reading Gunn, pay close attention to the Court’s discussion of prior cases Grable and Empire Health Choice with regards to that question. Here is a summary of that case.
Grable and Sons v. Darue, 545 U.S. 308 (2005) [ I have included an edited version of the case for you to SKIM]
Facts: In 1994, IRS seizes Michigan real estate belonging to Grable to satisfy the company’s tax delinquency. A statute (26 U.S.C. § 6335) required IRS to give Grable notice of the seizure, the property is sold, and Grable has a statutory right to redeem the property within 180 days, at which point the government lets it go forever and issues a quitclaim deed (deed extinguishing old owner’s rights) to the buyer. That’s what happens here and the quit claim deed is issued to Darue.
FIVE YEARS LATER Grable bring a quiet title action (NOTABLY A STATE LAW CAUSE OF ACTION) claiming that Darue’s title is invalid because the IRS did not comply with the exact manner of the statutory notice requirement – it did it by certified mail not by in hand delivery, which is what the statute requires. The case was initially brought in state court and then removed to federal court. Is FQ SMJ proper?
District court: Yes, the test is does it “pose a significant question of federal law,” the existence of a federal cause of action is not necessary. Finds FQ SMJ and gives Darue a win on the merits, under Summary Judgment, finding substantial compliance with the statute enough, personal service not required.
6th Cir. Affirms. Thought it was enough for FQ SMJ that the title claim raises an issue of federal law that has to be resolved, and there was a substantial federal interest involved (the construal of federal tax law). Also affirms the merits decision.
S. Ct. Holding: The question answered by the Court is whether Merrell Dow always requires a federal cause of action to get FQ jurisdiction. No, it is not required.
The Court finds that a federal cause of action is most common way of getting an FQ, but not the only way. You can have FQ with a state law claim “that implicate significant federal issues.” Why? Because those questions are substantial and federal and “justify resort to the experience, solicitude, and hope of uniformity that a federal forum offers on federal issues.”
-Full statement of test: “The question is, does a state-law claim necessarily raise a stated federal issue, actually disputed and substantial, which a federal forum may entertain without disturbing any congressionally approved balance of federal and state judicial responsibilities.””
What about MERRELL DOW? The Court says: don’t confuse what is a sufficient condition for FQ SMJ for what is a necessary one. Merrell Dow wasn’t silently overruling Smith. Indeed, we approved Smith. We explicitly said there are no bright line rules here, but instead we said careful judgments are in order. Absence of a federal cause of action is important but not dispositive of the inquiry. In Merrell Dow, we considered relevant the purpose of § 1331, and the combo of no federal cause of action and no preemption of state law remedies. It would also have attracted a horde of similar state law cases, and the line would have been swamped for any statutory violation. No indication that Congress wanted that, every indication it did not.
Here things are different. State quiet title actions turning on federal law are rare indeed, so not going to disrupt the balance [
Thomas, Concurring, argues for a return to the Holmes’ dissent, making §§ 1331 jurisdiction co-extensive with whether federal law creates a cause of action on a well-pleaded complaint.
BUT GRABLE DOES NOT RESOLVE THE QUESTION ENTIRELY:
Empire HealthChoice Assurance, Inc. v. McVeigh, 547 U.S. 677 (2006).
Facts: 3rd party injures federal employee McVeigh and he eventually dies. His insurance company pays for a lot of his care. His estate sues the 3rd party and gets lots of money on wrongful death. Insurance company then sues estate to recoup what it paid for his medical expenses. Those expenses were paid as part of a contract with the federal Office of Personnel Management (OPM) to give health insurance to federal employees. The statute governing employee health care is silent on recoupment, but the contract between OPM and insurance company says that insurance company has to take reasonable steps to recoup and employees are warned that if they win a verdict the insurer might seek recoupment. Recoupment is a state law action, is there a federal question?
S. Ct. Holding: NO. Congress did not create a federal cause of action for recoupment by insurance companies. There’s no FQ here, even though there are federal interests in OPM negotiations of master contract, the effect of contract on federal employees, etc.
Why? Because “countervailing considerations control” where Congress explicitly conferred jurisdiction over other benefits but was silent on reimbursement claims (expressio unis). [Dissent’s reply: Failure to confer federal jurisdiction explicitly over reimbursement claims may just reflect a view that it was already covered by §1331.]
Nor is this a substantial federal issue under Grable. Grable is a “special and small category.” Grable can be distinguished because:
Dissent (don’t dwell on it) (Breyer, J., joined by Kennedy, Souter, Alito): Thought this was federal common law because interpretation of a federal contract and Congress intended that federal courts have jurisdiction. The U.S. is real party in interest because the money is credited to the federal fund. Uniformity important here because benefits are provided under federal program.
Gunn v. Minton is the Supreme Court’s latest foray into this complicated area of jurisprudence and will be our main focuse. Pay attention to the Court's summaries of Empire Health and Grable
[1] Note: At the time the case is decided in determining whether there is a private right of action the court applies the Cort v. Ash 4-factor test. Here they assume 4 factors obtain as to FDCA:
(1) Pl are not in class of people for whose special benefit the FDCA was passed.
(2) No indication that Congress intended to confer on these people a private right of action.
(3) A federal cause of action would not further the legislative scheme of FDCA.
(4) Pl’s cause of action is of the kind traditionally relegated to state law.
Later the S.Ct somewhat backs off of this test, but at time of Merrell Dow this governs…more on this in leg reg + fed courts.
Summarized in cheat sheet. OPTIONAL if you want to read more of it.
Supreme Court of United States.
CERTIORARI TO THE CIRCUIT COURT OF APPEALS FOR THE SEVENTH CIRCUIT.
8[206] Mr. Edward Davidson, with whom Mr. John P. Bramhall was on the brief, for petitioner.
9Mr. Albert H. Cole for respondent.
10Petitioner brought this action in the District Court of the United States for the Northern District of Indiana, Fort Wayne Division, to recover for injuries which he sustained on November 29, 1930, in the course of his employment by respondent, an interstate carrier, in its yard [208] at Russell, Kentucky. In his complaint he set forth two "paragraphs" or counts, both being for the same injuries. In the first paragraph, petitioner alleged that at the time of the injuries he was employed in interstate commerce and that he brought the action under the Acts of Congress known as the Federal Employers' Liability Act[1] and the Safety Appliance Acts,[2] and the rules and orders which the Interstate Commerce Commission had promulgated under the latter.[3] In the second paragraph, he alleged that at the time of the injuries he was employed in intrastate commerce and he invoked the Safety Appliance Acts enacted by the Congress, and the rules and orders of the Interstate Commerce Commission thereunder, and the Employers' Liability Act of Kentucky. The provisions of the laws of Kentucky which were alleged to govern the rights of the parties at the time and place in question were set forth.[4] In each count petitioner stated that the injuries were received while he was engaged as a switchman in attempting to uncouple certain freight cars and were due to a defective uncoupling lever.
12Objections to the jurisdiction of the District Court as to each count were raised by plea in abatement. They were overruled and petitioner had a general verdict. The judgment, entered accordingly, was reversed by the Circuit Court of Appeals upon the ground that the District Court was without jurisdiction to entertain the case upon [209] either count. 64 F. (2d) 472. This Court granted certiorari.
13Distinct questions are presented with respect to each count and they will be considered separately.
14First. By the first paragraph, the jurisdiction of the Federal court was rested upon the sole ground that the injury had been sustained during petitioner's employment in interstate commerce and that the cause of action arose under the pertinent Federal legislation. To support the jurisdiction of the District Court for the Northern District of Indiana, the complaint alleged that respondent was engaged in business in that district at the time of the commencement of the action. Respondent's challenge to the jurisdiction was upon the grounds (1) that at the time of the injuries petitioner was not employed in interstate commerce and hence the action would not lie under the Federal Employers' Liability Act, and (2) that respondent was a corporation organized under the laws of Virginia and an inhabitant of the Eastern District of Virginia, and hence, so far as the action rested upon the Safety Appliance Acts of Congress, and the rules and orders of the Interstate Commerce Commission, it could not be brought in a Federal court in any district other than the Eastern District of Virginia. Jud. Code, § 51; 28 U.S.C. § 112.
15Petitioner's demurrer to the plea in abatement as to the first cause of action was sustained by the trial court. That court pointed out that the plea did not deny that respondent was doing business within the Northern District of Indiana and that the pleading, in substance, went to the merits. The Circuit Court of Appeals took a different view, holding that so far as petitioner relied upon a violation of the Safety Appliance Acts, the action must be brought in the district of respondent's residence. In reversing the judgment, the Circuit Court of Appeals remanded [210] the cause with instructions to grant permission to petitioner to amend his first paragraph to conform exclusively to the theory of a violation of the Federal Employers' Liability Act.
16This ruling of the appellate court cannot be sustained. The jurisdiction of the District Court is to be determined by the allegations of the complaint. Mosher v. Phoenix, 287 U.S. 29, 30; Levering & Garrigues Co. v. Morrin, 289 U.S. 103, 105. These allegations clearly set forth, in the first paragraph, a cause of action under the Federal Employers' Liability Act. Every essential ingredient of such a cause of action was appropriately alleged. The Federal Employers' Liability Act expressly recognized that in an action brought under its provisions the question of a violation of the Safety Appliance Acts might be presented and determined. This is the unmistakable effect of the provisions that, in such an action, the employee shall not be held "to have been guilty of contributory negligence," or "to have assumed the risks of his employment" in any case "where the violation by such common carrier of any statute enacted for the safety of employees contributed to the injury or death of such employee." Act of April 22, 1908, §§ 3, 4, 45 U.S.C. §§ 53, 54. By the phrase "any statute enacted for the safety of employees" the Congress evidently intended to embrace its Safety Appliance Acts. Seaboard Air Line Ry. v. Horton, 233 U.S. 492, 503. This Court has said that the statutes are in pari materia and that "where the Employers' Liability Act refers to `any defect or insufficiency, due to its negligence, in its cars, engines, appliances,' etc., it clearly is the legislative intent to treat a violation of the Safety Appliance Act as `negligence' — what is sometimes called negligence per se." San Antonio & Aransas Pass Ry. Co. v. Wagner, 241 U.S. 476, 484. Where an employee of an interstate carrier sustains injuries while employed in the interstate commerce of the carrier, his action [211] may thus be brought under the Federal Employers' Liability Act in connection with the Safety Appliance Acts.[5]
17Under the Federal Employers' Liability Act an action may be brought "in a District Court of the United States, in the district of the residence of the defendant, or in which the cause of action arose, or in which the defendant shall be doing business at the time of commencing such action." 45 U.S.C. § 56. It follows that, upon the allegations of the complaint, the action on the claim set forth in the first paragraph was properly brought in the District Court for the Northern District of Indiana where respondent was doing business when the action was begun.
18Second. In the second paragraph of the complaint, which treated the injuries as received in intrastate commerce, diversity of citizenship was alleged; that petitioner was a citizen of Indiana, and a resident of the city of Fort Wayne in that State, and that respondent was a citizen of Virginia doing business in Indiana. The plea in abatement, admitting respondent's citizenship in Virginia, denied that petitioner was a resident of Fort Wayne or of the Northern District of Indiana, or was a citizen of that State, and alleged that as the cause of action set forth in the second paragraph arose under the Federal Safety Appliance Acts, the action could not be brought [212] in any district other than the Eastern District of Virginia. The District Court took evidence on the issue of fact, found that the petitioner was a citizen of Indiana and a resident of Fort Wayne, and overruled the plea. The Circuit Court of Appeals held that the District Court of the Northern District of Indiana was without jurisdiction, in the view that the second count attempted to set forth a cause of action "under the Federal Safety Appliance Act as well as under the statutes of Kentucky" and hence that jurisdiction did not rest solely on diversity of citizenship. Jud. Code, § 51, 28 U.S.C. § 112. In remanding the cause, the Circuit Court of Appeals directed that petitioner be allowed to amend the second paragraph of his complaint so as to conform exclusively to the theory of a violation of the Kentucky statute.
19While invoking, in the second count, the Safety Appliance Acts, petitioner fully set forth and relied upon the laws of the State of Kentucky where the cause of action arose. In relation to injuries received in that State in intrastate commerce, aside from the particular bearing of the Federal Safety Appliance Acts, the liability of respondent was determined by the laws of Kentucky. Slater v. Mexican National R. Co., 194 U.S. 120, 126; Cuba R. Co. v. Crosby, 222 U.S. 473, 478; Young v. Masci, 289 U.S. 253, 258; Ormsby v. Chase, 290 U.S. 387. The statute of Kentucky, in prescribing the liability of common carriers for negligence causing injuries to employees while engaged in intrastate commerce, reproduced in substance, and with almost literal exactness, the corresponding provisions of the Federal Employers' Liability Act as to injuries received in interstate commerce. Ky. Acts, 1918, c. 52, §§ 1-3, p. 153; Carroll's Ky. Statutes, 1930, §§ 820 b-1, 820 b-2, 820 b-3. The Kentucky Act provided that no employee should be held "to have been guilty of contributory negligence" or "to have assumed the risk of his employment" in any case "where the violation by [213] such common carrier of any statute, state or federal, enacted for the safety of employees contributed to the injury or death of such employee." Id. The Kentucky legislature read into its statute the provisions of statutes both state and federal which were enacted for the safety of employees, and the Federal Safety Appliance Acts were manifestly embraced in this description. Louisville & Nashville R. Co. v. Layton, 243 U.S. 617, 619. Thus, the second count of the complaint, in invoking the Federal Safety Appliance Acts, while declaring on the Kentucky Employers' Liability Act, cannot be regarded as setting up a claim which lay outside the purview of the state statute. As in the analogous case under the Federal Employers' Liability Act, a violation of the acts for the safety of employees was to constitute negligence per se in applying the state statute and was to furnish the ground for precluding the defense of contributory negligence as well as that of assumption of risk.
20The Circuit Court of Appeals took the view that if it were assumed that the second count was based exclusively upon the Kentucky statute, that statute and the federal requirements could not be considered as being in pari materia because the latter applied only to interstate commerce, and that, if the petitioner were permitted to establish the negligence required by the state statute by showing the violation of the federal requirements the court would thereby be placed "in the anomalous position of extending the benefits of the Safety Appliance Act to intrastate commerce."
21This is an erroneous view. The original Safety Appliance Act of March 2, 1893, 27 Stat. 531, did not embrace all cars on the lines of interstate carriers but only those engaged in interstate commerce. Brinkmeier v. Missouri Pacific Ry. Co., 224 U.S. 268. By the amending Act of March 2, 1903, 32 Stat. 943, the scope of the statute was enlarged so as to include all cars "used on any railroad [214] engaged in interstate commerce." The statute as amended was intended to embrace all locomotives, cars, and similar vehicles used on any railroad which is a highway of interstate commerce. Southern Ry. Co. v. United States, 222 U.S. 20. With respect to such vehicles, the duty to protect employees by the prescribed safety appliances exists even though the vehicles and the employee injured through the failure to provide such protection are at the time engaged in intrastate commerce. Texas & Pacific Ry. Co. v. Rigsby, 241 U.S. 33; Louisville & Nashville R. Co. v. Layton, supra. The Federal Act in its application to such a case is thus in pari materia with the statute of Kentucky which prescribes the liability of carriers for injuries to employees while employed in intrastate commerce and which, in effect, reads into the provisions of the statute the requirements of the Federal Act for the safety of employees. There appears to be no anomaly in enforcing the state law with this defined content.
22The Federal Safety Appliance Acts prescribed duties, and injured employees are entitled to recover for injuries sustained through the breach of these duties. Johnson v. Southern Pacific Co., 196 U.S. 1; St. Louis, I.M. & S. Ry. Co. v. Taylor, 210 U.S. 281; Texas & Pacific Ry. Co. v. Rigsby, supra. Questions arising in actions in state courts to recover for injuries sustained by employees in intrastate commerce and relating to the scope or construction of the Federal Safety Appliance Acts are, of course, federal questions which may appropriately be reviewed in this Court. St. Louis, I.M. & S. Ry. Co. v. Taylor, supra; Louisville & Nashville R. Co. v. Layton, supra. But it does not follow that a suit brought under the state statute which defines liability to employees who are injured while engaged in intrastate commerce, and brings within the purview of the statute a breach of the duty imposed by the federal statute, should be regarded as a suit arising under the laws of the United States and cognizable in the [215] federal court in the absence of diversity of citizenship. The Federal Safety Appliance Acts, while prescribing absolute duties, and thus creating correlative rights in favor of injured employees, did not attempt to lay down rules governing actions for enforcing these rights. The original Act of 1893 made no provision for suits, except for penalties. That Act did impliedly recognize the employee's right of action by providing in § 8 that he should not be deemed to have assumed the risk of injury occasioned by the breach of duty. But the Act made no provision as to the place of suit or the time within which it should be brought, or as to the right to recover, or as to those who should be the beneficiaries of recovery, in case of the death of the employee. While dealing with assumption of risk, the statute did not affect the defense of contributory negligence and hence that defense was still available according to the applicable state law. Schlemmer v. Buffalo, R. & P. Ry. Co., 220 U.S. 590; Minneapolis, St. P. & S.S.M. Ry. Co. v. Popplar, 237 U.S. 369, 371, 372. In these respects the amended Act of 1903 made no change, notwithstanding the enlargement of the scope of the statutory requirements. The Act of 1910, by a proviso in § 4 relating to penalties (36 Stat. 299), provided that nothing in that section should "be construed to relieve such carrier from liability in any remedial action for the death or injury of any railroad employee" caused by the use of the prohibited equipment.
23The Safety Appliance Acts having prescribed the duty in this fashion, the right to recover damages sustained by the injured employee through the breach of duty sprang from the principle of the common law (Texas & Pacific R. Co. v. Rigsby, supra, at pp. 39, 40[6]) and was left to be enforced accordingly, or, in case of the death of [216] the injured employee, according to the applicable statute.[7] St. Louis, I.M. & S. Ry. Co. v. Taylor, supra, at p. 285; Minneapolis, St. P. & S.S.M. Ry. Co. v. Popplar, supra. When the Federal Employers' Liability Act was enacted, it drew to itself the right of action for injuries or death of the employees within its purview who were engaged in interstate commerce, including those cases in which injuries were due to a violation of the Safety Appliance Acts. Such an action must be brought as prescribed in the Federal Employers' Liability Act, and if brought in the state court, it cannot be removed to the federal court, although violation of the Safety Appliance Acts is involved. See St. Joseph & G.I. Ry. Co. v. Moore, 243 U.S. 311. With respect to injuries sustained in intrastate commerce, nothing in the Safety Appliance Acts precluded the State from incorporating in its legislation applicable to local transportation the paramount duty which the Safety Appliance Acts imposed as to the equipment of cars used on interstate railroads. As this Court said in Minneapolis, St. P. & S.S.M. Ry. Co. v. Popplar, supra, as to an action for injuries sustained in intrastate commerce: "The action fell within the familiar category of cases involving the duty of a master to his servant. This duty is defined by the common law, except as it may be modified by legislation. The federal statute, in the present case, touched the duty of the master at a single point and, save as provided in the statute, the right of the [217] plaintiff to recover was left to be determined by the law of the State."
24We are of the opinion that the second paragraph of the complaint set forth a cause of action under the Kentucky statute and, as to this cause of action, the suit is not to be regarded as one arising under the laws of the United States. In view of the diversity of citizenship and the residence of petitioner, the District Court of the Northern District of Indiana had jurisdiction.
25As the Circuit Court of Appeals did not consider any questions save those relating to the jurisdiction of the District Court, the judgment of the Circuit Court of Appeals will be reversed and the cause remanded to that court with directions to consider such other questions as may be presented by the appeal.
26Reversed.
27[1] Act of April 22, 1908, c. 149, 35 Stat. 65, 45 U.S.C. §§ 51 et seq.; Act of April 5, 1910, c. 143, 36 Stat. 291, 45 U.S.C. § 56.
28[2] Acts of March 2, 1893, c. 196, 27 Stat. 531, 45 U.S.C. §§ 1 et seq.; April 1, 1896, c. 87, 29 Stat. 85, 45 U.S.C. § 6; March 2, 1903, c. 976, 32 Stat. 943, 45 U.S.C. §§ 8, 9, 10; April 14, 1910, c. 160, 36 Stat. 298, 45 U.S.C. §§ 11 et seq.
29[3] Order of March 13, 1911; Roberts' Federal Liabilities of Carriers, Vol. 2, pp. 2010, 2016.
30[4] Ky. Acts, 1918, c. 52, §§ 1-3, p. 153; Carroll's Ky. Statutes, §§ 820 b-1, 820 b-2, 820 b-3.
31[5] See Southern Ry. Co. v. Crockett, 234 U.S. 725, 727; St. Louis & San Francisco R. Co. v. Conarty, 238 U.S. 243, 248; Great Northern Ry. Co. v. Otos, 239 U.S. 349, 350; San Antonio & Aransas Pass Ry. Co. v. Wagner, 241 U.S. 476, 484; Spokane & I.E.R. Co. v. Campbell, 241 U.S. 497, 498; Atlantic City R. Co. v. Parker, 242 U.S. 56, 58; St. Joseph & G.I. Ry. Co. v. Moore, 243 U.S. 311, 312; Minneapolis & St. Louis R. Co. v. Gotschall, 244 U.S. 66; Great Northern Ry. Co. v. Donaldson, 246 U.S. 121, 124; Davis v. Wolfe, 263 U.S. 239, 240; Baltimore & Ohio R. Co. v. Groeger, 266 U.S. 521, 528; Chicago Great Western R. Co. v. Schendel, 267 U.S. 287, 289; Minneapolis, St. P. & S.S.M. Ry. Co. v. Goneau, 269 U.S. 406, 407.
32[6] In Texas & Pacific R. Co. v. Rigsby, 241 U.S. 33, the action was brought in the state court and was removed to the federal court upon the ground that the defendant was a federal corporation.
33[7] In St. Louis, I.M. & S. Ry. Co. v. Taylor, 210 U.S. 281, 285, the Court said: "The accident by which the plaintiff's intestate lost his life occurred in the Indian Territory, where, contrary to the doctrine of the common law, a right of action for death exists. The cause of action arose under the laws of the Territory, and was enforced in the courts of Arkansas." The question whether the action was triable in those courts was held not to present a federal question, but the question as to the interpretation of the Safety Appliance Act of 1893 did present the federal question which was reviewed by this Court.
Summarized in cheat sheet. OPTIONAL if you want to read more of it.
Supreme Court of United States.
APPEAL FROM THE DISTRICT COURT OF THE UNITED STATES FOR THE WESTERN DISTRICT OF MISSOURI.
10[181] Mr. William Marshall Bullitt for appellant.
11Mr. Frank Hagerman for appellant.
12Mr. Charles E. Hughes for Federal Land Bank of Wichita, Kansas, appellee.
13Mr. W.W. Willoughby filed a separate brief on behalf of the Federal Land Bank of Wichita, Kansas, appellee.
14Mr. George W. Wickersham, with whom Mr. W.G. McAdoo was on the briefs, for First Joint Stock Land Bank of Chicago, appellee.
15Mr. Justin D. Bowersock filed a brief on behalf of Kansas City Title & Trust Company, appellee.
16The Solicitor General, Mr. W.G. McAdoo, Special Assistant to the Attorney General, and Mr. J.P. Cotton, by leave of court, filed a brief on behalf of the United States as amici curiae.[1]
17Restored to docket for reargument April 26, 1920.
18A bill was filed in the United States District Court for the Western Division of the Western District of Missouri by a shareholder in the Kansas City Title & Trust Company to enjoin the Company, its officers, agents and employees from investing the funds of the Company in farm loan bonds issued by Federal Land Banks or Joint Stock Land Banks under authority of the Federal Farm Loan Act of July 17, 1916, c. 245, 39 Stat. 360, as amended January 18, 1918, c. 9, 40 Stat. 431.
20The relief was sought on the ground that these acts were beyond the constitutional power of Congress. The bill avers that the Board of Directors of the Company are [196] about to invest its funds in the bonds to the amount of $10,000 in each of the classes described, and will do so unless enjoined by the court in this action. The bill avers the formation of twelve Federal Land Banks, and twenty-one Joint Stock Land Banks under the provisions of the act.
21As to the Federal Land Banks, it is averred that each of them has loaned upon farm lands large amounts secured by mortgage, and, after depositing the same with the Farm Loan Registrar, has executed and issued collateral trust obligations called Farm Loan Bonds, secured by the depositing of an equivalent amount of farm mortgages and notes; and that each of said Federal Land Banks has sold, and is continuing to offer for sale, large amounts of said Farm Loan Bonds. The bill also avers that various persons in different parts of the United States have organized twenty-one Joint Stock Land Banks, the capital stock of which is subscribed for and owned by private persons; that the Joint Stock Land Banks have deposited notes and mortgages with the Farm Loan Registrar, and issued an equivalent amount of collateral trust obligations called Farm Loan Bonds, which have been sold and will be continued to be offered for sale to investors in large amounts in the markets of the country. A statement is given of the amount of deposits by the Secretary of the Treasury with the Federal Land Banks, for which the banks have issued their certificates of indebtedness bearing interest at 2% per annum. It is averred that on September 30, 1919, Federal Land Banks owned United States bonds of the par value of $4,230,805; and the Joint Stock Land Banks owned like bonds of the par value of $3,287,503 on August 31, 1919; that pursuant to the provisions of the act the Secretary of the Treasury has invested $8,892,130 of the public funds in the capital stock of the Federal Land Banks, and that on July 1, 1919, the Secretary of the Treasury on behalf of the United States held $8,265,809 of the capital stock of the Federal Land Banks; [197] that pursuant to the provisions of § 32 of the act, as amended, the Secretary of the Treasury has purchased Farm Loan Bonds issued by the Federal Land Banks of the par value of $149,775,000; that up to September 30, 1919, bonds have been issued under the act by the Federal Land Banks to the amount of $285,600,000, of which about $135,000,000 are held in the Treasury of the United States, purchased under the authority of the amendment of January 18, 1918; that up to September 30, 1919, twenty-seven Joint Stock Land Banks have been incorporated under the act, having an aggregate capital of $8,000,000, all of which has been subscribed and $7,450,000 paid in; that bonds have been issued by Joint Stock Land Banks to the amount of $41,000,000, which are now in the hands of the public; that the Secretary of the Treasury up to the time of the filing of the bill has not designated any of the Federal Land Banks nor the Joint Stock Land Banks as depositaries of public money, nor, except as stated later in the bill, has he employed them or any of them as financial agents of the Government, nor have they or any of them performed any duties as depositaries of public money, nor have they or any of them accepted any deposits or engaged in any banking business. The bill avers that during the summer of 1918 the Federal Land Banks at Wichita, St. Paul and Spokane were designated as financial agents of the Government for making seed grain loans to farmers in drought-stricken sections, the President having at the request of the Secretary of Agriculture set aside $5,000,000 for that purpose out of the $100,000,000 war funds. The three banks mentioned made upwards of 15,000 loans of that character, aggregating a sum upwards of $4,500,000, and are now engaged in collecting these loans, all of which are secured by crop liens; that these banks act in that capacity without compensation, receiving only the actual expenses incurred.
22Section 27 of the act provides that Farm Loan Bonds [198] issued under the provisions of the act by Federal Land Banks or Joint Stock Land Banks shall be a lawful investment for all fiduciary and trust funds, and may be accepted as security for all public deposits. The bill avers that the defendant Trust Company is authorized to buy, invest in and sell government, state and municipal and other bonds, but it cannot buy, invest in or sell any such bonds, papers, stocks or securities which are not authorized to be issued by a valid law or which are not investment securities, but that nevertheless it is about to invest in Farm Loan Bonds; that the Trust Company has been induced to direct its officers to make the investment by reason of its reliance upon the provisions of the Farm Loan Acts, especially §§ 21, 26 and 27, by which the Farm Loan Bonds are declared to be instrumentalities of the Government of the United States, and as such with the income derived therefrom, are declared to be exempt from federal, state, municipal and local taxation, and are further declared to be lawful investments for all fiduciary and trust funds. The bill further avers that the acts by which it is attempted to authorize the bonds are wholly illegal, void and unconstitutional and of no effect because unauthorized by the Constitution of the United States.
23The bill prays that the acts of Congress authorizing the creation of the banks, especially §§ 21, 26 and 27 thereof, shall be adjudged and decreed to be unconstitutional, void and of no effect, and that the issuance of the Farm Loan Bonds, and the taxation exemption feature thereof, shall be adjudged and decreed to be invalid.
24The First Joint Stock Land Bank of Chicago and the Federal Land Bank of Wichita, Kansas, were allowed to intervene and became parties defendant to the suit. The Kansas City Title & Trust Company filed a motion to dismiss in the nature of a general demurrer, and upon hearing the District Court entered a decree dismissing the bill. From this decree appeal was taken to this court.
25[199] No objection is made to the federal jurisdiction, either original or appellate, by the parties to this suit, but that question will be first examined. The Company is authorized to invest its funds in legal securities only. The attack upon the proposed investment in the bonds described is because of the alleged unconstitutionality of the acts of Congress undertaking to organize the banks and authorize the issue of the bonds. No other reason is set forth in the bill as a ground of objection to the proposed investment by the Board of Directors acting in the Company's behalf. As diversity of citizenship is lacking, the jurisdiction of the District Court depends upon whether the cause of action set forth arises under the Constitution or laws of the United States. Judicial Code, § 24.
26The general rule is that where it appears from the bill or statement of the plaintiff that the right to relief depends upon the construction or application of the Constitution or laws of the United States, and that such federal claim is not merely colorable, and rests upon a reasonable foundation, the District Court has jurisdiction under this provision.
27At an early date, considering the grant of constitutional power to confer jurisdiction upon the federal courts, Chief Justice Marshall said: "A case in law or equity consists of the right of the one party, as well as of the other, and may truly be said to arise under the Constitution or a law of the United States, whenever its correct decision depends on the construction of either," Cohens v. Virginia, 6 Wheat. 264, 379; and again, when "the title or right set up by the party may be defeated by one construction of the Constitution or law of the United States, and sustained by the opposite construction." Osborn v. Bank of the United States, 9 Wheat. 738, 822. These definitions were quoted and approved in Patton v. Brady, 184 U.S. 608, 611, citing Gold-Washing Co. v. Keyes, 96 U.S. 199, 201; Tennessee v. Davis, 100 U.S. 257; White v. Greenhow, 114 U.S. 307; Railroad Company v. Mississippi, 102 U.S. 135, 139.
28[200] This characterization of a suit arising under the Constitution or laws of the United States has been followed in many decisions of this and other federal courts. See Macon Grocery Co. v. Atlantic Coast Line R.R. Co., 215 U.S. 501 506, 507; Shulthis v. McDougal, 225 U.S. 561, 569, paragraph 3. The principle was applied in Brushaber v. Union Pacific R.R. Co., 240 U.S. 1, in which a shareholder filed a bill to enjoin the defendant corporation from complying with the income tax provisions of the Tariff Act of October 3, 1913. In that case while there was diversity of citizenship, a direct appeal to this court was sustained because of the constitutional questions raised in the bill, which had been dismissed by the court below. The repugnancy of the statute to the Constitution of the United States, as well as grounds of equitable jurisdiction, were set forth in the bill, and the right to come here on direct appeal was sustained because of the averments based upon constitutional objections to the act. Reference was made to Pollock v. Farmers' Loan & Trust Co., 157 U.S. 429, where a similar shareholder's right to sue was maintained, and a direct appeal to this court from a decree of the Circuit Court was held to be authorized.
29In the Brushaber Case the Chief Justice, speaking for the court, said:
3031"The right to prevent the corporation from returning and paying the tax was based upon many averments as to the repugnancy of the statute to the Constitution of the United States, of the peculiar relation of the corporation to the stockholders and their particular interests resulting from many of the administrative provisions of the assailed act, of the confusion, wrong and multiplicity of suits and the absence of all means of redress which would result if the corporation paid the tax and complied with the act in other respects without protest, as it was alleged it was its intention to do. To put out of the way a question of jurisdiction we at once say that in view of these averments and [201] the ruling in Pollock v. Farmers' Loan & Trust Co., 157 U.S. 429, sustaining the right of a stockholder to sue to restrain a corporation under proper averments from voluntarily paying a tax charged to be unconstitutional on the ground that to permit such a suit did not violate the prohibitions of § 3224, Rev. Stat., against enjoining the enforcement of taxes, we are of opinion that the contention here made that there was no jurisdiction of the cause since to entertain it would violate the provisions of the Revised Statutes referred to is without merit. . . .
"Aside from averments as to citizenship and residence, recitals as to the provisions of the statute and statements as to the business of the corporation contained in the first ten paragraphs of the bill advanced to sustain jurisdiction, the bill alleged twenty-one constitutional objections specified in that number of paragraphs or subdivisions. As all the grounds assert a violation of the Constitution, it follows that in a wide sense they all charge a repugnancy of the statute to the Sixteenth Amendment under the more immediate sanction of which the statute was adopted."
The jurisdiction of this court is to be determined upon the principles laid down in the cases referred to. In the instant case the averments of the bill show that the directors were proceeding to make the investments in view of the act authorizing the bonds about to be purchased, maintaining that the act authorizing them was constitutional and the bonds valid and desirable investments. The objecting shareholder avers in the bill that the securities were issued under an unconstitutional law, and hence of no validity. It is, therefore, apparent that the controversy concerns the constitutional validity of an act of Congress which is directly drawn in question. The decision depends upon the determination of this issue.
32The general allegations as to the interest of the shareholder, and his right to have an injunction to prevent the purchase of the alleged unconstitutional securities by misapplication [202] of the funds of the corporation, give jurisdiction under the principles settled in Pollock v. Farmers' Loan & Trust Co., and Brushaber v. Union Pacific R.R. Co., supra. We are, therefore, of the opinion that the District Court had jurisdiction under the averments of the bill, and that a direct appeal to this court upon constitutional grounds is authorized.
33We come to examine the questions presented by the attack upon the constitutionality of the legislation in question. The Federal Farm Loan Act is too lengthy to set out in full. It is entitled: "An Act To provide capital for agricultural development, to create standard forms of investment based upon farm mortgage, to equalize rates of interest upon farm loans, to furnish a market for United States bonds, to create Government depositaries and financial agents for the United States, and for other purposes."
34The administration of the act is placed under the direction and control of a Federal Farm Loan Bureau established at the seat of Government in the Treasury Department, under the general supervision of the Federal Farm Loan Board, consisting of the Secretary of the Treasury and four members appointed by the President by and with the advice and consent of the Senate. The United States is divided into twelve districts for the purpose of establishing Federal Land Banks. Each of the banks must have a subscribed capital of not less than $750,000, divided into shares of $5.00 each, which may be subscribed for by any individual, firm or corporation, or by the government of any State, or of the United States. No dividends shall be paid on the stock owned by the United States, but all other stock shall share in dividend distributions without preference. The Federal Farm Loan Board is to designate five directors who shall temporarily manage the affairs of each Federal Land Bank, and who shall prepare an organization certificate which, when approved by the Federal Farm Loan Board and filed with the Farm Loan Commissioner, [203] shall operate to create the bank a body corporate. The Federal Farm Loan Board is required to open books of subscription for the capital stock of each Federal Land Bank, and, if within thirty days thereafter any part of the minimum capitalization of $750,000 of any such bank shall remain unsubscribed, it is made the duty of the Secretary of the Treasury to subscribe the balance on behalf of the United States.
35The amendment of January 18, 1918, authorizes the Secretary of the Treasury to purchase bonds issued by Federal Land Banks, and provides that the temporary organization of any such bank shall be continued so long as any Farm Loan Bonds shall be held by the Treasury, and until the subscription to stock in such bank by National Farm Loan Associations shall equal the amount of the stock held by the United States Government. When these conditions are complied with a permanent organization is to take over the management of the bank consisting of a Board of Directors composed of nine members, three of whom shall be known as district directors and shall be appointed by the Farm Loan Board, who shall represent the public interest, six of whom to be known as local directors, shall be chosen by, and be representative of National Farm Loan Associations.
36Federal Land Banks are empowered to invest their funds in the purchase of qualified first mortgages on farm lands situated within the Federal Land Bank District within which they are organized or acting. Loans on farm mortgages are to be made to cooperative borrowers through the organization of corporations known as National Farm Loan Associations, by persons desiring to borrow money on farm mortgage security under the terms of the act. Ten or more natural persons who are the owners of or are about to become the owners of farm land qualified as security for mortgage loans, and who desire to borrow money on farm mortgage security, may unite to form a National Farm [204] Loan Association. The manner of forming these associations, and the qualifications for membership, are set out in the act.
37A loan desired by each such person must be for not more than $10,000 nor less than $100, and the aggregate of the desired loans not less than $20,000. The application for loan must be accompanied by subscriptions to stock of a Federal Land Bank equal to 5% of the aggregate sum desired on the mortgage loan. Provision is made for appraisal of the land, and report to the Federal Farm Loan Board. No persons but borrowers on farm loan mortgages shall be members or shareholders of National Farm Loan Associations.
38Shareholders in Farm Loan Associations are made individually responsible for the debts of the Association to the extent of the amount of the stock owned by them respectively, in addition to the amount paid in and represented by their shares.
39When any National Farm Loan Association shall desire to secure for any member a loan on first mortgage from the Federal Land Bank in its district, it must subscribe to the capital stock of the Federal Land Bank to an amount of 5% of such loan, which capital stock shall be held by the Federal Land Bank as collateral security for the payment of the loan, the Association shall be paid any dividends accruing and payable on the capital stock while it is outstanding. Such stock may, in the discretion of the directors and with the approval of the Federal Farm Loan Board, be paid off at par and retired, and shall be so retired upon the full payment of the mortgage loan. In such event, the National Farm Loan Association must pay off at par and retire the corresponding shares of its stock which were issued when the Land Bank stock so retired was issued; but it is further provided that the capital stock of the Land Bank shall not be reduced to less than 5% of the principal of the outstanding Farm [205] Loan Bonds issued by it. The shares in National Farm Loan Associations shall be of the par value of $5.00 each.
40At least 25% of that part of the capital of any Federal Land Bank for which stock is outstanding in the name of National Farm Loan Associations must be held in quick assets. Not less than 5% of such capital must be invested in United States Government Bonds.
41The loans which Federal Land Banks may make upon first mortgages on farm lands are provided for in § 12 of the act. By § 13 these banks are empowered, subject to the provisions of the act, to issue and sell Farm Loan Bonds of the kind described in the act, and to invest funds in their possession in qualified first mortgages on farm lands, to receive and to deposit in trust with the Farm Loan Registrar, to be held by him as collateral security for Farm Loan Bonds, first mortgages upon farm lands, and, with the approval of the Farm Loan Board, to issue and to sell their bonds secured by the deposit of first mortgages on qualified farm lands as collateral, in conformity with the provisions of § 18 of the act. By the amendment of January 18, 1918, the Secretary of the Treasury was empowered during the years 1918 and 1919, to purchase Farm Loan Bonds issued by Federal Land Banks to an amount not exceeding $100,000,000 each year, and any Federal Land Bank was authorized at any time to repurchase at par and accrued interest, for the purpose of redemption or resale, any of the bonds so purchased from it and held in the United States Treasury.
42It is also provided that the bonds of any Federal Land Bank so purchased and held in the Treasury one year after the termination of the pending war shall, upon thirty days' notice from the Secretary of the Treasury, be redeemed and repurchased by such bank at par and accrued interest. By § 15 it is provided that whenever, after the act shall have been in effect for one year, it shall appear to the Federal Farm Loan Board that National Farm Loan [206] Associations have not been formed and are not likely to be formed, in any locality, because of peculiar local conditions, the Board may in its discretion authorize Federal Land Banks to make loans on farm lands through agents approved by the Board, on the terms and conditions and subject to the restrictions prescribed in that section.
43The act also authorizes the incorporation of Joint Stock Land Banks, with capital provided by private subscription. They are organized by not less than ten natural persons, and are subject to the requirements of the provisions of § 4 of the act so far as applicable. The board of directors shall consist of not less than five members. Each shareholder shall have the same voting privileges as the holders of shares in National Banking Associations, and shall be held individually responsible, equally and ratably, and not one for another, for all contracts, debts, and engagements of such bank to the extent of the amount of stock owned by them at the par value thereof, in addition to the amount paid in and represented by their shares. The Joint Stock Land Bank is authorized to do business when capital stock to the amount of $250,000 has been subscribed, and one-half paid in cash, the balance remaining subject to call by the board of directors, the charter to be issued by the Federal Farm Loan Board. No bonds shall be issued until the capital stock is entirely paid up. Except as otherwise provided, Joint Stock Land Banks shall have the powers of and be subject to all the restrictions and conditions imposed on Federal Land Banks by the act, so far as such conditions or restrictions are applicable.
44Federal Land Banks may issue Farm Loan Bonds up to twenty times their capital and surplus. Joint Stock Land Banks are limited to the issue of Farm Loan Bonds not in excess of fifteen times the amount of their capital and surplus. Joint Stock Land Banks can only loan on first mortgages upon land in the State where located, or in a State [207] contiguous thereto. No loan on mortgage may be made by any bank at a rate exceeding 6% per annum exclusive of amortization payments. Joint Stock Land Banks shall in no case charge a rate of interest on farm loans which shall exceed by more than 1% the rate established by the last series of Farm Loan Bonds issued by them, which rate shall not exceed 5% per annum.
45Provisions for the issue of Farm Loan Bonds secured by first mortgages on farm lands or United States bonds, as collateral, are made for Federal Land Banks and Joint Stock Land Banks; in each case the issue is made subject to the approval of the Federal Farm Loan Board. The farm loan mortgages, or United States bonds, which constitute the collateral security for the bonds, must be deposited with the Farm Loan Registrar.
46Section 26 of the act provides as follows: "That every Federal land bank and every national farm loan association, including the capital and reserve or surplus therein and the income derived therefrom, shall be exempt from Federal, State, municipal, and local taxation, except taxes upon real estate held, purchased, or taken by said bank or association under the provisions of section eleven and section thirteen of this Act. First mortgages executed to Federal land banks, or to joint stock land banks, and farm loan bonds issued under the provisions of this Act, shall be deemed and held to be instrumentalities of the Government of the United States, and as such they and the income derived therefrom shall be exempt from Federal, State, municipal, and local taxation.
4748"Nothing herein shall prevent the shares in any joint stock land bank from being included in the valuation of the personal property of the owner or holder of such shares, in assessing taxes imposed by authority of the State within which the bank is located; but such assessment and taxation shall be in manner and subject to the conditions and limitations contained in section fifty-two [208] hundred and nineteen of the Revised Statutes with reference to the shares of national banking associations.
"Nothing herein shall be construed to exempt the real property of Federal and joint stock land banks and national farm loan associations from either State, county, or municipal taxes, to the same extent, according to its value, as other real property is taxed."
Since the decision of the great cases of McCulloch v. Maryland, 4 Wheat. 316, and Osborn v. Bank, 9 Wheat. 738, it is no longer an open question that Congress may establish banks for national purposes, only a small part of the capital of which is held by the Government, and a majority of the ownership in which is represented by shares of capital stock privately owned and held; the principal business of such banks being private banking conducted with the usual methods of such business. While the express power to create a bank or incorporate one is not found in the Constitution, the court, speaking by Chief Justice Marshall, in McCulloch v. Maryland, found authority so to do in the broad general powers conferred by the Constitution upon the Congress to levy and collect taxes, to borrow money, to regulate commerce, to pay the public debts, to declare and conduct war, to raise and support armies, and to provide and maintain a navy, etc. Congress it was held had authority to use such means as were deemed appropriate to exercise the great powers of the Government by virtue of Article I, § 8, cl. 18, of the Constitution granting to Congress the right to make all laws necessary and proper to make the grant effectual. In First National Bank v. Union Trust Co., 244 U.S. 416, 419, the Chief Justice, speaking for the court, after reviewing McCulloch v. Maryland, and Osborn v. Bank, and considering the power given to Congress to pass laws to make the specific powers granted effectual, said:
49"In terms it was pointed out that this broad authority [209] was not stereotyped as of any particular time but endured, thus furnishing a perpetual and living sanction to the legislative authority within the limits of a just discretion enabling it to take into consideration the changing wants and demands of society and to adopt provisions appropriate to meet every situation which it was deemed required to be provided for."
50That the formation of the bank was required in the judgment of the Congress for the fiscal operations of the Government, was a principal consideration upon which Chief Justice Marshall rested the authority to create the bank; and for that purpose being an appropriate measure in the judgment of the Congress, it was held not to be within the authority of the court to question the conclusion reached by the legislative branch of the Government.
51Upon the authority of McCulloch v. Maryland, and Osborn v. Bank, the national banking system was established, and upon them this court has rested the constitutionality of the legislation establishing such banks. Farmers' & Mechanics' National Bank v. Dearing, 91 U.S. 29, 33, 34.
52Congress has seen fit in § 6 of the act to make both classes of banks, when designated for that purpose by the Secretary of the Treasury, depositaries of public money, except receipts from customs, under regulations to be prescribed by the Secretary of the Treasury, and has authorized their employment as financial agents of the Government, and the banks are required to perform such reasonable duties, as depositaries of public moneys and financial agents as may be required of them. The Secretary of the Treasury shall require of the Federal Land Banks and the Joint Stock Land Banks, thus designated, satisfactory security, by the deposit of United States bonds or otherwise, for the safe-keeping and prompt payment of the public money deposited with them, and [210] for the faithful performance of their duties as the financial agents of the Government.
53Section 6 also provides that no government funds deposited under the provisions of the section shall be invested in mortgage loans or Farm Loan Bonds.
54It is said that the power to designate these banks as such depositaries has not been exercised by the Government, and that the Federal Land Banks have acted as federal agents only in the case of loans of money for seed purposes made in the summer of 1918, to which we have already referred. But the existence of the power under the Constitution is not determined by the extent of the exercise of the authority conferred under it. Congress declared it necessary to create these fiscal agencies, and to make them authorized depositaries of public money. Its power to do so is no longer open to question.
55But, it is urged, the attempt to create these federal agencies, and to make these banks fiscal agents and public depositaries of the Government, is but a pretext. But nothing is better settled by the decisions of this court than that when Congress acts within the limits of its constitutional authority, it is not the province of the judicial branch of the Government to question its motives. Veazie Bank v. Fenno, 8 Wall. 533, 541; McCray v. United States, 195 U.S. 27; Flint v. Stone Tracy Co., 220 U.S. 107, 147, 153, 156, and cases cited.
56That Congress has seen fit, in making these banks fiscal agencies and depositaries of public moneys, to grant to them banking powers of a limited character, in nowise detracts from the authority of Congress to use them for the governmental purposes named, if it sees fit to do so. A bank may be organized with or without the authority to issue currency. It may be authorized to receive deposits in only a limited way. Speaking generally, a bank is a moneyed institution to facilitate the borrowing, lending and caring for money. But whether [211] technically banks, or not, these organizations may serve the governmental purposes declared by Congress in their creation. Furthermore, these institutions are organized to serve as a market for United States bonds. Not less than 5% of the capital of the Federal Land Banks, for which stock is outstanding to Farm Loan Associations, is required to be invested in United States bonds. Both kinds of banks are empowered to buy and sell United States bonds.
57In First National Bank v. Union Trust Co., supra, this court sustained the power of Congress to enable a national bank to transact business, which, by itself considered, might be beyond the power of Congress to authorize. In that case it was held to be within the authority of Congress to permit national banks to exercise, by permission of the Federal Reserve Board, when not in contravention of local law, the office of trustee, executor, administrator or registrar of stocks or bonds.
58We, therefore, conclude that the creation of these banks, and the grant of authority to them to act for the Government as depositaries of public moneys and purchasers of Government bonds, brings them within the creative power of Congress although they may be intended, in connection with other privileges and duties, to facilitate the making of loans upon farm security at low rates of interest. This does not destroy the validity of these enactments any more than the general banking powers destroyed the authority of Congress to create the United States Bank, or the authority given to national banks to carry on additional activities, destroyed the authority of Congress to create those institutions.
59In the brief filed upon reargument counsel for the appellant seem to admit the power of Congress to appropriate money for the direct purposes named, and in that brief they say: "Tax exemption is the real issue sought to be settled here." Deciding, as we do, that these institutions [212] have been created by Congress within the exercise of its legitimate authority, we think the power to make the securities here involved tax exempt necessarily follows. This principle was settled in McCulloch v. Maryland, and Osborn v. Bank, supra.
60That the Federal Government can, if it sees fit to do so, exempt such securities from taxation, seems obvious upon the clearest principles. But, it is said to be an invasion of state authority to extend the tax exemption so as to restrain the power of the State. Of a similar contention made in McCulloch v. Maryland, Chief Justice Marshall uttered his often quoted statement: "That the power to tax involves the power to destroy; that the power to destroy may defeat and render useless the power to create; that there is a plain repugnance in conferring on one government a power to control the constitutional measures of another, which other, with respect to those very measures, is declared to be supreme over that which exerts the control, are propositions not to be denied." 4 Wheat. 431.
61The same principle has been recognized in the National Bank Cases declaring the power of the States to tax the property and franchises of national banks only to the extent authorized by the laws of Congress. Owensboro National Bank v. Owensboro, 173 U.S. 664, involved the validity of a franchise tax in Kentucky on national banks. In that case this court declared (pp. 668, 669) that the States were wholly without power to levy any tax directly or indirectly upon national banks, their property, assets or franchises, except so far as the permissive legislation of Congress allowed such taxation; and the court declared that the right granted to tax the real estate of such banks, and the shares in the names of the shareholders, constituted the extent of the permission given by Congress, and any tax beyond these was declared to be void.
62[213] In Farmers & Mechanics Savings Bank v. Minnesota, 232 U.S. 516, this court held that a State may not tax bonds issued by the municipality of a territory; that to tax such bonds as property in the hands of the holder is, in the last analysis, an imposition upon the right of a municipality to issue them.
63The exercise of such taxing power by the States might be so used as to hamper and destroy the exercise of authority conferred by Congress, and this justifies the exemption. If the States can tax these bonds they may destroy the means provided for obtaining the necessary funds for the future operation of the banks. With the wisdom and policy of this legislation we have nothing to do. Ours is only the function of ascertaining whether Congress in the creation of the banks, and in exempting these securities from taxation, federal and state, has acted within the limits of its constitutional authority. For the reasons stated, we think the contention of the Government, and of the appellees, that these banks are constitutionally organized and the securities here involved legally exempted from taxation, must be sustained.
64It follows that the decree of the District Court is
65Affirmed.
66No doubt it is desirable that the question raised in this case should be set at rest, but that can be done by the Courts of the United States only within the limits of the jurisdiction conferred upon them by the Constitution and the laws of the United States. As this suit was brought by a citizen of Missouri against a Missouri corporation the [214] single ground upon which the jurisdiction of the District Court can be maintained is that the suit "arises under the Constitution or laws of the United States" within the meaning of § 24 of the Judicial Code. I am of opinion that this case does not arise in that way and therefore that the bill should have been dismissed.
69It is evident that the cause of action arises not under any law of the United States but wholly under Missouri law. The defendant is a Missouri corporation and the right claimed is that of a stockholder to prevent the directors from doing an act, that is, making an investment, alleged to be contrary to their duty. But the scope of their duty depends upon the charter of their corporation and other laws of Missouri. If those laws had authorized the investment in terms the plaintiff would have had no case, and this seems to me to make manifest what I am unable to deem even debatable, that, as I have said, the cause of action arises wholly under Missouri law. If the Missouri law authorizes or forbids the investment according to the determination of this Court upon a point under the Constitution or acts of Congress, still that point is material only because the Missouri law saw fit to make it so. The whole foundation of the duty is Missouri law, which at its sole will incorporated the other law as it might incorporate a document. The other law or document depends for its relevance and effect not on its own force but upon the law that took it up, so I repeat once more the cause of action arises wholly from the law of the State.
70But it seems to me that a suit cannot be said to arise under any other law than that which creates the cause of action. It may be enough that the law relied upon creates a part of the cause of action although not the whole, as held in Osborn v. Bank of the United States, 9 Wheat. 738, 819-823, which perhaps is all that is meant by the less guarded expressions in Cohens v. Virginia, 6 Wheat. 264, 379. I am content to assume this to be so, although the Osborn Case [215] has been criticized and regretted. But the law must create at least a part of the cause of action by its own force, for it is the suit, not a question in the suit, that must arise under the law of the United States. The mere adoption by a state law of a United States law as a criterion or test, when the law of the United States has no force proprio vigore, does not cause a case under the state law to be also a case under the law of the United States, and so it has been decided by this Court again and again. Miller v. Swann, 150 U.S. 132, 136, 137; Louisville & Nashville R.R. Co v. Western Union Telegraph Co., 237 U.S. 300, 303. See also Shoshone Mining Co. v. Rutter, 177 U.S. 505, 508, 509.
71I find nothing contrary to my views in Brushaber v. Union Pacific R.R. Co., 240 U.S. 1, 10. It seems to me plain that the objection that I am considering was not before the mind of the Court or the subject of any of its observations, if open. I am confirmed in my view of that case by the fact that in the next volume of reports is a decision, reached not without discussion and with but a single dissent, that "a suit arises under the law that creates the cause of action." That was the ratio decidendi of American Well Works Co. v. Layne & Bowler Co., 241 U.S. 257, 260. I know of no decisions to the contrary and see no reason for overruling it now.
72[1] At the first hearing Mr. Solicitor General King and Mr. W.G. McAdoo, by leave of court, filed a brief on behalf of the United States as amici curiae.
Read this AFTER you've read the cheat sheet which will walk you through most of the case.
Supreme Court of United States.
CERTIORARI TO THE UNITED STATES COURT OF APPEALS FOR THE SIXTH CIRCUIT
8[805] Frank C. Woodside III argued the cause for petitioner. With him on the briefs was Christine L. McBroom.
9Stanley M. Chesley argued the cause and filed a brief for respondents.
10The question presented is whether the incorporation of a federal standard in a state-law private action, when Congress has intended that there not be a federal private action for violations of that federal standard, makes the action one "arising under the Constitution, laws, or treaties of the United States," 28 U. S. C. § 1331.
12The Thompson respondents are residents of Canada and the MacTavishes reside in Scotland. They filed virtually identical complaints against petitioner, a corporation, that manufactures and distributes the drug Bendectin. The complaints were filed in the Court of Common Pleas in Hamilton County, Ohio. Each complaint alleged that a child was born with multiple deformities as a result of the mother's ingestion of Bendectin during pregnancy. In five of the six counts, the recovery of substantial damages was requested on common-law theories of negligence, breach of warranty, strict liability, fraud, and gross negligence. In Count IV, respondents alleged that the drug Bendectin was "misbranded" in violation of the Federal Food, Drug, and Cosmetic Act (FDCA), 52 Stat. 1040, as amended, 21 U. S. C. § 301 et seq. (1982 ed. and Supp. III), because its labeling did not provide adequate [806] warning that its use was potentially dangerous. Paragraph 26 alleged that the violation of the FDCA "in the promotion" of Bendectin "constitutes a rebuttable presumption of negligence." Paragraph 27 alleged that the "violation of said federal statutes directly and proximately caused the injuries suffered" by the two infants. App. 22, 32.
14Petitioner filed a timely petition for removal from the state court to the Federal District Court alleging that the action was "founded, in part, on an alleged claim arising under the laws of the United States."[1] After removal, the two cases were consolidated. Respondents filed a motion to remand to the state forum on the ground that the federal court lacked subject-matter jurisdiction. Relying on our decision in Smith v. Kansas City Title & Trust Co., 255 U. S. 180 (1921), the District Court held that Count IV of the complaint alleged a cause of action arising under federal law and denied the motion to remand. It then granted petitioner's motion to dismiss on forum non conveniens grounds.
15The Court of Appeals for the Sixth Circuit reversed. 766 F. 2d 1005 (1985). After quoting one sentence from the concluding paragraph in our recent opinion in Franchise Tax Board v. Construction Laborers Vacation Trust, 463 U. S. 1 (1983),[2] and noting "that the FDCA does not create or imply [807] a private right of action for individuals injured as a result of violations of the Act," it explained:
16"Federal question jurisdiction would, thus, exist only if plaintiffs' right to relief depended necessarily on a substantial question of federal law. Plaintiffs' causes of action referred to the FDCA merely as one available criterion for determining whether Merrell Dow was negligent. Because the jury could find negligence on the part of Merrell Dow without finding a violation of the FDCA, the plaintiffs' causes of action did not depend necessarily upon a question of federal law. Consequently, the causes of action did not arise under federal law and, therefore, were improperly removed to federal court." 766 F. 2d, at 1006.17
We granted certiorari, 474 U. S. 1004 (1985), and we now affirm.
18Article III of the Constitution gives the federal courts power to hear cases "arising under" federal statutes.[3] That grant of power, however, is not self-executing, and it was not until the Judiciary Act of 1875 that Congress gave the federal courts general federal-question jurisdiction.[4] Although the constitutional meaning of "arising under" may extend to all cases in which a federal question is "an ingredient" of the action, Osborn v. Bank of the United States, 9 Wheat. 738, 823 (1824), we have long construed the statutory grant of federal-question jurisdiction as conferring a more limited power. [808] Verlinden B. V. v. Central Bank of Nigeria, 461 U. S. 480, 494-495 (1983); Romero v. International Terminal Operating Co., 358 U. S. 354, 379 (1959).
20Under our longstanding interpretation of the current statutory scheme, the question whether a claim "arises under" federal law must be determined by reference to the "well-pleaded complaint." Franchise Tax Board, 463 U. S., at 9-10. A defense that raises a federal question is inadequate to confer federal jurisdiction. Louisville & Nashville R. Co. v. Mottley, 211 U. S. 149 (1908). Since a defendant may remove a case only if the claim could have been brought in federal court, 28 U. S. C. § 1441(b), moreover, the question for removal jurisdiction must also be determined by reference to the "well-pleaded complaint."
21As was true in Franchise Tax Board, supra, the propriety of the removal in this case thus turns on whether the case falls within the original "federal question" jurisdiction of the federal courts. There is no "single, precise definition" of that concept; rather, "the phrase `arising under' masks a welter of issues regarding the interrelation of federal and state authority and the proper management of the federal judicial system." Id., at 8.
22This much, however, is clear. The "vast majority" of cases that come within this grant of jurisdiction are covered by Justice Holmes' statement that a " `suit arises under the law that creates the cause of action.' " Id., at 8-9, quoting American Well Works Co. v. Layne & Bowler Co., 241 U. S. 257, 260 (1916). Thus, the vast majority of cases brought under the general federal-question jurisdiction of the federal courts are those in which federal law creates the cause of action.
23We have, however, also noted that a case may arise under federal law "where the vindication of a right under state law necessarily turned on some construction of federal law." [809] Franchise Tax Board, 463 U. S., at 9.[5] Our actual holding in Franchise Tax Board demonstrates that this statement must be read with caution; the central issue presented in that case turned on the meaning of the Employee Retirement Income Security Act of 1974, 29 U. S. C. § 1001 et seq. (1982 ed. and Supp. III), but we nevertheless concluded that federal jurisdiction was lacking.
24This case does not pose a federal question of the first kind; respondents do not allege that federal law creates any of the causes of action that they have asserted.[6] This case thus poses what Justice Frankfurter called the "litigation-provoking problem," Textile Workers v. Lincoln Mills, 353 [810] U. S. 448, 470 (1957) (dissenting opinion) — the presence of a federal issue in a state-created cause of action.
25In undertaking this inquiry into whether jurisdiction may lie for the presence of a federal issue in a nonfederal cause of action, it is, of course, appropriate to begin by referring to our understanding of the statute conferring federal-question jurisdiction. We have consistently emphasized that, in exploring the outer reaches of § 1331, determinations about federal jurisdiction require sensitive judgments about congressional intent, judicial power, and the federal system. "If the history of the interpretation of judiciary legislation teaches us anything, it teaches the duty to reject treating such statutes as a wooden set of self-sufficient words. . . . The Act of 1875 is broadly phrased, but it has been continuously construed and limited in the light of the history that produced it, the demands of reason and coherence, and the dictates of sound judicial policy which have emerged from the Act's function as a provision in the mosaic of federal judiciary legislation." Romero v. International Terminal Operating Co., 358 U. S., at 379. In Franchise Tax Board, we forcefully reiterated this need for prudence and restraint in the jurisdictional inquiry: "We have always interpreted what Skelly Oil [Co. v. Phillips Petroleum Co., 339 U. S. 667, 673 (1950)] called `the current of jurisdictional legislation since the Act of March 3, 1875' . . . with an eye to practicality and necessity." 463 U. S., at 20.
26In this case, both parties agree with the Court of Appeals' conclusion that there is no federal cause of action for FDCA violations. For purposes of our decision, we assume that this is a correct interpretation of the FDCA. Thus, as the case comes to us, it is appropriate to assume that, under the settled framework for evaluating whether a federal cause of action lies, some combination of the following factors is present: (1) the plaintiffs are not part of the class for whose special benefit the statute was passed; (2) the indicia of legislative [811] intent reveal no congressional purpose to provide a private cause of action; (3) a federal cause of action would not further the underlying purposes of the legislative scheme; and (4) the respondents' cause of action is a subject traditionally relegated to state law.[7] In short, Congress did not intend a private federal remedy for violations of the statute that it enacted.
27This is the first case in which we have reviewed this type of jurisdictional claim in light of these factors. That this is so is not surprising. The development of our framework for determining whether a private cause of action exists has proceeded only in the last 11 years, and its inception represented a significant change in our approach to congressional silence on the provision of federal remedies.[8]
28The recent character of that development does not, however, diminish its importance. Indeed, the very reasons for the development of the modern implied remedy doctrine — the "increased complexity of federal legislation and the increased volume of federal litigation," as well as "the desirability of a more careful scrutiny of legislative intent," Merrill Lynch, Pierce, Fenner & Smith, Inc. v. Curran, 456 U. S. 353, 377 (1982) (footnote omitted) — are precisely the kind of considerations that should inform the concern for "practicality and necessity" that Franchise Tax Board advised for the construction of § 1331 when jurisdiction is asserted [812] because of the presence of a federal issue in a state cause of action.
29The significance of the necessary assumption that there is no federal private cause of action thus cannot be overstated. For the ultimate import of such a conclusion, as we have repeatedly emphasized, is that it would flout congressional intent to provide a private federal remedy for the violation of the federal statute.[9] We think it would similarly flout, or at least undermine, congressional intent to conclude that the federal courts might nevertheless exercise federal-question jurisdiction and provide remedies for violations of that federal statute solely because the violation of the federal statute is said to be a "rebuttable presumption" or a "proximate cause" under state law, rather than a federal action under federal law.[10]
30Petitioner advances three arguments to support its position that, even in the face of this congressional preclusion of a federal cause of action for a violation of the federal statute, federal-question jurisdiction may lie for the violation of the federal statute as an element of a state cause of action.
32First, petitioner contends that the case represents a straightforward application of the statement in Franchise Tax Board that federal-question jurisdiction is appropriate when "it appears that some substantial, disputed question of federal law is a necessary element of one of the well-pleaded state claims." 463 U. S., at 13. Franchise Tax Board, however, did not purport to disturb the long-settled understanding that the mere presence of a federal issue in a state cause of action does not automatically confer federal-question jurisdiction.[11] Indeed, in determining that federal-question jurisdiction was not appropriate in the case before us, we stressed Justice Cardozo's emphasis on principled, pragmatic distinctions: " `What is needed is something of that common-sense accommodation of judgment to kaleidoscopic situations which characterizes the law in its treatment of causation . . . a selective process which picks the substantial causes out of the web [814] and lays the other ones aside.' " Id., at 20-21 (quoting Gully v. First National Bank, 299 U. S. 109, 117-118 (1936)).
33Far from creating some kind of automatic test, Franchise Tax Board thus candidly recognized the need for careful judgments about the exercise of federal judicial power in an area of uncertain jurisdiction. Given the significance of the assumed congressional determination to preclude federal private remedies, the presence of the federal issue as an element of the state tort is not the kind of adjudication for which jurisdiction would serve congressional purposes and the federal system. This conclusion is fully consistent with the very sentence relied on so heavily by petitioner. We simply conclude that the congressional determination that there should be no federal remedy for the violation of this federal statute is tantamount to a congressional conclusion that the presence of a claimed violation of the statute as an element of a state cause of action is insufficiently "substantial" to confer federal-question jurisdiction.[12]
34[815] Second, petitioner contends that there is a powerful federal interest in seeing that the federal statute is given uniform interpretations, and that federal review is the best way of insuring such uniformity. In addition to the significance of the congressional decision to preclude a federal remedy, we do [816] not agree with petitioner's characterization of the federal interest and its implications for federal-question jurisdiction. To the extent that petitioner is arguing that state use and interpretation of the FDCA pose a threat to the order and stability of the FDCA regime, petitioner should be arguing, not that federal courts should be able to review and enforce state FDCA-based causes of action as an aspect of federal-question jurisdiction, but that the FDCA pre-empts state-court jurisdiction over the issue in dispute.[13] Petitioner's concern about the uniformity of interpretation, moreover, is considerably mitigated by the fact that, even if there is no original district court jurisdiction for these kinds of action, this Court retains power to review the decision of a federal issue in a state cause of action.[14]
35Finally, petitioner argues that, whatever the general rule, there are special circumstances that justify federal-question jurisdiction in this case. Petitioner emphasizes that it is unclear whether the FDCA applies to sales in Canada and Scotland; there is, therefore, a special reason for having a federal [817] court answer the novel federal question relating to the extra-territorial meaning of the Act. We reject this argument. We do not believe the question whether a particular claim arises under federal law depends on the novelty of the federal issue. Although it is true that federal jurisdiction cannot be based on a frivolous or insubstantial federal question, "the interrelation of federal and state authority and the proper management of the federal judicial system," Franchise Tax Board, 463 U. S., at 8, would be ill served by a rule that made the existence of federal-question jurisdiction depend on the district court's case-by-case appraisal of the novelty of the federal question asserted as an element of the state tort. The novelty of an FDCA issue is not sufficient to give it status as a federal cause of action; nor should it be sufficient to give a state-based FDCA claim status as a jurisdiction-triggering federal question.[15]
36We conclude that a complaint alleging a violation of a federal statute as an element of a state cause of action, when Congress has determined that there should be no private, federal cause of action for the violation, does not state a claim "arising under the Constitution, laws, or treaties of the United States." 28 U. S. C. § 1331.
38The judgment of the Court of Appeals is affirmed.
39It is so ordered.
40Article III, § 2, of the Constitution provides that the federal judicial power shall extend to "all Cases, in Law and Equity, arising under this Constitution, the Laws of the United States, and Treaties made, or which shall be made, under their Authority." We have long recognized the great breadth of this grant of jurisdiction, holding that there is federal jurisdiction whenever a federal question is an "ingredient" of the action, Osborn v. Bank of the United States, 9 Wheat. 738, 823 (1824), and suggesting that there may even be jurisdiction simply because a case involves "potential federal questions," Textile Workers v. Lincoln Mills, 353 U. S. 448, 471 (1957) (Frankfurter, J., dissenting); see also Osborn, supra, at 824; Martin v. Hunter's Lessee, 1 Wheat. 304 (1816); Pacific Railroad Removal Cases, 115 U. S. 1 (1885); Verlinden B. V. v. Central Bank of Nigeria, 461 U. S. 480, 492-493 (1983).
42Title 28 U. S. C. § 1331 provides, in language that parrots the language of Article III, that the district courts shall have original jurisdiction "of all civil actions arising under the Constitution, laws, or treaties of the United States." Although this language suggests that Congress intended in § 1331 to confer upon federal courts the full breadth of permissible "federal question" jurisdiction (an inference that is supported by the contemporary evidence, see Franchise Tax Board v. Construction Laborers Vacation Trust, 463 U. S. 1, 8, n. 8 (1983); Forrester, The Nature of a "Federal Question," 16 Tulane L. Rev. 362, 374-376 (1942); Shapiro, Jurisdiction and Discretion, 60 N. Y. U. L. Rev. 543, 568 (1985)), § 1331 has been construed more narrowly than its constitutional counterpart. See Verlinden B. V., supra, at 494-495; Romero v. International Terminal Operating Co., 358 U. S. 354, 379 (1959). Nonetheless, given the language of the statute and its close relation to the constitutional grant of federal-question jurisdiction, limitations on federal-question jurisdiction under § 1331 must be justified by careful consideration of the reasons [819] underlying the grant of jurisdiction and the need for federal review. Ibid. I believe that the limitation on federal jurisdiction recognized by the Court today is inconsistent with the purposes of § 1331. Therefore, I respectfully dissent.
43While the majority of cases covered by § 1331 may well be described by Justice Holmes' adage that "[a] suit arises under the law that creates the cause of action," American Well Works Co. v. Layne & Bowler Co., 241 U. S. 257, 260 (1916), it is firmly settled that there may be federal-question jurisdiction even though both the right asserted and the remedy sought by the plaintiff are state created. See C. Wright, Federal Courts § 17, pp. 95-96 (4th ed. 1983) (hereinafter Wright); M. Redish, Federal Jurisdiction: Tensions in the Allocation of Judicial Power 64-71 (1980) (hereinafter Redish). The rule as to such cases was stated in what Judge Friendly described as "[t]he path-breaking opinion" in Smith v. Kansas City Title & Trust Co., 255 U. S. 180 (1921). T. B. Harms Co. v. Eliscu, 339 F. 2d 823, 827 (CA2 1964). In Smith, a shareholder of the defendant corporation brought suit in the federal court to enjoin the defendant from investing corporate funds in bonds issued under the authority of the Federal Farm Loan Act. The plaintiff alleged that Missouri law imposed a fiduciary duty on the corporation to invest only in bonds that were authorized by a valid law and argued that, because the Farm Loan Act was unconstitutional, the defendant could not purchase bonds issued under its authority. Although the cause of action was wholly state created, the Court held that there was original federal jurisdiction over the case:
45"The general rule is that where it appears from the bill or statement of the plaintiff that the right to relief depends upon the construction or application of the Constitution or laws of the United States, and that such federal claim is not merely colorable, and rests upon a reasonable foundation, the District Court has jurisdiction [820] under [the statute granting federal question jurisdiction]." 255 U. S., at 199.46
The continuing vitality of Smith is beyond challenge. We have cited it approvingly on numerous occasions, and reaffirmed its holding several times — most recently just three Terms ago by a unanimous Court in Franchise Tax Board v. Construction Laborers Vacation Trust, supra, at 9. See American Bank & Trust Co. v. Federal Reserve Bank of Atlanta, 256 U. S. 350, 357 (1921); Bell v. Hood, 327 U. S. 678, 685 (1946); Association of Westinghouse Salaried Employees v. Westinghouse Electric Corp., 348 U. S. 437, 450, and n. 18 (1955) (plurality opinion); Machinists v. Central Airlines, Inc., 372 U. S. 682, 696 (1963); Duke Power Co. v. Carolina Environmental Study Group, Inc., 438 U. S. 59, 70 (1978). See also Ashwander v. TVA, 297 U. S. 288, 356 (1936) (separate opinion of McReynolds, J.); Textile Workers v. Lincoln Mills, supra, at 470 (Frankfurter, J., dissenting); Wheeldin v. Wheeler, 373 U. S. 647, 659 (1963) (BRENNAN, J., dissenting). Cf. Gully v. First National Bank, 299 U. S. 109, 112 (1936) ("To bring a case within [§ 1331], a right or immunity created by the Constitution or laws of the United States must be an element, and an essential one, of the plaintiff's cause of action"). Moreover, in addition to Judge Friendly's authoritative opinion in T. B. Harms Co. v. Eliscu, supra, at 827, Smith has been widely cited and followed in the lower federal courts. See, e. g., Hanes Corp. v. Millard, 174 U. S. App. D. C. 253, 263, n. 8, 531 F. 2d 585, 595, n. 8 (1976); Mungin v. Florida East Coast R. Co., 416 F. 2d 1169, 1176-1177 (CA5 1969); Ivy Broadcasting Co. v. American Tel. & Tel. Co., 391 F. 2d 486, 492 (CA2 1968); Warrington Sewer Co. v. Tracy, 463 F. 2d 771, 772 (CA3 1972) (per curiam); New York by Abrams v. Citibank, N. A., 537 F. Supp. 1192, 1196 (SDNY 1982); Kravitz v. Homeowners Warranty Corp., 542 F. Supp. 317, 319 (ED Pa. 1982). See also Stone & Webster Engineering Corp. v. Ilsley, 690 F. 2d 323 (CA2 1982); Christopher v. Cavallo, 662 F. 2d 1082 (CA4 1981); Mountain Fuel Supply Co. v. Johnson Oil Co., 586 F. 2d 1375 (CA10 1978), [821] cert. denied, 441 U. S. 952 (1979); Garrett v. Time-D. C., Inc., 502 F. 2d 627 (CA9 1974), cert. denied, 421 U. S. 913 (1975); Sweeney v. Abramovitz, 449 F. Supp. 213 (Conn. 1978). Furthermore, the principle of the Smith case has been recognized and endorsed by most commentators as well. Redish 67, 69; American Law Institute, Study of the Division of Jurisdiction Between State and Federal Courts 178 (1969) (hereinafter ALI); Wright § 17, at 96; P. Bator, P. Mishkin, D. Shapiro, & H. Wechsler, Hart & Wechsler's The Federal Courts and the Federal System 889 (2d ed., 1973); Mishkin, The Federal "Question" in the District Courts, 53 Colum. L. Rev. 157, 166 (1953); Wechsler, Federal Jurisdiction and the Revision of the Judicial Code, 13 Law & Contemp. Prob. 216, 225 (1948).[16]
47[822] There is, to my mind, no question that there is federal jurisdiction over the respondents' fourth cause of action under the rule set forth in Smith and reaffirmed in Franchise Tax [823] Board. Respondents pleaded that petitioner's labeling of the drug Bendectin constituted "misbranding" in violation of §§ 201 and 502(f)(2) and (j) of the Federal Food, Drug, and Cosmetic Act (FDCA), 52 Stat. 1040, as amended, 21 U. S. C. § 301 et seq. (1982 ed. and Supp. III), and that this violation "directly and proximately caused" their injuries. App. 21-22 (Thompson complaint), 31-32 (MacTavish complaint). Respondents asserted in the complaint that this violation established petitioner's negligence per se and entitled them to recover damages without more. Ibid. No other basis for finding petitioner negligent was asserted in connection with this claim. As pleaded, then, respondents' "right to relief depend[ed] upon the construction or application of the Constitution or laws of the United States." Smith, 255 U. S., at 199; see also Franchise Tax Board, 463 U. S., at 28 (there is federal jurisdiction under § 1331 where the plaintiff's right to relief "necessarily depends" upon resolution of a federal question).[17] Furthermore, although petitioner disputes its liability under the FDCA, it concedes that respondents' claim that petitioner violated the FDCA is "colorable, and rests upon a reasonable foundation." Smith, supra, at 199.[18] [824] Of course, since petitioner must make this concession to prevail in this Court, it need not be accepted at face value. However, independent examination of respondents' claim substantiates the conclusion that it is neither frivolous nor meritless. As stated in the complaint, a drug is "misbranded" under the FDCA if "the labeling or advertising fails to reveal facts material . . . with respect to consequences which may result from the use of the article to which the labeling or advertising relates . . . ." 21 U. S. C. § 321(n). Obviously, the possibility that a mother's ingestion of Bendectin during pregnancy could produce malformed children is material. Petitioner's principal defense is that the Act does not govern the branding of drugs that are sold in foreign countries. It is certainly not immediately obvious whether this argument is correct. Thus, the statutory question is one which "discloses a need for determining the meaning or application of [the FDCA]," T. B. Harms Co. v. Eliscu, 339 F. 2d, at 827, and the claim raised by the fourth cause of action is one "arising under" federal law within the meaning of § 1331.
48The Court apparently does not disagree with any of this — except, of course, for the conclusion. According to the Court, if we assume that Congress did not intend that there be a private federal cause of action under a particular federal law (and, presumably, a fortiori if Congress' decision not to create a private remedy is express), we must also assume that Congress did not intend that there be federal jurisdiction over a state cause of action that is determined by that federal law. Therefore, assuming — only because the parties [825] have made a similar assumption — that there is no private cause of action under the FDCA,[19] the Court holds that there is no federal jurisdiction over the plaintiffs' claim:
50"The significance of the necessary assumption that there is no federal private cause of action thus cannot be overstated. For the ultimate import of such a conclusion, as we have repeatedly emphasized, is that it would flout congressional intent to provide a private federal remedy for the violation of the federal statute. We think it would similarly flout, or at least undermine, congressional intent to conclude that the federal courts might nevertheless exercise federal-question jurisdiction and provide remedies for violations of that federal statute solely because the violation of the federal statute is said to be a `rebuttable presumption' or a `proximate cause' under state law, rather than a federal action under federal law." Ante, at 812 (footnotes omitted).51
The Court nowhere explains the basis for this conclusion. Yet it is hardly self-evident. Why should the fact that Congress chose not to create a private federal remedy mean that Congress would not want there to be federal jurisdiction to adjudicate a state claim that imposes liability for violating the federal law? Clearly, the decision not to provide a private federal remedy should not affect federal jurisdiction unless the reasons Congress withholds a federal remedy are also reasons for withholding federal jurisdiction. Thus, it is necessary [826] to examine the reasons for Congress' decisions to grant or withhold both federal jurisdiction and private remedies, something the Court has not done.
52In the early days of our Republic, Congress was content to leave the task of interpreting and applying federal laws in the first instance to the state courts; with one short-lived exception,[20] Congress did not grant the inferior federal courts original jurisdiction over cases arising under federal law until 1875. Judiciary Act of 1875, ch. 137, § 1, 18 Stat. 470. The reasons Congress found it necessary to add this jurisdiction to the district courts are well known. First, Congress recognized "the importance, and even necessity of uniformity of decisions throughout the whole United States, upon all subjects within the purview of the constitution." Martin v. Hunter's Lessee, 1 Wheat., at 347-348 (Story, J.) (emphasis in original). See also, Comment, Federal Preemption, Removal Jurisdiction, and the Well-Pleaded Complaint Rule, 51 U. Chi. L. Rev. 634, 636 (1984) (hereinafter Comment); D. Currie, Federal Courts 160 (3d ed. 1982) (hereinafter Currie). Concededly, because federal jurisdiction is not always exclusive and because federal courts may disagree with one another, absolute uniformity has not been obtained even under § 1331. However, while perfect uniformity may not have been achieved, experience indicates that the availability of a federal forum in federal-question cases has done much to advance that goal. This, in fact, was the conclusion of the American Law Institute's Study of the Division of Jurisdiction Between State and Federal Courts. ALI 164-168.
54In addition, § 1331 has provided for adjudication in a forum that specializes in federal law and that is therefore more likely to apply that law correctly. Because federal-question [827] cases constitute the basic grist for federal tribunals, "[t]he federal courts have acquired a considerable expertness in the interpretation and application of federal law." Id., at 164-165. By contrast, "it is apparent that federal question cases must form a very small part of the business of [state] courts." Id., at 165. As a result, the federal courts are comparatively more skilled at interpreting and applying federal law, and are much more likely correctly to divine Congress' intent in enacting legislation.[21] See ibid.; Redish 71; Currie 160; Comment 636; Hornstein, Federalism, Judicial Power and the "Arising Under" Jurisdiction of the Federal Courts: A Hierarchical Analysis, 56 Ind. L. J. 563, 564-565 (1981).
55These reasons for having original federal-question jurisdiction explain why cases like this one and Smith — i. e., cases where the cause of action is a creature of state law, but an [828] essential element of the claim is federal — "arise under" federal law within the meaning of § 1331. Congress passes laws in order to shape behavior; a federal law expresses Congress' determination that there is a federal interest in having individuals or other entities conform their actions to a particular norm established by that law. Because all laws are imprecise to some degree, disputes inevitably arise over what specifically Congress intended to require or permit. It is the duty of courts to interpret these laws and apply them in such a way that the congressional purpose is realized. As noted above, Congress granted the district courts power to hear cases "arising under" federal law in order to enhance the likelihood that federal laws would be interpreted more correctly and applied more uniformly. In other words, Congress determined that the availability of a federal forum to adjudicate cases involving federal questions would make it more likely that federal laws would shape behavior in the way that Congress intended.
56By making federal law an essential element of a state-law claim, the State places the federal law into a context where it will operate to shape behavior: the threat of liability will force individuals to conform their conduct to interpretations of the federal law made by courts adjudicating the state-law claim. It will not matter to an individual found liable whether the officer who arrives at his door to execute judgment is wearing a state or a federal uniform; all he cares about is the fact that a sanction is being imposed — and may be imposed again in the future — because he failed to comply with the federal law. Consequently, the possibility that the federal law will be incorrectly interpreted in the context of adjudicating the state-law claim implicates the concerns that led Congress to grant the district courts power to adjudicate cases involving federal questions in precisely the same way as if it was federal law that "created" the cause of action. It therefore follows that there is federal jurisdiction under § 1331.
57The only remaining question is whether the assumption that Congress decided not to create a private cause of action alters this analysis in a way that makes it inappropriate to exercise original federal jurisdiction. According to the Court, "the very reasons for the development of the modern implied remedy doctrine" support the conclusion that, where the legislative history of a particular law shows (whether expressly or by inference) that Congress intended that there be no private federal remedy, it must also mean that Congress would not want federal courts to exercise jurisdiction over a state-law claim making violations of that federal law actionable. Ante, at 811. These reasons are " `the increased complexity of federal legislation,' " " `the increased volume of federal litigation,' " and " `the desirability of a more careful scrutiny of legislative intent.' " Ibid. (quoting Merrill Lynch, Pierce, Fenner & Smith, Inc. v. Curran, 456 U. S. 353, 377 (1982)).
59These reasons simply do not justify the Court's holding. Given the relative expertise of the federal courts in interpreting federal law, supra, at 826-827, the increased complexity of federal legislation argues rather strongly in favor of recognizing federal jurisdiction. And, while the increased volume of litigation may appropriately be considered in connection with reasoned arguments that justify limiting the reach of § 1331, I do not believe that the day has yet arrived when this Court may trim a statute solely because it thinks that Congress made it too broad.[22]
60[830] This leaves only the third reason: " `the desirability of a more careful scrutiny of legislative intent.' " Ante, at 811. I certainly subscribe to the proposition that the Court should consider legislative intent in determining whether or not there is jurisdiction under § 1331. But the Court has not examined the purposes underlying either the FDCA or § 1331 in reaching its conclusion that Congress' presumed decision not to provide a private federal remedy under the FDCA must be taken to withdraw federal jurisdiction over a private state remedy that imposes liability for violating the FDCA. Moreover, such an examination demonstrates not only that it is consistent with legislative intent to find that there is federal jurisdiction over such a claim, but, indeed, that it is the Court's contrary conclusion that is inconsistent with congressional intent.
61The enforcement scheme established by the FDCA is typical of other, similarly broad regulatory schemes. Primary responsibility for overseeing implementation of the Act has been conferred upon a specialized administrative agency, here the Food and Drug Administration (FDA).[23] Congress has provided the FDA with a wide-ranging arsenal of weapons to combat violations of the FDCA, including authority to obtain an ex parte court order for the seizure of goods subject to the Act, see 21 U. S. C. § 334, authority to initiate proceedings in a federal district court to enjoin continuing violations of the FDCA, see § 332, and authority to request a United States Attorney to bring criminal proceedings against violators, see § 333. See generally 1 J. O'Reilly, Food and Drug Administration, chs. 6-10 (1979 and Supp. 1985). Significantly, the FDA has no independent enforcement authority; final enforcement must come from the federal courts. [831] which have exclusive jurisdiction over actions under the FDCA. See §§ 332(a), 333, 334(a)(1). Thus, while the initial interpretive function has been delegated to an expert administrative body whose interpretations are entitled to considerable deference, final responsibility for interpreting the statute in order to carry out the legislative mandate belongs to the federal courts. Cf. Chevron U. S. A. Inc. v. Natural Resources Defense Council, Inc., 467 U. S. 837, 843, n. 9 (1984) ("The judiciary is the final authority on issues of statutory construction and must reject administrative constructions which are contrary to clear congressional intent").
62Given that Congress structured the FDCA so that all express remedies are provided by the federal courts, it seems rather strange to conclude that it either "flout[s]" or "undermine[s]" congressional intent for the federal courts to adjudicate a private state-law remedy that is based upon violating the FDCA. See ante, at 812. That is, assuming that a state cause of action based on the FDCA is not preempted, it is entirely consistent with the FDCA to find that it "arises under" federal law within the meaning of § 1331. Indeed, it is the Court's conclusion that such a state cause of action must be kept out of the federal courts that appears contrary to legislative intent inasmuch as the enforcement provisions of the FDCA quite clearly express a preference for having federal courts interpret the FDCA and provide remedies for its violation.
63It may be that a decision by Congress not to create a private remedy is intended to preclude all private enforcement. If that is so, then a state cause of action that makes relief available to private individuals for violations of the FDCA is pre-empted. But if Congress' decision not to provide a private federal remedy does not pre-empt such a state remedy, then, in light of the FDCA's clear policy of relying on the federal courts for enforcement, it also should not foreclose federal jurisdiction over that state remedy. Both § 1331 and the enforcement provisions of the FDCA reflect Congress' strong [832] desire to utilize the federal courts to interpret and enforce the FDCA, and it is therefore at odds with both these statutes to recognize a private state-law remedy for violating the FDCA but to hold that this remedy cannot be adjudicated in the federal courts.
64The Court's contrary conclusion requires inferring from Congress' decision not to create a private federal remedy that, while some private enforcement is permissible in state courts, it is "bad" if that enforcement comes from the federal courts. But that is simply illogical. Congress' decision to withhold a private right of action and to rely instead on public enforcement reflects congressional concern with obtaining more accurate implementation and more coordinated enforcement of a regulatory scheme. See National Railroad Passenger Corporation v. National Assn. of Railroad Passengers, 414 U. S. 453, 462-465 (1974); Holloway v. Bristol-Myers Corp., 158 U. S. App. D. C. 207, 218-220, 485 F. 2d 986, 997-999 (1973); Stewart & Sunstein, Public Programs and Private Rights, 95 Harv. L. Rev. 1193, 1208-1209 (1982). These reasons are closely related to the Congress' reasons for giving federal courts original federal-question jurisdiction. Thus, if anything, Congress' decision not to create a private remedy strengthens the argument in favor of finding federal jurisdiction over a state remedy that is not pre-empted.
65[1] App. 36-37. The petition also alleged that the action "is between citizens of a State and citizens or subjects of a foreign state." Id.,at 36. Because petitioner is a corporation with its principal place of business in Ohio, however, the removal was not proper unless the action was founded on a claim arising under federal law. Title 28 U. S. C. § 1441(b) provides:
6667"(b) Any civil action of which the district courts have original jurisdiction founded on a claim or right arising under the Constitution, treaties or laws of the United States shall be removable without regard to the citizenship or residence of the parties. Any other such action shall be removable only if none of the parties in interest properly joined and served as defendants is a citizen of the State in which such action is brought."
[2] " `Under our interpretations, Congress has given the lower courts jurisdiction to hear, originally or by removal from a state court, only those cases in which a well-pleaded complaint establishes either that federal law creates the cause of action or that the plaintiff's right to relief necessarily depends on resolution of a substantial question of federal law.' " 766 F. 2d, at 1006 (quoting Franchise Tax Board, 463 U. S., at 28).
68[3] See Art. III, § 2 ("The judicial Power shall extend to all Cases, in Law and Equity, arising under this Constitution, the Laws of the United States, and Treaties made, or which shall be made, under their Authority . . .").
69[4] Act of Mar. 3, 1875, § 1, 18 Stat. 470. As currently codified, the statute provides: "The district courts shall have original jurisdiction of all civil actions arising under the Constitution, laws or treaties of the United States." 28 U. S. C. § 1331.
70[5] The case most frequently cited for that proposition is Smith v. Kansas City Title & Trust Co.,255 U. S. 180 (1921). In that case the Court upheld federal jurisdiction of a shareholder's bill to enjoin the corporation from purchasing bonds issued by the federal land banks under the authority of the Federal Farm Loan Act on the ground that the federal statute that authorized the issuance of the bonds was unconstitutional. The Court stated:
7172"The general rule is that where it appears from the bill or statement of the plaintiff that the right to relief depends upon the construction or application of the Constitution or laws of the United States, and that such federal claim is not merely colorable, and rests upon a reasonable foundation, the District Court has jurisdiction under this provision." Id., at 199.
The effect of this view, expressed over Justice Holmes' vigorous dissent, on his American Well Works formulation has been often noted. See, e. g., Franchise Tax Board, 463 U. S., at 9 ("[I]t is well settled that Justice Holmes' test is more useful for describing the vast majority of cases that come within the district courts' original jurisdiction than it is for describing which cases are beyond district court jurisdiction"); T. B. Harms Co. v. Eliscu, 339 F. 2d 823, 827 (CA2 1964) (Friendly, J.) ("It has come to be realized that Mr. Justice Holmes' formula is more useful for inclusion than for the exclusion for which it was intended").
73[6] Jurisdiction may not be sustained on a theory that the plaintiff has not advanced. See Healy v. Sea Gull Specialty Co., 237 U. S. 479, 480 (1915) ("[T]he plaintiff is absolute master of what jurisdiction he will appeal to"); The Fair v. Kohler Die & Specialty Co., 228 U. S. 22, 25 (1913) ("[T]he party who brings a suit is master to decide what law he will rely upon"). See also United States v. Mottaz, 476 U. S. 834, 850 (1986).
74[7] See California v. Sierra Club, 451 U. S. 287, 293 (1981); Cannon v. University of Chicago, 441 U. S. 677, 689-709 (1979); Cort v. Ash, 422 U. S. 66, 78 (1975).
75[8] See Merrill Lynch, Pierce, Fenner & Smith, Inc. v. Curran, 456 U. S. 353, 377 (1982) ("In 1975 the Court unanimously decided to modify its approach to the question whether a federal statute includes a private right of action"). Cf. Middlesex County Sewerage Authority v. National Sea Clammers Assn., 453 U. S. 1, 25 (1981) (STEVENS, J., concurring in judgment in part and dissenting in part) ("In 1975, in Cort v. Ash, 422 U. S. 66, the Court cut back on the simple common-law presumption by fashioning a four-factor formula that led to the denial of relief in that case").
76[9] See, e. g., Daily Income Fund, Inc. v. Fox, 464 U. S. 523, 535-536 (1984) ("In evaluating such a claim, our focus must be on the intent of Congress when it enacted the statute in question"); Middlesex County Sewerage Authority v. National Sea Clammers Assn., 453 U. S., at 13 ("The key to the inquiry is the intent of the Legislature"); Texas Industries, Inc. v. Radcliff Materials, Inc., 451 U. S. 630, 639 (1981) ("Our focus, as it is in any case involving the implication of a right of action, is on the intent of Congress"); California v. Sierra Club, 451 U. S., at 293 ("[T]he ultimate issue is whether Congress intended to create a private right of action"); Northwest Airlines, Inc. v. Transport Workers, 451 U. S. 77, 91 (1981) ("The ultimate question in cases such as this is whether Congress intended to create the private remedy"); Transamerica Mortgage Advisors, Inc. v. Lewis, 444 U. S. 11, 15 (1979) ("The question whether a statute creates a cause of action, either expressly or by implication, is basically a matter of statutory construction"); Touche Ross & Co. v. Redington, 442 U. S. 560, 568 (1979) ("The question of the existence of a statutory cause of action is, of course, one of statutory construction").
77[10] When we conclude that Congress has decided not to provide a particular federal remedy, we are not free to "supplement" that decision in a way that makes it "meaningless." Cf. Mobil Oil Corp. v. Higginbotham, 436 U. S. 618, 625 (1978) (When Congress "does speak directly to a question, the courts are not free to `supplement' Congress' answer so thoroughly that the Act becomes meaningless"). See also California v. Sierra Club, 451 U. S., at 297 ("The federal judiciary will not engraft a remedy on a statute, no matter how salutary, that Congress did not intend to provide").
78[11] See, e. g., Textile Workers v. Lincoln Mills, 353 U. S. 448, 470 (1957) (Frankfurter, J., dissenting) (defining inquiry as "the degree to which federal law must be in the forefront of the case and not collateral, peripheral or remote"); Gully v. First National Bank, 299 U. S. 109, 115 (1936) ("Not every question of federal law emerging in a suit is proof that a federal law is the basis of the suit"); id., at 118 ("If we follow the ascent far enough, countless claims of right can be discovered to have their source or their operative limits in the provisions of a federal statute or in the Constitution itself with its circumambient restrictions upon legislative power. To set bounds to the pursuit, the courts have formulated the distinction between controversies that are basic and those that are collateral, between disputes that are necessary and those that are merely possible. We shall be lost in a maze if we put that compass by").
79[12] Several commentators have suggested that our § 1331 decisions can best be understood as an evaluation of the nature of the federal interest at stake. See, e. g., Shapiro, Jurisdiction and Discretion, 60 N. Y. U. L. Rev. 543, 568 (1985); C. Wright, Federal Courts 96 (4th ed. 1983); Cohen, The Broken Compass: The Requirement That a Case Arise "Directly" Under Federal Law, 115 U. Pa. L. Rev. 890, 916 (1967). Cf. Kravitz v. Homeowners Warranty Corp.,542 F. Supp. 317, 320 (ED Pa. 1982) (Pollak, J.) ("I cannot identify any compelling reasons of federal judicial policy for embracing a case of this kind as a federal question case. The essential Pennsylvania elements of plaintiffs' suit for rescission would be more appropriately dealt with by a Court of Common Pleas than by this court; and, with respect to the lesser-included issue of federal law, Pennsylvania's courts are fully competent to interpret the Magnuson-Moss Warranty Act and the relevant F. T. C. regulations, subject to review by the United States Supreme Court").
80Focusing on the nature of the federal interest, moreover, suggests that the widely perceived "irreconcilable" conflict between the finding of federal jurisdiction in Smith v. Kansas City Title & Trust Co., 255 U. S. 180 (1921), and the finding of no jurisdiction in Moore v. Chesapeake & Ohio R. Co., 291 U. S. 205 (1934), see, e. g., M. Redish, Federal Jurisdiction: Tensions in the Allocation of Judicial Power 67 (1980), is far from clear. For the difference in results can be seen as manifestations of the differences in the nature of the federal issues at stake. In Smith, as the Court emphasized, the issue was the constitutionality of an important federal statute. See 255 U. S., at 201 ("It is . . . apparent that the controversy concerns the constitutional validity of an act of Congress which is directly drawn in question. The decision depends upon the determination of this issue"). In Moore, in contrast, the Court emphasized that the violation of the federal standard as an element of state tort recovery did not fundamentally change the state tort nature of the action. See 291 U. S., at 216-217 (" `The action fell within the familiar category of cases involving the duty of a master to his servant. This duty is defined by the common law, except as it may be modified by legislation. The federal statute, in the present case, touched the duty of the master at a single point and, save as provided in the statute, the right of the plaintiff to recover was left to be determined by the law of the State' ") (quoting Minneapolis, St. P. & S. S. M. R. Co. v. Popplar, 237 U. S. 369, 372 (1915)).
81The importance of the nature of the federal issue in federal-question jurisdiction is highlighted by the fact that, despite the usual reliability of the Holmes test as an inclusionary principle, this Court has sometimes found that formally federal causes of action were not properly brought under federal-question jurisdiction because of the overwhelming predominance of state-law issues. See Shulthis v. McDougal, 225 U. S. 561, 569-570 (1912) ("A suit to enforce a right which takes its origin in the laws of the United States is not necessarily, or for that reason alone, one arising under those laws, for a suit does not so arise unless it really and substantially involves a dispute or controversy respecting the validity, construction or effect of such a law, upon the determination of which the result depends. This is especially so of a suit involving rights to land acquired under a law of the United States. If it were not, every suit to establish title to land in the central and western States would so arise, as all titles in those States are traceable back to those laws"); Shoshone Mining Co. v. Rutter, 177 U. S. 505, 507 (1900) ("We pointed out in the former opinion that it was well settled that a suit to enforce a right which takes its origin in the laws of the United States is not necessarily one arising under the Constitution or laws of the United States, within the meaning of the jurisdiction clauses, for if it did every action to establish title to real estate (at least in the newer States) would be such a one, as all titles in those States come from the United States or by virtue of its laws").
82[13] Cf. Longshoremen v. Davis, 476 U. S. 380, 391 (1986) ("[O]ur decisions describing the nature of Garmon pre-emption and defining its boundaries have rested on a determination that in enacting the [National Labor Relations Act] Congress intended for the [National Labor Relations] Board generally to exercise exclusive jurisdiction in this area").
83[14] See Moore v. Chesapeake & Ohio R. Co., 291 U. S., at 214-215 ("Questions arising in actions in state courts to recover for injuries sustained by employees in intrastate commerce and relating to the scope or construction of the Federal Safety Appliance Acts are, of course, federal questions which may appropriately be reviewed in this Court. . . . But it does not follow that a suit brought under the state statute which defines liability to employees who are injured while engaged in intrastate commerce, and brings within the purview of the statute a breach of the duty imposed by the federal statute, should be regarded as a suit arising under the laws of the United States and cognizable in the federal court in the absence of diversity of citizenship"). Cf. Franchise Tax Board, 463 U. S., at 12, n. 12 ("[T]he absence of original jurisdiction does not mean that there is no federal forum in which a pre-emption defense may be heard. If the state courts reject a claim of federal pre-emption, that decision may ultimately be reviewed on appeal by this Court").
84[15] Petitioner also contends that the Court of Appeals opinion rests on a view that federal-question jurisdiction was inappropriate because, whatever the role of the federal issue in the FDCA-related count, the plaintiff could recover on other, strictly state-law claims. See 766 F. 2d, at 1006 (noting that "the jury could find negligence on the part of Merrell Dow without finding a violation of the FDCA"). To the extent that the opinion can be read to express such a view, we agree that it was erroneous. If the FDCA-related count presented a sufficient federal question, its relationship to the other, state-law claims would be determined by the ordinary principles of pendent jurisdiction described in Mine Workers v. Gibbs, 383 U. S. 715 (1966). For the reasons that we have stated, however, there is no federal-question jurisdiction even with that possible error corrected.
85[16] Some commentators have argued that the result in Smith conflicts with our decision in Moore v. Chesapeake & Ohio R. Co., 291 U. S. 205 (1934). See, e. g., Greene, Hybrid State Law in the Federal Courts, 83 Harv. L. Rev. 289, 323 (1969). In Moore, the plaintiff brought an action under Kentucky's Employer Liability Act, which provided that a plaintiff could not be held responsible for contributory negligence or assumption of risk where his injury resulted from the violation of any state or federal statute enacted for the safety of employees. The plaintiff in Moorealleged that his injury was due to the defendant's failure to comply with the Federal Safety Appliance Act; therefore, an important issue in the adjudication of the state cause of action was whether the terms of the federal law had been violated. The Court could have dismissed the complaint on the ground that the federal issue would arise only in response to a defense of contributory negligence or assumption of risk, and that therefore there was no jurisdiction under the well-pleaded complaint rule. Instead, the Court held that "a suit brought under the state statute which defines liability to employees who are injured while engaged in intrastate commerce, and brings within the purview of the statute a breach of the duty imposed by the federal statute, should [not] be regarded as a suit arising under the laws of the United States and cognizable in the federal court in the absence of diversity of citizenship." 291 U. S., at 214-215.
86The Court suggests that Smith and Moore may be reconciled if one views the question whether there is jurisdiction under § 1331 as turning upon "an evaluation of the nature of the federal interest at stake." Ante, at 814, n. 12 (emphasis in original). Thus, the Court explains, while in Smith the issue was the constitutionality of "an important federal statute," in Moore the federal interest was less significant in that "the violation of the federal standard as an element of state tort recovery did not fundamentally change the state tort nature of the action." Ante, at 815, n. 12.
87In one sense, the Court is correct in asserting that we can reconcile Smith and Moore on the ground that the "nature" of the federal interest was more significant in Smith than in Moore. Indeed, as the Court appears to believe, ante, at 814-815, n. 12, we could reconcile many of the seemingly inconsistent results that have been reached under § 1331 with such a test. But this is so only because a test based upon an ad hoc evaluation of the importance of the federal issue is infinitely malleable: at what point does a federal interest become strong enough to create jurisdiction? What principles guide the determination whether a statute is "important" or not? Why, for instance, was the statute in Smith so "important" that direct review of a state-court decision (under our mandatory appellate jurisdiction) would have been inadequate? Would the result in Moore have been different if the federal issue had been a more important element of the tort claim? The point is that if one makes the test sufficiently vague and general, virtually any set of results can be "reconciled." However, the inevitable — and undesirable — result of a test such as that suggested in the Court's footnote 12 is that federal jurisdiction turns in every case on an appraisal of the federal issue, its importance and its relation to state-law issues. Yet it is precisely because the Court believes that federal jurisdiction would be "ill served" by such a case-by-case appraisal that it rejects petitioner's claim that the difficulty and importance of the statutory issue presented by its claim suffices to confer jurisdiction under § 1331. Ante, at 817. The Court cannot have it both ways.
88My own view is in accord with those commentators who view the results in Smith and Moore as irreconcilable. See, e. g., Redish 67; D. Currie, Federal Jurisdiction in a Nutshell 109 (2d ed. 1981). That fact does not trouble me greatly, however, for I view Moore as having been a "sport" at the time it was decided and having long been in a state of innocuous desuetude. Unlike the jurisdictional holding in Smith, the jurisdictional holding in Moore has never been relied upon or even cited by this Court. Moore has similarly borne little fruit in the lower courts, leading Professor Redish to conclude after comparing the vitality of Smith and Moore that "the principle enunciated in Smith is the one widely followed by modern lower federal courts." Redish 67. Finally, as noted in text, the commentators have also preferred Smith. Supra, at 821. Moore simply has not survived the test of time; it is presently moribund, and, to the extent that it is inconsistent with the well-established rule of the Smith case, it ought to be overruled.
89[17] As the Court correctly notes, the Court of Appeals erred in holding that respondents' right to relief did not depend upon the resolution of a federal question because respondents might prevail on one of their other, wholly state-law claims. The fourth cause of action presents an independent and independently sufficient claim for relief. Whether it "arises under" federal law within the meaning of § 1331 must therefore be determined without reference to any other claims, as if only that claim was asserted. If, after such consideration, it is determined that there is jurisdiction, the plaintiff may join additional state-law claims meeting the test for pendent jurisdiction set forth in Mine Workers v. Gibbs, 383 U. S. 715 (1966). See ante, at 817, n. 15.
90[18] Franchise Tax Board states that the plaintiff's right to relief must necessarily depend upon resolution of a "substantial" federal question. 463 U. S., at 28. In context, however, it is clear that this was simply another way of stating that the federal question must be colorable and have a reasonable foundation. This understanding is consistent with the manner in which the Smith test has always been applied, as well as with the way we have used the concept of a "substantial" federal question in other cases concerning federal jurisdiction. See, e. g., Hagans v. Lavine, 415 U. S. 528, 536-537 (1974); Bell v. Hood, 327 U. S. 678, 682 (1946).
91[19] It bears emphasizing that the Court does not hold that there is no private cause of action under the FDCA. Rather, it expressly states that "[f]or purposes of our decision, we assume that this is a correct interpretation of the FDCA." Ante, at 810. The Court simply holds petitioner to its concession that the FDCA provides no private remedy, and decides petitioner's claim on the basis of this concession. I shall do the same. Under the Court's analysis, however, if a party persuaded a court that there is a private cause of action under the FDCA, there would be federal jurisdiction under Smith and Franchise Tax Board over a state cause of action making violations of the FDCA actionable. Such jurisdiction would apparently exist even if the plaintiff did not seek the federal remedy.
92[20] Congress granted original federal-question jurisdiction briefly in the Midnight Judges Act, ch. 4, § 11, 2 Stat. 92 (1801), which was repealed in 1802, Act of Mar. 8, 1802, ch. 8, § 1, 2 Stat. 132.
93[21] Another reason Congress conferred original federal-question jurisdiction on the district courts was its belief that state courts are hostile to assertions of federal rights. See Hornstein, Federalism, Judicial Power and the "Arising Under" Jurisdiction of the Federal Courts: A Hierarchical Analysis, 56 Ind. L. J. 563, 564-565 (1981); Comment 636; Redish 71. Although this concern may be less compelling today than it once was, the American Law Institute reported as recently as 1969 that "it is difficult to avoid concluding that federal courts are more likely to apply federal law sympathetically and understandingly than are state courts." ALI 166. In any event, this rationale is, like the rationale based on the expertise of the federal courts, simply an expression of Congress' belief that federal courts are more likely to interpret federal law correctly.
94One might argue that this Court's appellate jurisdiction over state-court judgments in cases arising under federal law can be depended upon to correct erroneous state-court decisions and to insure that federal law is interpreted and applied uniformly. However, as any experienced observer of this Court can attest, "Supreme Court review of state courts, limited by docket pressures, narrow review of the facts, the debilitating possibilities of delay, and the necessity of deferring to adequate state grounds of decision, cannot do the whole job." Currie 160. Indeed, having served on this Court for 30 years, it is clear to me that, realistically, it cannot even come close to "doing the whole job" and that § 1331 is essential if federal rights are to be adequately protected.
95[22] Cf. Cohens v. Virginia, 6 Wheat. 264, 404 (1821) (Marshall, C. J.) ("It is most true that this Court will not take jurisdiction if it should not; but it is equally true, that it must take jurisdiction if it should. . . . We have no more right to decline the exercise of jurisdiction which is given, than to usurp that which is not given"). The narrow exceptions we have recognized to Chief Justice Marshall's famous dictum have all been justified by compelling judicial concerns of comity and federalism. See, e. g., Younger v. Harris, 401 U. S. 37 (1971); Burford v. Sun Oil Co., 319 U. S. 315 (1943). It would be wholly illegitimate, however, for this Court to determine that there was no jurisdiction over a class of cases simply because the Court thought that there were too many cases in the federal courts.
96[23] The Federal Trade Commission retains regulatory and enforcement authority over the advertising (as opposed to the labeling) of foods, drugs, and cosmetics. See 15 U. S. C. §§ 52-55.
Supreme Court of United States.
[1062] Jane Webre, Austin, Texas, for Petitioners.
8Thomas M. Michel, Fort Worth, Texas, for Respondent.
9Jane M.N. Webre, Cynthia S. Connolly, Scott, Douglass & McConnico, L.L.P., Austin, Texas, Robert S. Harrell, Charles B. Walker, Jr., Fulbright & Jaworski, L.L.P., Houston, Texas, David E. Keltner, Kelly Hart & Hallman, L.L.P., Fort Worth, Texas, for Petitioners.
10Thomas M. Michel, Robley E. Sicard, Griffith, Jay & Michel, LLP, Fort Worth, Texas, Coyt Randal Johnston, Robert L. Tobey, Coyt Randal Johnston, Jr., Johnston Tobey, P.C., Dallas, Texas, Theodore F. Shiells, Shiells Law Firm P.C., Dallas, Texas, Gregory W. Carr, Carr LLP, Frisco, Texas, Daniel R. Ortiz, Charlottesville, Virginia, for Respondent.
11Federal courts have exclusive jurisdiction over cases "arising under any Act of Congress relating to patents." 28 U.S.C. § 1338(a). The question presented is whether a state law claim alleging legal malpractice in the handling of a patent case must be brought in federal court.
13In the early 1990s, respondent Vernon Minton developed a computer program and telecommunications network designed to facilitate securities trading. In March 1995, he leased the system — known as the Texas Computer Exchange Network, or TEXCEN — to R.M. Stark & Co., a securities brokerage. A little over a year later, he applied for a patent for an interactive securities trading system that was based substantially on TEXCEN. The U.S. Patent and Trademark Office issued the patent in January 2000.
15Patent in hand, Minton filed a patent infringement suit in Federal District Court against the National Association of Securities Dealers, Inc. (NASD) and the NASDAQ Stock Market, Inc. He was represented by Jerry Gunn and the other petitioners. NASD and NASDAQ moved for summary judgment on the ground that Minton's patent was invalid under the "on sale" bar, 35 U.S.C. § 102(b). That provision specifies that an inventor is not entitled to a patent if "the invention was ... on sale in [the United States], more than one year prior to the date of the application," and Minton had leased TEXCEN to Stark more than one year prior to filing his patent application. Rejecting Minton's argument that there were differences between TEXCEN and the patented system that precluded application of the on-sale bar, the District Court granted the summary judgment motion and declared Minton's patent invalid. Minton v. National Assn. of Securities Dealers, Inc., 226 F.Supp.2d 845, 873, 883-884 (E.D.Tex. 2002).
16Minton then filed a motion for reconsideration in the District Court, arguing for the first time that the lease agreement with Stark was part of ongoing testing of TEXCEN and therefore fell within the [1063] "experimental use" exception to the on-sale bar. See generally Pfaff v. Wells Electronics, Inc., 525 U.S. 55, 64, 119 S.Ct. 304, 142 L.Ed.2d 261 (1998) (describing the exception). The District Court denied the motion. Minton v. National Assn. of Securities Dealers, Inc., No. 9:00-cv-00019 (ED Tex., July 15, 2002).
17Minton appealed to the U.S. Court of Appeals for the Federal Circuit. That court affirmed, concluding that the District Court had appropriately held Minton's experimental-use argument waived. See Minton v. National Assn. of Securities Dealers, Inc., 336 F.3d 1373, 1379-1380 (C.A.Fed.2003).
18Minton, convinced that his attorneys' failure to raise the experimental-use argument earlier had cost him the lawsuit and led to invalidation of his patent, brought this malpractice action in Texas state court. His former lawyers defended on the ground that the lease to Stark was not, in fact, for an experimental use, and that therefore Minton's patent infringement claims would have failed even if the experimental-use argument had been timely raised. The trial court agreed, holding that Minton had put forward "less than a scintilla of proof" that the lease had been for an experimental purpose. App. 213. It accordingly granted summary judgment to Gunn and the other lawyer defendants.
19On appeal, Minton raised a new argument: Because his legal malpractice claim was based on an alleged error in a patent case, it "aris[es] under" federal patent law for purposes of 28 U.S.C. § 1338(a). And because, under § 1338(a), "[n]o State court shall have jurisdiction over any claim for relief arising under any Act of Congress relating to patents," the Texas court — where Minton had originally brought his malpractice claim — lacked subject matter jurisdiction to decide the case. Accordingly, Minton argued, the trial court's order should be vacated and the case dismissed, leaving Minton free to start over in the Federal District Court.
20A divided panel of the Court of Appeals of Texas rejected Minton's argument. Applying the test we articulated in Grable & Sons Metal Products, Inc. v. Darue Engineering & Mfg., 545 U.S. 308, 314, 125 S.Ct. 2363, 162 L.Ed.2d 257 (2005), it held that the federal interests implicated by Minton's state law claim were not sufficiently substantial to trigger § 1338 "arising under" jurisdiction. It also held that finding exclusive federal jurisdiction over state legal malpractice actions would, contrary to Grable's commands, disturb the balance of federal and state judicial responsibilities. Proceeding to the merits of Minton's malpractice claim, the Court of Appeals affirmed the trial court's determination that Minton had failed to establish experimental use and that arguments on that ground therefore would not have saved his infringement suit.
21The Supreme Court of Texas reversed, relying heavily on a pair of cases from the U.S. Court of Appeals for the Federal Circuit. 355 S.W.3d 634, 641-642 (2011) (discussing Air Measurement Technologies, Inc. v. Akin Gump Strauss Hauer & Feld, L.L. P., 504 F.3d 1262 (2007); Immunocept, LLC v. Fulbright & Jaworski, LLP, 504 F.3d 1281 (2007)). The Court concluded that Minton's claim involved "a substantial federal issue" within the meaning of Grable "because the success of Minton's malpractice claim is reliant upon the viability of the experimental use exception as a defense to the on-sale bar." 355 S.W.3d, at 644. Adjudication of Minton's claim in federal court was consistent with the appropriate balance between federal and state judicial responsibilities, it held, because "the federal government and patent litigants have an interest in the uniform application of patent law by courts [1064] well-versed in that subject matter." Id., at 646 (citing Immunocept, supra, at 1285-1286; Air Measurement Technologies, supra, at 1272).
22Justice Guzman, joined by Justices Medina and Willett, dissented. The dissenting justices would have held that the federal issue was neither substantial nor disputed, and that maintaining the proper balance of responsibility between state and federal courts precluded relegating state legal malpractice claims to federal court.
23We granted certiorari. 568 U.S. ___, 133 S.Ct. 420, 184 L.Ed.2d 251 (2012).
24"Federal courts are courts of limited jurisdiction," possessing "only that power authorized by Constitution and statute." Kokkonen v. Guardian Life Ins. Co. of America, 511 U.S. 375, 377, 114 S.Ct. 1673, 128 L.Ed.2d 391 (1994). There is no dispute that the Constitution permits Congress to extend federal court jurisdiction to a case such as this one, see Osborn v. Bank of United States, 9 Wheat. 738, 823-824, 6 L.Ed. 204 (1824); the question is whether Congress has done so, see Powell v. McCormack, 395 U.S. 486, 515-516, 89 S.Ct. 1944, 23 L.Ed.2d 491 (1969).
26As relevant here, Congress has authorized the federal district courts to exercise original jurisdiction in "all civil actions arising under the Constitution, laws, or treaties of the United States," 28 U.S.C. § 1331, and, more particularly, over "any civil action arising under any Act of Congress relating to patents," § 1338(a). Adhering to the demands of "[l]inguistic consistency," we have interpreted the phrase "arising under" in both sections identically, applying our § 1331 and § 1338(a) precedents interchangeably. See Christianson v. Colt Industries Operating Corp., 486 U.S. 800, 808-809, 108 S.Ct. 2166, 100 L.Ed.2d 811 (1988). For cases falling within the patent-specific arising under jurisdiction of § 1338(a), however, Congress has not only provided for federal jurisdiction but also eliminated state jurisdiction, decreeing that "[n]o State court shall have jurisdiction over any claim for relief arising under any Act of Congress relating to patents." § 1338(a) (2006 ed., Supp. V). To determine whether jurisdiction was proper in the Texas courts, therefore, we must determine whether it would have been proper in a federal district court — whether, that is, the case "aris[es] under any Act of Congress relating to patents."
27For statutory purposes, a case can "aris[e] under" federal law in two ways. Most directly, a case arises under federal law when federal law creates the cause of action asserted. See American Well Works Co. v. Layne & Bowler Co., 241 U.S. 257, 260, 36 S.Ct. 585, 60 L.Ed. 987 (1916) ("A suit arises under the law that creates the cause of action"). As a rule of inclusion, this "creation" test admits of only extremely rare exceptions, see, e.g., Shoshone Mining Co. v. Rutter, 177 U.S. 505, 20 S.Ct. 726, 44 L.Ed. 864 (1900), and accounts for the vast bulk of suits that arise under federal law, see Franchise Tax Bd. of Cal. v. Construction Laborers Vacation Trust for Southern Cal., 463 U.S. 1, 9, 103 S.Ct. 2841, 77 L.Ed.2d 420 (1983). Minton's original patent infringement suit against NASD and NASDAQ, for example, arose under federal law in this manner because it was authorized by 35 U.S.C. §§ 271, 281.
28But even where a claim finds its origins in state rather than federal law — as Minton's legal malpractice claim indisputably does — we have identified a "special and small category" of cases in which arising under jurisdiction still lies. Empire Healthchoice Assurance, Inc. v. McVeigh, [1065] 547 U.S. 677, 699, 126 S.Ct. 2121, 165 L.Ed.2d 131 (2006). In outlining the contours of this slim category, we do not paint on a blank canvas. Unfortunately, the canvas looks like one that Jackson Pollock got to first. See 13D C. Wright, A. Miller, E. Cooper, & R. Freer, Federal Practice and Procedure § 3562, pp. 175-176 (3d ed. 2008) (reviewing general confusion on question).
29In an effort to bring some order to this unruly doctrine several Terms ago, we condensed our prior cases into the following inquiry: Does the "state-law claim necessarily raise a stated federal issue, actually disputed and substantial, which a federal forum may entertain without disturbing any congressionally approved balance of federal and state judicial responsibilities"? Grable, 545 U.S., at 314, 125 S.Ct. 2363. That is, federal jurisdiction over a state law claim will lie if a federal issue is: (1) necessarily raised, (2) actually disputed, (3) substantial, and (4) capable of resolution in federal court without disrupting the federal-state balance approved by Congress. Where all four of these requirements are met, we held, jurisdiction is proper because there is a "serious federal interest in claiming the advantages thought to be inherent in a federal forum," which can be vindicated without disrupting Congress's intended division of labor between state and federal courts. Id., at 313-314, 125 S.Ct. 2363.
30Applying Grable's inquiry here, it is clear that Minton's legal malpractice claim does not arise under federal patent law. Indeed, for the reasons we discuss, we are comfortable concluding that state legal malpractice claims based on underlying patent matters will rarely, if ever, arise under federal patent law for purposes of § 1338(a). Although such cases may necessarily raise disputed questions of patent law, those cases are by their nature unlikely to have the sort of significance for the federal system necessary to establish jurisdiction.
32To begin, we acknowledge that resolution of a federal patent question is "necessary" to Minton's case. Under Texas law, a plaintiff alleging legal malpractice must establish four elements: (1) that the defendant attorney owed the plaintiff a duty; (2) that the attorney breached that duty; (3) that the breach was the proximate cause of the plaintiff's injury; and (4) that damages occurred. See Alexander v. Turtur & Associates, Inc., 146 S.W.3d 113, 117 (Tex.2004). In cases like this one, in which the attorney's alleged error came in failing to make a particular argument, the causation element requires a "case within a case" analysis of whether, had the argument been made, the outcome of the earlier litigation would have been different. 355 S.W.3d, at 639; see 4 R. Mallen & J. Smith, Legal Malpractice § 37:15, pp. 1509-1520 (2012). To prevail on his legal malpractice claim, therefore, Minton must show that he would have prevailed in his federal patent infringement case if only petitioners had timely made an experimental-use argument on his behalf. 355 S.W.3d, at 644. That will necessarily require application of patent law to the facts of Minton's case.
34The federal issue is also "actually disputed" here — indeed, on the merits, it is the central point of dispute. Minton argues that the experimental-use exception properly applied to his lease to Stark, saving his patent from the on-sale bar; petitioners argue that it did not. This is just the sort of "`dispute ... respecting the [1066] ... effect of [federal] law'" that Grable envisioned. 545 U.S., at 313, 125 S.Ct. 2363 (quoting Shulthis v. McDougal, 225 U.S. 561, 569, 32 S.Ct. 704, 56 L.Ed. 1205 (1912)).
36Minton's argument founders on Grable's next requirement, however, for the federal issue in this case is not substantial in the relevant sense. In reaching the opposite conclusion, the Supreme Court of Texas focused on the importance of the issue to the plaintiff's case and to the parties before it. 355 S.W.3d, at 644 ("because the success of Minton's malpractice claim is reliant upon the viability of the experimental use exception as a defense to the on-sale bar, we hold that it is a substantial federal issue"); see also Air Measurement Technologies, 504 F.3d, at 1272 ("the issue is substantial, for it is a necessary element of the malpractice case"). As our past cases show, however, it is not enough that the federal issue be significant to the particular parties in the immediate suit; that will always be true when the state claim "necessarily raise[s]" a disputed federal issue, as Grable separately requires. The substantiality inquiry under Grable looks instead to the importance of the issue to the federal system as a whole.
38In Grable itself, for example, the Internal Revenue Service had seized property from the plaintiff and sold it to satisfy the plaintiff's federal tax delinquency. 545 U.S., at 310-311, 125 S.Ct. 2363. Five years later, the plaintiff filed a state law quiet title action against the third party that had purchased the property, alleging that the IRS had failed to comply with certain federally imposed notice requirements, so that the seizure and sale were invalid. Ibid. In holding that the case arose under federal law, we primarily focused not on the interests of the litigants themselves, but rather on the broader significance of the notice question for the Federal Government. We emphasized the Government's "strong interest" in being able to recover delinquent taxes through seizure and sale of property, which in turn "require[d] clear terms of notice to allow buyers ... to satisfy themselves that the Service has touched the bases necessary for good title." Id., at 315, 125 S.Ct. 2363. The Government's "direct interest in the availability of a federal forum to vindicate its own administrative action" made the question "an important issue of federal law that sensibly belong[ed] in a federal court." Ibid.
39A second illustration of the sort of substantiality we require comes from Smith v. Kansas City Title & Trust Co., 255 U.S. 180, 41 S.Ct. 243, 65 L.Ed. 577 (1921), which Grable described as "[t]he classic example" of a state claim arising under federal law. 545 U.S., at 312, 125 S.Ct. 2363. In Smith, the plaintiff argued that the defendant bank could not purchase certain bonds issued by the Federal Government because the Government had acted unconstitutionally in issuing them. 255 U.S., at 198, 41 S.Ct. 243. We held that the case arose under federal law, because the "decision depends upon the determination" of "the constitutional validity of an act of Congress which is directly drawn in question." Id., at 201, 41 S.Ct. 243. Again, the relevant point was not the importance of the question to the parties alone but rather the importance more generally of a determination that the Government "securities were issued under an unconstitutional law, and hence of no validity." Ibid.; see also Merrell Dow Pharmaceuticals Inc. v. Thompson, 478 U.S. 804, 814, n. 12, 106 S.Ct. 3229, 92 L.Ed.2d 650 (1986).
40Here, the federal issue carries no such significance. Because of the backward-looking [1067] nature of a legal malpractice claim, the question is posed in a merely hypothetical sense: If Minton's lawyers had raised a timely experimental-use argument, would the result in the patent infringement proceeding have been different? No matter how the state courts resolve that hypothetical "case within a case," it will not change the real-world result of the prior federal patent litigation. Minton's patent will remain invalid.
41Nor will allowing state courts to resolve these cases undermine "the development of a uniform body of [patent] law." Bonito Boats, Inc. v. Thunder Craft Boats, Inc., 489 U.S. 141, 162, 109 S.Ct. 971, 103 L.Ed.2d 118 (1989). Congress ensured such uniformity by vesting exclusive jurisdiction over actual patent cases in the federal district courts and exclusive appellate jurisdiction in the Federal Circuit. See 28 U.S.C. §§ 1338(a), 1295(a)(1). In resolving the nonhypothetical patent questions those cases present, the federal courts are of course not bound by state court case-within-a-case patent rulings. See Tafflin v. Levitt, 493 U.S. 455, 465, 110 S.Ct. 792, 107 L.Ed.2d 887 (1990). In any event, the state court case-within-a-case inquiry asks what would have happened in the prior federal proceeding if a particular argument had been made. In answering that question, state courts can be expected to hew closely to the pertinent federal precedents. It is those precedents, after all, that would have applied had the argument been made. Cf. ibid. ("State courts adjudicating civil RICO claims will ... be guided by federal court interpretations of the relevant federal criminal statutes, just as federal courts sitting in diversity are guided by state court interpretations of state law").
42As for more novel questions of patent law that may arise for the first time in a state court "case within a case," they will at some point be decided by a federal court in the context of an actual patent case, with review in the Federal Circuit. If the question arises frequently, it will soon be resolved within the federal system, laying to rest any contrary state court precedent; if it does not arise frequently, it is unlikely to implicate substantial federal interests. The present case is "poles apart from Grable," in which a state court's resolution of the federal question "would be controlling in numerous other cases." Empire Healthchoice Assurance, Inc., 547 U.S., at 700, 126 S.Ct. 2121.
43Minton also suggests that state courts' answers to hypothetical patent questions can sometimes have real-world effect on other patents through issue preclusion. Brief for Respondent 33-36. Minton, for example, has filed what is known as a "continuation patent" application related to his original patent. See 35 U.S.C. § 120; 4A D. Chisum, Patents § 13.03 (2005) (describing continuation applications). He argues that, in evaluating this separate application, the patent examiner could be bound by the Texas trial court's interpretation of the scope of Minton's original patent. See Brief for Respondent 35-36. It is unclear whether this is true. The Patent and Trademark Office's Manual of Patent Examining Procedure provides that res judicata is a proper ground for rejecting a patent "only when the earlier decision was a decision of the Board of Appeals" or certain federal reviewing courts, giving no indication that state court decisions would have preclusive effect. See Dept. of Commerce, Patent and Trademark Office, Manual of Patent Examining Procedure § 706.03(w), p. 700-79 (rev. 8th ed. 2012); 35 U.S.C. §§ 134(a), 141, 145; Reply Brief 9-10. In fact, Minton has not identified any case finding such preclusive effect based on a state court decision. But even assuming that a state court's case-within-a-case adjudication may be preclusive under [1068] some circumstances, the result would be limited to the parties and patents that had been before the state court. Such "fact-bound and situation-specific" effects are not sufficient to establish federal arising under jurisdiction. Empire Healthchoice Assurance, Inc., supra, at 701, 126 S.Ct. 2121.
44Nor can we accept the suggestion that the federal courts' greater familiarity with patent law means that legal malpractice cases like this one belong in federal court. See Air Measurement Technologies, 504 F.3d, at 1272 ("The litigants will also benefit from federal judges who have experience in claim construction and infringement matters"); 355 S.W.3d, at 646 ("patent litigants have an interest in the uniform application of patent law by courts well-versed in that subject matter"). It is true that a similar interest was among those we considered in Grable, 545 U.S., at 314, 125 S.Ct. 2363. But the possibility that a state court will incorrectly resolve a state claim is not, by itself, enough to trigger the federal courts' exclusive patent jurisdiction, even if the potential error finds its root in a misunderstanding of patent law.
45There is no doubt that resolution of a patent issue in the context of a state legal malpractice action can be vitally important to the particular parties in that case. But something more, demonstrating that the question is significant to the federal system as a whole, is needed. That is missing here.
46It follows from the foregoing that Grable's fourth requirement is also not met. That requirement is concerned with the appropriate "balance of federal and state judicial responsibilities." Ibid. We have already explained the absence of a substantial federal issue within the meaning of Grable. The States, on the other hand, have "a special responsibility for maintaining standards among members of the licensed professions." Ohralik v. Ohio State Bar Assn., 436 U.S. 447, 460, 98 S.Ct. 1912, 56 L.Ed.2d 444 (1978). Their "interest ... in regulating lawyers is especially great since lawyers are essential to the primary governmental function of administering justice, and have historically been officers of the courts." Goldfarb v. Virginia State Bar, 421 U.S. 773, 792, 95 S.Ct. 2004, 44 L.Ed.2d 572 (1975) (internal quotation marks omitted). We have no reason to suppose that Congress — in establishing exclusive federal jurisdiction over patent cases — meant to bar from state courts state legal malpractice claims simply because they require resolution of a hypothetical patent issue.
48As we recognized a century ago, "[t]he Federal courts have exclusive jurisdiction of all cases arising under the patent laws, but not of all questions in which a patent may be the subject-matter of the controversy." New Marshall Engine Co. v. Marshall Engine Co., 223 U.S. 473, 478, 32 S.Ct. 238, 56 L.Ed. 513 (1912). In this case, although the state courts must answer a question of patent law to resolve Minton's legal malpractice claim, their answer will have no broader effects. It will not stand as binding precedent for any future patent claim; it will not even affect the validity of Minton's patent. Accordingly, there is no "serious federal interest in claiming the advantages thought to be inherent in a federal forum," Grable, supra, at 313, 125 S.Ct. 2363. Section 1338(a) does not deprive the state courts of subject matter jurisdiction.
50The judgment of the Supreme Court of Texas is reversed, and the case is remanded [1069] for further proceedings not inconsistent with this opinion.
51It is so ordered.
This is summarized in the cheat sheet and in Gun v. Minton, so reading it is OPTIONAL if you want more.
Supreme Court of United States.
CERTIORARI TO THE UNITED STATES COURT OF APPEALS FOR THE SIXTH CIRCUIT
8[309] SOUTER, J., delivered the opinion for a unanimous Court. THOMAS, J., filed a concurring opinion, post, p. 320.
9Eric H. Zagrans argued the cause for petitioner. On the briefs was Charles E. McFarland.
10[310] Michael C. Walton argued the cause for respondent. With him on the brief were John M. Lichtenberg, Gregory G. Timmer, and Mary L. Tabin.
11Irving L. Gornstein argued the cause for the United States as amicus curiae urging affirmance. With him on the brief were Acting Solicitor General Clement, Assistant Attorney General O'Connor, Deputy Solicitor General Hungar, and Gilbert S. Rothenberg.[1]
12The question is whether want of a federal cause of action to try claims of title to land obtained at a federal tax sale precludes removal to federal court of a state action with nondiverse parties raising a disputed issue of federal title law. We answer no, and hold that the national interest in providing a federal forum for federal tax litigation is sufficiently substantial to support the exercise of federal-question jurisdiction over the disputed issue on removal, which would not distort any division of labor between the state and federal courts, provided or assumed by Congress.
14In 1994, the Internal Revenue Service seized Michigan real property belonging to petitioner Grable & Sons Metal Products, Inc., to satisfy Grable's federal tax delinquency. Title 26 U. S. C. § 6335 required the IRS to give notice of the seizure, and there is no dispute that Grable received actual notice by certified mail before the IRS sold the property to respondent Darue Engineering & Manufacturing. Although Grable also received notice of the sale itself, it did not exercise its statutory right to redeem the property within 180 days of the sale, § 6337(b)(1), and after that period [311] had passed, the Government gave Darue a quitclaim deed, § 6339.
16Five years later, Grable brought a quiet title action in state court, claiming that Darue's record title was invalid because the IRS had failed to notify Grable of its seizure of the property in the exact manner required by § 6335(a), which provides that written notice must be "given by the Secretary to the owner of the property [or] left at his usual place of abode or business." Grable said that the statute required personal service, not service by certified mail.
17Darue removed the case to Federal District Court as presenting a federal question, because the claim of title depended on the interpretation of the notice statute in the federal tax law. The District Court declined to remand the case at Grable's behest after finding that the "claim does pose a `significant question of federal law," Tr. 17 (Apr. 2, 2001), and ruling that Grable's lack of a federal right of action to enforce its claim against Darue did not bar the exercise of federal jurisdiction. On the merits, the court granted summary judgment to Darue, holding that although § 6335 by its terms required personal service, substantial compliance with the statute was enough. 207 F. Supp. 2d 694 (WD Mich. 2002).
18The Court of Appeals for the Sixth Circuit affirmed. 377 F. 3d 592 (2004). On the jurisdictional question, the panel thought it sufficed that the title claim raised an issue of federal law that had to be resolved, and implicated a substantial federal interest (in construing federal tax law). The court went on to affirm the District Court's judgment on the merits. We granted certiorari on the jurisdictional question alone,[2] 543 U. S. 1042 (2005), to resolve a split within the Courts of Appeals on whether Merrell Dow Pharmaceuticals Inc. v. Thompson, 478 U. S. 804 (1986), always requires [312] a federal cause of action as a condition for exercising federal-question jurisdiction.[3] We now affirm.
19Darue was entitled to remove the quiet title action if Grable could have brought it in federal district court originally, 28 U. S. C. § 1441(a), as a civil action "arising under the Constitution, laws, or treaties of the United States," § 1331. This provision for federal-question jurisdiction is invoked by and large by plaintiffs pleading a cause of action created by federal law (e. g., claims under 42 U. S. C. § 1983). There is, however, another longstanding, if less frequently encountered, variety of federal "arising under" jurisdiction, this Court having recognized for nearly 100 years that in certain cases federal-question jurisdiction will lie over state-law claims that implicate significant federal issues. E. g., Hopkins v. Walker, 244 U. S. 486, 490-491 (1917). The doctrine captures the commonsense notion that a federal court ought to be able to hear claims recognized under state law that nonetheless turn on substantial questions of federal law, and thus justify resort to the experience, solicitude, and hope of uniformity that a federal forum offers on federal issues, see ALI, Study of the Division of Jurisdiction Between State and Federal Courts 164-166 (1968).
21The classic example is Smith v. Kansas City Title & Trust Co., 255 U. S. 180 (1921), a suit by a shareholder claiming that the defendant corporation could not lawfully buy certain bonds of the National Government because their issuance was unconstitutional. Although Missouri law provided the cause of action, the Court recognized federal-question jurisdiction because the principal issue in the case was the federal constitutionality of the bond issue. Smith thus held, in a [313] somewhat generous statement of the scope of the doctrine, that a state-law claim could give rise to federal-question jurisdiction so long as it "appears from the [complaint] that the right to relief depends upon the construction or application of [federal law]." Id., at 199.
22The Smith statement has been subject to some trimming to fit earlier and later cases recognizing the vitality of the basic doctrine, but shying away from the expansive view that mere need to apply federal law in a state-law claim will suffice to open the "arising under" door. As early as 1912, this Court had confined federal-question jurisdiction over state-law claims to those that "really and substantially involv[e] a dispute or controversy respecting the validity, construction or effect of [federal] law." Shulthis v. McDougal, 225 U. S. 561, 569. This limitation was the ancestor of Justice Cardozo's later explanation that a request to exercise federal-question jurisdiction over a state action calls for a "common-sense accommodation of judgment to [the] kaleidoscopic situations" that present a federal issue, in "a selective process which picks the substantial causes out of the web and lays the other ones aside." Gully v. First Nat. Bank in Meridian, 299 U. S. 109, 117-118 (1936). It has in fact become a constant refrain in such cases that federal jurisdiction demands not only a contested federal issue, but a substantial one, indicating a serious federal interest in claiming the advantages thought to be inherent in a federal forum. E. g., Chicago v. International College of Surgeons, 522 U. S. 156, 164 (1997); Merrell Dow, supra, at 814, and n. 12; Franchise Tax Bd. of Cal. v. Construction Laborers Vacation Trust for Southern Cal., 463 U. S. 1, 28 (1983).
23But even when the state action discloses a contested and substantial federal question, the exercise of federal jurisdiction is subject to a possible veto. For the federal issue will ultimately qualify for a federal forum only if federal jurisdiction is consistent with congressional judgment about the sound division of labor between state and federal courts governing [314] the application of § 1331. Thus, Franchise Tax Bd. explained that the appropriateness of a federal forum to hear an embedded issue could be evaluated only after considering the "welter of issues regarding the interrelation of federal and state authority and the proper management of the federal judicial system." Id., at 8. Because arising-under jurisdiction to hear a state-law claim always raises the possibility of upsetting the state-federal line drawn (or at least assumed) by Congress, the presence of a disputed federal issue and the ostensible importance of a federal forum are never necessarily dispositive; there must always be an assessment of any disruptive portent in exercising federal jurisdiction. See also Merrell Dow, supra, at 810.
24These considerations have kept us from stating a "single, precise, all-embracing" test for jurisdiction over federal issues embedded in state-law claims between nondiverse parties. Christianson v. Colt Industries Operating Corp., 486 U. S. 800, 821 (1988) (STEVENS, J., concurring). We have not kept them out simply because they appeared in state raiment, as Justice Holmes would have done, see Smith, supra, at 214 (dissenting opinion), but neither have we treated "federal issue" as a password opening federal courts to any state action embracing a point of federal law. Instead, the question is, does a state-law claim necessarily raise a stated federal issue, actually disputed and substantial, which a federal forum may entertain without disturbing any congressionally approved balance of federal and state judicial responsibilities.
25This case warrants federal jurisdiction. Grable's state complaint must specify "the facts establishing the superiority of [its] claim," Mich. Ct. Rule 3.411(B)(2)(c) (West 2005), and Grable has premised its superior title claim on a failure by the IRS to give it adequate notice, as defined by federal [315] law. Whether Grable was given notice within the meaning of the federal statute is thus an essential element of its quiet title claim, and the meaning of the federal statute is actually in dispute; it appears to be the only legal or factual issue contested in the case. The meaning of the federal tax provision is an important issue of federal law that sensibly belongs in a federal court. The Government has a strong interest in the "prompt and certain collection of delinquent taxes," United States v. Rodgers, 461 U. S. 677, 709 (1983), and the ability of the IRS to satisfy its claims from the property of delinquents requires clear terms of notice to allow buyers like Darue to satisfy themselves that the Service has touched the bases necessary for good title. The Government thus has a direct interest in the availability of a federal forum to vindicate its own administrative action, and buyers (as well as tax delinquents) may find it valuable to come before judges used to federal tax matters. Finally, because it will be the rare state title case that raises a contested matter of federal law, federal jurisdiction to resolve genuine disagreement over federal tax title provisions will portend only a microscopic effect on the federal-state division of labor. See n. 3, infra.
28This conclusion puts us in venerable company, quiet title actions having been the subject of some of the earliest exercises of federal-question jurisdiction over state-law claims. In Hopkins, 244 U. S., at 490-491, the question was federal jurisdiction over a quiet title action based on the plaintiffs' allegation that federal mining law gave them the superior claim. Just as in this case, "the facts showing the plaintiffs' title and the existence and invalidity of the instrument or record sought to be eliminated as a cloud upon the title are essential parts of the plaintiffs' cause of action."[4] Id., at [316] 490. As in this case again, "it is plain that a controversy respecting the construction and effect of the [federal] laws is involved and is sufficiently real and substantial." Id., at 489. This Court therefore upheld federal jurisdiction in Hopkins, as well as in the similar quiet title matters of Northern Pacific R. Co. v. Soderberg, 188 U. S. 526, 528 (1903), and Wilson Cypress Co. v. Del Pozo y Marcos, 236 U. S. 635, 643-644 (1915). Consistent with those cases, the recognition of federal jurisdiction is in order here.
29Merrell Dow Pharmaceuticals Inc. v. Thompson, 478 U. S. 804 (1986), on which Grable rests its position, is not to the contrary. Merrell Dow considered a state tort claim resting in part on the allegation that the defendant drug company had violated a federal misbranding prohibition, and was thus presumptively negligent under Ohio law. Id., at 806. The Court assumed that federal law would have to be applied to resolve the claim, but after closely examining the strength of the federal interest at stake and the implications of opening the federal forum, held federal jurisdiction unavailable. Congress had not provided a private federal cause of action for violation of the federal branding requirement, and the Court found "it would . . . flout, or at least undermine, congressional intent to conclude that federal courts might nevertheless exercise federal-question jurisdiction and provide remedies for violations of that federal statute solely because the violation . . . is said to be a . . . `proximate cause' under state law." Id., at 812.
31[317] Because federal law provides for no quiet title action that could be brought against Darue,[5] Grable argues that there can be no federal jurisdiction here, stressing some broad language in Merrell Dow (including the passage just quoted) that on its face supports Grable's position, see Note, Mr. Smith Goes to Federal Court: Federal Question Jurisdiction over State Law Claims Post-Merrell Dow, 115 Harv. L. Rev. 2272, 2280-2282 (2002) (discussing split in Courts of Appeals over private right of action requirement after Merrell Dow). But an opinion is to be read as a whole, and Merrell Dow cannot be read whole as overturning decades of precedent, as it would have done by effectively adopting the Holmes dissent in Smith, see supra, at 314, and converting a federal cause of action from a sufficient condition for federal-question jurisdiction[6] into a necessary one.
32In the first place, Merrell Dow disclaimed the adoption of any bright-line rule, as when the Court reiterated that "in exploring the outer reaches of § 1331, determinations about federal jurisdiction require sensitive judgments about congressional intent, judicial power, and the federal system." 478 U. S., at 810. The opinion included a lengthy footnote explaining that questions of jurisdiction over state-law claims require "careful judgments," id., at 814, about the "nature of the federal interest at stake," id., at 814, n. 12 (emphasis deleted). And as a final indication that it did not mean to make a federal right of action mandatory, it expressly approved the exercise of jurisdiction sustained in Smith, despite the want of any federal cause of action available to Smith's shareholder plaintiff. 478 U. S., at 814, n. 12. [318] Merrell Dow then, did not toss out, but specifically retained, the contextual enquiry that had been Smith's hallmark for over 60 years. At the end of Merrell Dow, Justice Holmes was still dissenting.
33Accordingly, Merrell Dow should be read in its entirety as treating the absence of a federal private right of action as evidence relevant to, but not dispositive of, the "sensitive judgments about congressional intent" that § 1331 requires. The absence of any federal cause of action affected Merrell Dow's result two ways. The Court saw the fact as worth some consideration in the assessment of substantiality. But its primary importance emerged when the Court treated the combination of no federal cause of action and no preemption of state remedies for misbranding as an important clue to Congress's conception of the scope of jurisdiction to be exercised under § 1331. The Court saw the missing cause of action not as a missing federal door key, always required, but as a missing welcome mat, required in the circumstances, when exercising federal jurisdiction over a state misbranding action would have attracted a horde of original filings and removal cases raising other state claims with embedded federal issues. For if the federal labeling standard without a federal cause of action could get a state claim into federal court, so could any other federal standard without a federal cause of action. And that would have meant a tremendous number of cases.
34One only needed to consider the treatment of federal violations generally in garden variety state tort law. "The violation of federal statutes and regulations is commonly given negligence per se effect in state tort proceedings."[7] Restatement [319] (Third) of Torts § 14, Reporters' Note, Comment a, p. 195 (Tent. Draft No. 1, Mar. 28, 2001) (hereinafter Restatement). See also W. Keeton, D. Dobbs, R. Keeton, & D. Owen, Prosser and Keeton on Law of Torts § 36, p. 221, n. 9 (5th ed. 1984) ("[T]he breach of a federal statute may support a negligence per se claim as a matter of state law" (collecting authority)). A general rule of exercising federal jurisdiction over state claims resting on federal mislabeling and other statutory violations would thus have heralded a potentially enormous shift of traditionally state cases into federal courts. Expressing concern over the "increased volume of federal litigation," and noting the importance of adhering to "legislative intent," Merrell Dow thought it improbable that the Congress, having made no provision for a federal cause of action, would have meant to welcome any state-law tort case implicating federal law "solely because the violation of the federal statute is said to [create] a rebuttable presumption [of negligence] . . . under state law." 478 U. S., at 811-812 (internal quotation marks omitted). In this situation, no welcome mat meant keep out. Merrell Dow's analysis thus fits within the framework of examining the importance of having a federal forum for the issue, and the consistency of such a forum with Congress's intended division of labor between state and federal courts.
35As already indicated, however, a comparable analysis yields a different jurisdictional conclusion in this case. Although Congress also indicated ambivalence in this case by providing no private right of action to Grable, it is the rare state quiet title action that involves contested issues of federal law, see n. 3, supra. Consequently, jurisdiction over actions like Grable's would not materially affect, or threaten to affect, the normal currents of litigation. Given the absence of threatening structural consequences and the clear interest the Government, its buyers, and its delinquents have in the availability of a federal forum, there is no good reason to [320] shirk from federal jurisdiction over the dispositive and contested federal issue at the heart of the state-law title claim.[8]
36The judgment of the Court of Appeals, upholding federal jurisdiction over Grable's quiet title action, is affirmed.
38It is so ordered.
39The Court faithfully applies our precedents interpreting 28 U. S. C. § 1331 to authorize federal-court jurisdiction over some cases in which state law creates the cause of action but requires determination of an issue of federal law, e. g., Smith v. Kansas City Title & Trust Co., 255 U. S. 180 (1921); Merrell Dow Pharmaceuticals Inc. v. Thompson, 478 U. S. 804 (1986). In this case, no one has asked us to overrule those precedents and adopt the rule Justice Holmes set forth in American Well Works Co. v. Layne & Bowler Co., 241 U. S. 257 (1916), limiting § 1331 jurisdiction to cases in which federal law creates the cause of action pleaded on the face of the plaintiff's complaint. Id., at 260. In an appropriate case, and perhaps with the benefit of better evidence as to the original meaning of § 1331's text, I would be willing to consider that course.[9]
41[321] Jurisdictional rules should be clear. Whatever the virtues of the Smith standard, it is anything but clear. Ante, at 313 (the standard "calls for a `common-sense accommodation of judgment to [the] kaleidoscopic situations' that present a federal issue, in `a selective process which picks the substantial causes out of the web and lays the other ones aside'" (quoting Gully v. First Nat. Bank in Meridian, 299 U. S. 109, 117-118 (1936))); ante, at 314 ("[T]he question is, does a state-law claim necessarily raise a stated federal issue, actually disputed and substantial, which a federal forum may entertain without disturbing any congressionally approved balance of federal and state judicial responsibilities"); ante, at 317, 318 ("`[D]eterminations about federal jurisdiction require sensitive judgments about congressional intent, judicial power, and the federal system'"; "the absence of a federal private right of action [is] evidence relevant to, but not dispositive of, the `sensitive judgments about congressional intent' that § 1331 requires" (quoting Merrell Dow, supra, at 810)).
42Whatever the vices of the American Well Works rule, it is clear. Moreover, it accounts for the "`vast majority'" of cases that come within § 1331 under our current case law, Merrell Dow, supra, at 808 (quoting Franchise Tax Bd. of Cal. v. Construction Laborers Vacation Trust for Southern Cal., 463 U. S. 1, 9 (1983)) — further indication that trying to sort out which cases fall within the smaller Smith category may not be worth the effort it entails. See R. Fallon, D. Meltzer, & D. Shapiro, Hart and Wechsler's The Federal [322] Courts and the Federal System 885-886 (5th ed. 2003). Accordingly, I would be willing in appropriate circumstances to reconsider our interpretation of § 1331.
43[1] Mr. Zagrans filed a brief for Jerome R. Mikulski et ux. as amici curiae urging reversal.
44[2] Accordingly, we have no occasion to pass upon the proper interpretation of the federal tax provision at issue here.
45[3] Compare Seinfeld v. Austen, 39 F. 3d 761, 764 (CA7 1994) (finding that federal-question jurisdiction over a state-law claim requires a parallel federal private right of action), with Ormet Corp. v. Ohio Power Co., 98 F. 3d 799, 806 (CA4 1996) (finding that a federal private action is not required).
46[4] The quiet title cases also show the limiting effect of the requirement that the federal issue in a state-law claim must actually be in dispute to justify federal-question jurisdiction. In Shulthis v. McDougal, 225 U. S. 561 (1912), this Court found that there was no federal-question jurisdiction to hear a plaintiff's quiet title claim in part because the federal statutes on which title depended were not subject to "any controversy respecting their validity, construction, or effect." Id., at 570. As the Court put it, the requirement of an actual dispute about federal law was "especially" important in "suit[s] involving rights to land acquired under a law of the United States," because otherwise "every suit to establish title to land in the central and western states would so arise [under federal law], as all titles in those States are traceable back to those laws." Id., at 569-570.
47[5] Federal law does provide a quiet title cause of action against the Federal Government. 28 U. S. C. § 2410. That right of action is not relevant here, however, because the Federal Government no longer has any interest in the property, having transferred its interest to Darue through the quitclaim deed.
48[6] For an extremely rare exception to the sufficiency of a federal right of action, see Shoshone Mining Co. v. Rutter, 177 U. S. 505, 507 (1900).
49[7] Other jurisdictions treat a violation of a federal statute as evidence of negligence or, like Ohio itself in Merrell Dow Pharmaceuticals Inc. v. Thompson, 478 U. S. 804 (1986), as creating a rebuttable presumption of negligence. Restatement § 14, Reporters' Note, Comment c, at 196. Either approach could still implicate issues of federal law.
50[8] At oral argument Grable's counsel espoused the position that after Merrell Dow, federal-question jurisdiction over state-law claims absent a federal right of action could be recognized only where a constitutional issue was at stake. There is, however, no reason in text or otherwise to draw such a rough line. As Merrell Dow itself suggested, constitutional questions may be the more likely ones to reach the level of substantiality that can justify federal jurisdiction. 478 U. S., at 814, n. 12. But a flat ban on statutory questions would mechanically exclude significant questions of federal law like the one this case presents.
51[9] This Court has long construed the scope of the statutory grant of federal-question jurisdiction more narrowly than the scope of the constitutional grant of such jurisdiction. See Merrell Dow Pharmaceuticals Inc. v. Thompson, 478 U. S. 804, 807-808 (1986). I assume for present purposes that this distinction is proper — that is, that the language of 28 U. S. C. § 1331, "[t]he district courts shall have original jurisdiction of all civil actions arising under the Constitution, laws, or treaties of the United States" (emphasis added), is narrower than the language of Art. III, § 2, cl. 1, of the Constitution, "[t]he judicial Power shall extend to all Cases, in Law and Equity, arising under this Constitution, the Laws of the United States, and Treaties made, or which shall be made, under their Authority . . ." (emphases added).
This is summarized in the cheat sheet and in Gunn v. Minton, so reading it is OPTIONAL if you want more.
Supreme Court of United States.
[682] Anthony F. Shelley argued the cause for petitioner. With him on the briefs were Alan I. Horowitz, Laura G. Ferguson, Kathleen M. Sullivan, Roger G. Wilson, Paul F. Brown, and William A. Breskin.
8Sri Srinivasan argued the cause for the United States as amicus curiae urging reversal. On the brief were Solicitor General Clement, Assistant Attorney General Keisler, Deputy Solicitor General Kneedler, James A. Feldman, Mark B. Stern, Alisa B. Klein, Mark A. Robbins, and James S. Green.
9Thomas J. Stock argued the cause for respondent. With him on the brief were Harry Raptakis and Victor A. Carr.[1]
10The Federal Employees Health Benefits Act of 1959 (FEHBA), 5 U. S. C. § 8901 et seq. (2000 ed. and Supp. III), establishes a comprehensive program of health insurance for federal employees. The Act authorizes the Office of Personnel Management (OPM) to contract with private carriers to offer federal employees an array of health-care plans. See § 8902(a) (2000 ed.). Largest of the plans for which OPM has contracted, annually since 1960, is the Blue Cross Blue Shield Service Benefit Plan (Plan), administered by local Blue Cross Blue Shield companies. This case concerns the proper forum for reimbursement claims when a Plan beneficiary, injured in an accident, whose medical bills have been paid by the Plan administrator, recovers damages (unaided by the carrier-administrator) in a state-court tort action against a third party alleged to have caused the accident.
12[683] FEHBA contains a preemption clause, § 8902(m)(1), displacing state law on issues relating to "coverage or benefits" afforded by health-care plans. The Act contains no provision addressing the subrogation or reimbursement rights of carriers. Successive annual contracts between OPM and the Blue Cross Blue Shield Association (BCBSA) have obligated the carrier to make "a reasonable effort" to recoup amounts paid for medical care. App. 95, 125. The statement of benefits distributed by the carrier alerts enrollees that all recoveries they receive "must be used to reimburse the Plan for benefits paid." Id., at 132; see also id., at 146, 152.
13The instant case originated when the administrator of a Plan beneficiary's estate pursued tort litigation in state court against parties alleged to have caused the beneficiary's injuries. The carrier had notice of the state-court action, but took no part in it. When the tort action terminated in a settlement, the carrier filed suit in federal court seeking reimbursement of the full amount it had paid for the beneficiary's medical care. The question presented is whether 28 U. S. C. § 1331 (authorizing jurisdiction over "civil actions arising under the . . . laws . . . of the United States") encompasses the carrier's action. We hold it does not.
14FEHBA itself provides for federal-court jurisdiction only in actions against the United States. Congress could decide and provide that reimbursement claims of the kind here involved warrant the exercise of federal-court jurisdiction. But claims of this genre, seeking recovery from the proceeds of state-court litigation, are the sort ordinarily resolved in state courts. Federal courts should await a clear signal from Congress before treating such auxiliary claims as "arising under" the laws of the United States.
15FEHBA assigns to OPM responsibility for negotiating and regulating health-benefits plans for federal employees. See [684] 5 U. S. C. § 8902(a). OPM contracts with carriers, FEHBA instructs, "shall contain a detailed statement of benefits offered and shall include such maximums, limitations, exclusions, and other definitions of benefits as [OPM] considers necessary or desirable." § 8902(d). Pursuant to FEHBA, OPM entered into a contract in 1960 with the BCBSA to establish a nationwide fee-for-service health plan, the terms of which are renegotiated annually. As FEHBA prescribes, the Federal Government pays about 75% of the premiums; the enrollee pays the rest. § 8906(b). Premiums thus shared are deposited in a special Treasury Fund, the Federal Employees Health Benefits Fund, § 8909(a). Carriers draw against the Fund to pay for covered health-care benefits. Ibid.; see also 48 CFR § 1632.170(b) (2005).
17The contract between OPM and the BCBSA provides: "By enrolling or accepting services under this contract, [enrollees and their eligible dependents] are obligated to all terms, conditions, and provisions of this contract." App. 90. An appended brochure sets out the benefits the carrier shall provide, see id., at 89, and the carrier's subrogation and recovery rights, see id., at 100. Each enrollee, as FEHBA directs, receives a statement of benefits conveying information about the Plan's coverage and conditions. 5 U. S. C. § 8907(b). Concerning reimbursement and subrogation, matters FEHBA itself does not address, the BCBSA Plan's statement of benefits reads in part:
18"If another person or entity . . . causes you to suffer an injury or illness, and if we pay benefits for that injury or illness, you must agree to the following:19
"All recoveries you obtain (whether by lawsuit, settlement, or otherwise), no matter how described or designated, must be used to reimburse us in full for benefits we paid. Our share of any recovery extends only to the amount of benefits we have paid or will pay to you or, if applicable, to your heirs, administrators, successors, or assignees.20
.....21
[685] "If you do not seek damages for your illness or injury, you must permit us to initiate recovery on your behalf (including the right to bring suit in your name). This is called subrogation.22
"If we pursue a recovery of the benefits we have paid, you must cooperate in doing what is reasonably necessary to assist us. You must not take any action that may prejudice our rights to recover." App. 165.[2]23
If the participant does not voluntarily reimburse the Plan, the contract requires the carrier to make a "reasonable effort to seek recovery of amounts . . . it is entitled to recover in cases . . . brought to its attention." Id., at 95, 125. Pursuant to the OPM—BCBSA master contract, reimbursements obtained by the carrier must be returned to the Treasury Fund. See id., at 92, 118-119.
24FEHBA contains a preemption provision, which originally provided:
25"The provisions of any contract under this chapter which relate to the nature or extent of coverage or benefits (including payments with respect to benefits) shall supersede and preempt any State or local law, or any regulation issued thereunder, which relates to health insurance or plans to the extent that such law or regulation is inconsistent with such contractual provisions." 5 U. S. C. § 8902(m)(1) (1994 ed.).26
[686] To ensure uniform coverage and benefits under plans OPM negotiates for federal employees, see H. R. Rep. No. 95-282, p. 1 (1977), § 8902(m)(1) preempted "State laws or regulations which specify types of medical care, providers of care, extent of benefits, coverage of family members, age limits for family members, or other matters relating to health benefits or coverage," id., at 4-5 (noting that some States mandated coverage for services not included in federal plans, for example, chiropractic services). In 1998, Congress amended § 8902(m)(1) by deleting the words "to the extent that such law or regulation is inconsistent with such contractual provisions." Thus, under § 8902(m)(1) as it now reads, state law—whether consistent or inconsistent with federal plan provisions—is displaced on matters of "coverage or benefits."
27FEHBA contains but one provision addressed to federalcourt jurisdiction. That provision vests in federal district courts "original jurisdiction, concurrent with the United States Court of Federal Claims, of a civil action or claim against the United States founded on this chapter." § 8912. The purpose of this provision—evident from its reference to the Court of Federal Claims—was to carve out an exception to the statutory rule that claims brought against the United States and exceeding $10,000 must originate in the Court of Federal Claims. See 28 U. S. C. § 1346(a)(2) (establishing district courts' jurisdiction, concurrent with the Court of Federal Claims, over claims against the United States that do not exceed $10,000); see also S. Rep. No. 1654, 83d Cong., 2d Sess., 4-5 (1954) (commenting, with respect to an identical provision in the Federal Employees' Group Life Insurance Act, 5 U. S. C. § 8715, that the provision "would extend the jurisdiction of United States district courts above the $10,000 limitation now in effect").
28Under a 1995 OPM regulation, suits contesting final OPM action denying health benefits "must be brought against OPM and not against the carrier or carrier's subcontractors." 5 CFR § 890.107(c) (2005). While this regulation channels [687] disputes over coverage or benefits into federal court by designating a United States agency (OPM) sole defendant, no law opens federal courts to carriers seeking reimbursement from beneficiaries or recovery from tortfeasors. Cf. 29 U. S. C. § 1132(e)(1) (provision of the Employee Retirement Income Security Act (ERISA) vesting in federal district courts "exclusive jurisdiction of civil actions under this subchapter"). And nothing in FEHBA's text prescribes a federal rule of decision for a carrier's claim against its insured or an alleged tortfeasor to share in the proceeds of a state-court tort action.
29Petitioner Empire HealthChoice Assurance, Inc., doing business as Empire Blue Cross Blue Shield (Empire), is the entity that administers the BCBSA Plan as it applies to federal employees in New York State. Respondent Denise Finn McVeigh (McVeigh) is the administrator of the estate of Joseph E. McVeigh (Decedent), a former enrollee in the Plan. The Decedent was injured in an accident in 1997. Plan payments for the medical care he received between 1997 and his death in 2001 amounted to $157,309. McVeigh, on behalf of herself, the Decedent, and a minor child, commenced tort litigation in state court against parties alleged to have caused Decedent's injuries. On learning that the parties to the state-court litigation had agreed to settle the tort claims, Empire sought to recover the $157,309 it had paid out for the Decedent's medical care.[3] Of the $3,175,000 for which the settlement provided, McVeigh, in response to Empire's asserted reimbursement right, agreed to place $100,000 in escrow.
31Empire then filed suit in the United States District Court for the Southern District of New York, alleging that McVeigh [688] was in breach of the reimbursement provision of the Plan. As relief, Empire demanded $157,309, with no offset for attorney's fees or other litigation costs McVeigh incurred in pursuing the state-court settlement. McVeigh moved to dismiss on various grounds, among them, lack of subject-matter jurisdiction. See 396 F. 3d 136, 139 (CA2 2005). Answering McVeigh's motion, Empire urged that the District Court had jurisdiction under 28 U. S. C. § 1331 because federal common law governed its reimbursement claim. In the alternative, Empire asserted that the Plan itself constituted federal law. See 396 F. 3d, at 140. The District Court rejected both arguments and granted McVeigh's motion to dismiss for want of subject-matter jurisdiction. Ibid.
32A divided panel of the Court of Appeals for the Second Circuit affirmed, holding that "Empire's clai[m] arise[s] under state law." Id., at 150. FEHBA's text, the court observed, contains no authorization for carriers "to vindicate [in federal court] their rights [against enrollees] under FEHBAauthorized contracts"; therefore, the court concluded, "federal jurisdiction exists over this dispute only if federal common law governs Empire's claims." Id., at 140. Quoting Boyle v. United Technologies Corp., 487 U. S. 500, 507, 508 (1988), the appeals court stated that courts may create federal common law only when "the operation of state law would (1) `significant[ly] conflict' with (2) `uniquely federal interest[s].'" 396 F. 3d, at 140.
33Empire maintained that its contract-derived claim against McVeigh implicated "`uniquely federal interest[s],'" because (1) reimbursement directly affects the United States Treasury and the cost of providing health benefits to federal employees; and (2) Congress had expressed its interest in maintaining uniformity among the States on matters relating to federal health-plan benefits. Id., at 141. The court acknowledged that the case involved distinctly federal interests, but found that Empire had not identified "specific ways in which the operation of state contract law, or indeed of [689] other laws of general application, would conflict materially with the federal policies underlying FEHBA in the circumstances presented." Id., at 150 (Sack, J., concurring); see id., at 142.
34The Court of Appeals next considered and rejected Empire's argument that FEHBA's preemption provision, 5 U. S. C. § 8902(m)(1), independently conferred federal jurisdiction. 396 F. 3d, at 145-149. That provision, the court observed, is "a limited preemption clause that the instant dispute does not trigger." Id., at 145. Unlike § 8912, which "authoriz[es] federal jurisdiction over FEHBA-related . . . claims `against the United States,'" the court noted, § 8902(m)(1) "makes no reference to a federal right of action [in] or to federal jurisdiction [over]" the contract-derived reimbursement claim here at issue. 396 F. 3d, at 145, and n. 7.
35Judge Raggi dissented. Id., at 151. In her view, FEHBA's preemption provision, § 8902(m)(1), as amended in 1998, both calls for the application of uniform federal common law to terms in a FEHBA plan and establishes federal jurisdiction over Empire's complaint.
36We granted certiorari, 546 U. S. 1085 (2005), to resolve a conflict among lower federal courts concerning the proper forum for claims of the kind Empire asserts. Compare Blue Cross & Blue Shield of Ill. v. Cruz, 396 F. 3d 793, 799-800 (CA7 2005) (upholding federal jurisdiction), Caudill v. Blue Cross & Blue Shield of N. C., 999 F. 2d 74, 77 (CA4 1993) (same), and Medcenters Health Care v. Ochs, 854 F. Supp. 589, 593, and n. 3 (Minn. 1993) (same), aff'd, 26 F. 3d 865 (CA8 1994), with Goepel v. National Postal Mail Handlers Union, 36 F. 3d 306, 314-315 (CA3 1994) (rejecting federal jurisdiction), and 396 F. 3d, at 139 (decision below) (same).
37Title 28 U. S. C. § 1331 vests in federal district courts "original jurisdiction" over "all civil actions arising under the Constitution, laws, or treaties of the United States." A [690] case "aris[es] under" federal law within the meaning of § 1331, this Court has said, if "a well-pleaded complaint establishes either that federal law creates the cause of action or that the plaintiff's right to relief necessarily depends on resolution of a substantial question of federal law." Franchise Tax Bd. of Cal. v. Construction Laborers Vacation Trust for Southern Cal., 463 U. S. 1, 27-28 (1983).
39Empire and the United States, as amicus curiae, present two principal arguments in support of federal-question jurisdiction. Emphasizing our opinion in Jackson Transit Authority v. Transit Union, 457 U. S. 15, 22 (1982), and cases cited therein, they urge that Empire's complaint raises a federal claim because it seeks to vindicate a contractual right contemplated by a federal statute, a right that Congress intended to be federal in nature. See Brief for Petitioner 14-31; Brief for United States 12-23. FEHBA's preemption provision, Empire and the United States contend, demonstrates Congress' intent in this regard. The United States argues, alternatively, that there is federal jurisdiction here, as demonstrated by our recent decision in Grable & Sons Metal Products, Inc. v. Darue Engineering & Mfg., 545 U. S. 308 (2005), because "federal law is a necessary element of [Empire's] claim." Brief for United States 25; accord Brief for Petitioner 41, n. 5. We address these arguments in turn. But first, we respond to the dissent's view that Empire and the United States have engaged in unnecessary labor, for Clearfield Trust Co. v. United States, 318 U. S. 363 (1943), provides "a basis for federal jurisdiction" in this case. Post, at 702.
40Clearfield is indeed a pathmarking precedent on the authority of federal courts to fashion uniform federal common law on issues of national concern. See Friendly, In Praise of Erie—and of the New Federal Common Law, 39 N. Y. U. L. Rev. 383, 409-410 (1964). But the dissent is mistaken in supposing that the Clearfield doctrine covers this case. [691] Clearfield was a suit by the United States to recover from a bank the amount paid on a Government check on which the payee's name had been forged. 318 U. S., at 365. Because the United States was the plaintiff, federal-court jurisdiction was solidly grounded. See ibid. ("This suit was instituted. . . by the United States . . ., the jurisdiction of the federal District Court being invoked pursuant to the provisions of § 24(1) of the Judicial Code, 28 U. S. C. § 41(1)," now contained in 28 U. S. C. §§ 1332, 1345, 1359). The case presented a vertical choice-of-law issue: Did state law under Erie R. Co. v. Tompkins, 304 U. S. 64 (1938), or a court-fashioned federal rule of decision (federal common law) determine the merits of the controversy? The Court held that "[t]he rights and duties of the United States on commercial paper which it issues are governed by federal rather than [state] law." 318 U. S., at 366.
42In post-Clearfield decisions, and with the benefit of enlightened commentary, see, e. g., Friendly, supra, at 410, the Court has "made clear that uniform federal law need not be applied to all questions in federal government litigation, even in cases involving government contracts," R. Fallon, D. Meltzer, & D. Shapiro, Hart and Wechsler's The Federal Courts and the Federal System 700 (5th ed. 2003) (hereinafter Hart and Wechsler).[4] "[T]he prudent course," we have recognized, is often "to adopt the readymade body of state [692] law as the federal rule of decision until Congress strikes a different accommodation." United States v. Kimbell Foods, Inc., 440 U. S. 715, 740 (1979).
43Later, in Boyle, the Court telescoped the appropriate inquiry, focusing it on the straightforward question whether the relevant federal interest warrants displacement of state law. See 487 U. S., at 507, n. 3. Referring simply to "the displacement of state law," the Court recognized that prior cases had treated discretely (1) the competence of federal courts to formulate a federal rule of decision, and (2) the appropriateness of declaring a federal rule rather than borrowing, incorporating, or adopting state law in point. The Court preferred "the more modest terminology," questioning whether "the distinction between displacement of state law and displacement of federal law's incorporation of state law ever makes a practical difference." Ibid. Boyle made two further observations here significant. First, Boyle explained, the involvement of "an area of uniquely federal interest. . . establishes a necessary, not a sufficient, condition for the displacement of state law." Id., at 507. Second, in some cases, an "entire body of state law" may conflict with the federal interest and therefore require replacement. Id., at 508. But in others, the conflict is confined, and "only particular elements of state law are superseded." Ibid.
44The dissent describes this case as pervasively federal, post, at 702, and "the provisions . . . here [as] just a few scattered islands in a sea of federal contractual provisions," post, at 709. But there is nothing "scattered" about the provisions on reimbursement and subrogation in the OPM-BCBSA master contract. See supra, at 684-685. Those provisions are linked together and depend upon a recovery from a third party under terms and conditions ordinarily governed by state law. See infra, at 698.[5] The Court of [693] Appeals, whose decision we review, trained on the matter of reimbursement, not, as the dissent does, on FEHBA-authorized contracts at large. So focused, the appeals court determined that Empire has not demonstrated a "significant conflict . . . between an identifiable federal policy or interest and the operation of state law." 396 F. 3d, at 150 (Sack, J., concurring) (quoting Boyle, 487 U. S., at 507); see 396 F. 3d, at 140-141. Unless and until that showing is made, there is no cause to displace state law, much less to lodge this case in federal court.
45We take up next Empire's Jackson Transit-derived argument, which is, essentially, a more tailored variation of the theme sounded in the dissent. It is undisputed that Congress has not expressly created a federal right of action enabling insurance carriers like Empire to sue health-care beneficiaries in federal court to enforce reimbursement rights under contracts contemplated by FEHBA. Empire and the United States nevertheless argue that, under our 1982 opinion in Jackson Transit, Empire's claim for reimbursement, arising under the contract between OPM and the BCBSA, "states a federal claim" because Congress intended all rights and duties stemming from that contract to be "federal in nature." Brief for United States as Amicus Curiae 12; see Brief for Petitioner 18-29. We are not persuaded by this argument.
47The reliance placed by Empire and the United States on Jackson Transit is surprising, for that decision held there was no federal jurisdiction over the claim in suit. The federal statute there involved, § 13(c) of the Urban Mass Transportation Act of 1964 (UMTA), 78 Stat. 307 (then codified at 49 U. S. C. § 1609(c) (1976 ed.)), conditioned a governmental unit's receipt of federal funds to acquire a privately owned transit company on preservation of collective-bargaining rights enjoyed by the acquired company's employees. 457 U. S., at 17-18. The city of Jackson, Tennessee, with federal financial assistance, acquired a failing private bus company [694] and turned it into a public entity, the Jackson Transit Authority. Id., at 18. To satisfy the condition on federal aid, the transit authority entered into a "§ 13(c) agreement" with the union that represented the private company's employees, and the Secretary of Labor certified that agreement as "fair and equitable." Ibid. (internal quotation marks omitted).
48For several years thereafter, the transit authority covered its unionized workers in a series of collective-bargaining agreements. Eventually, however, the Authority notified the union that it would no longer adhere to collective-bargaining undertakings. Id., at 19. The union commenced suit in federal court alleging breach of the § 13(c) agreement and of the latest collective-bargaining agreement. Ibid. This Court determined that the case did not arise under federal law, but was instead "governed by state law [to be] applied in state cour[t]." Id., at 29.
49The Court acknowledged in Jackson Transit that "on several occasions [we had] determined that a plaintiff stated a federal claim when he sued to vindicate contractual rights set forth by federal statutes, [even though] the relevant statutes lacked express provisions creating federal causes of action." Id., at 22 (emphasis added) (citing Machinists v. Central Airlines, Inc., 372 U. S. 682 (1963) (union had a federal right of action to enforce an airline-adjustment-board award included in a collective-bargaining contract pursuant to a provision of the Railway Labor Act); Norfolk & Western R. Co. v. Nemitz, 404 U. S. 37 (1971) (railroad's employees stated federal claims when they sought to enforce assurances made by the railroad to secure Interstate Commerce Commission approval of a consolidation under a provision of the Interstate Commerce Act); Transamerica Mortgage Advisors, Inc. v. Lewis, 444 U. S. 11, 18-19 (1979) (permitting federal suit for rescission of a contract declared void by a provision of the Investment Advisers Act of 1940)). But prior decisions, we said, "d[id] not dictate the result in [the Jackson Transit] case," for in each case, "the critical factor" in determining "the scope of [695] rights and remedies under a federal statute . . . is the congressional intent behind the particular provision at issue." 457 U. S., at 22.
50"In some ways," the Jackson Transit Court said, the UMTA "seem[ed] to make § 13(c) agreements and collective-bargaining contracts creatures of federal law." Id., at 23. In this regard, the Court noted, § 13(c)
51"demand[ed] `fair and equitable arrangements' as prerequisites for federal aid; it require[d] the approval of the Secretary of Labor for those arrangements; it specifie[d] five different varieties of protective provisions that must be included among the § 13(c) arrangements; and it expressly incorporate[d] the protective arrangements into the grant contract between the recipient and the Federal Government." Ibid. (quoting 49 U. S. C. § 1609(c) (1976 ed.)).52
But there were countervailing considerations. The Court observed that "labor relations between local governments and their employees are the subject of a longstanding statutory exemption from the National Labor Relations Act." 457 U. S., at 23. "Section 13(c)," the Court continued, "evince[d] no congressional intent to upset the decision in the [NLRA] to permit state law to govern the relationships between local governmental entities and the unions representing their employees." Id., at 23-24. Legislative history was corroborative. "A consistent theme," the Court found, "[ran] throughout the consideration of § 13(c): Congress intended that labor relations between transit workers and local governments would be controlled by state law." Id., at 24. We therefore held that the union had come to the wrong forum. Congress had indeed provided for § 13(c) agreements and collective-bargaining contracts stemming from them, but in the Court's judgment, the union's proper recourse for enforcement of those contracts was a suit in state court.
53[696] Measured against the Court's discussion in Jackson Transit about when a claim arises under federal law, Empire's contract-derived claim for reimbursement is not a "creatur[e] of federal law." Id., at 23. True, distinctly federal interests are involved. Principally, reimbursements are credited to a federal fund, and the OPM-BCBSA master contract could be described as "federal in nature" because it is negotiated by a federal agency and concerns federal employees. See supra, at 683-684. But, as in Jackson Transit, countervailing considerations control. Among them, the reimbursement right in question, predicated on a FEHBA-authorized contract, is not a prescription of federal law. See supra, at 684. And, of prime importance, "Congress considered jurisdictional issues in enacting FEHBA[,] . . . confer[ring] federal jurisdiction where it found it necessary to do so." 396 F. 3d, at 145, n. 7.
54FEHBA's jurisdictional provision, 5 U. S. C. § 8912, opens the federal district-court door to civil actions "against the United States." See supra, at 686. OPM's regulation, 5 CFR § 890.107(c) (2005), instructs enrollees who seek to challenge benefit denials to proceed in court against OPM "and not against the carrier or carrier's subcontractors." See ibid. Read together, these prescriptions "ensur[e] that suits brought by beneficiaries for denial of benefits will land in federal court." 396 F. 3d, at 145, n. 7. Had Congress found it necessary or proper to extend federal jurisdiction further, in particular, to encompass contract-derived reimbursement claims between carriers and insured workers, it would have been easy enough for Congress to say so. Cf. 29 U. S. C. § 1132(a)(3) (authorizing suit in federal court "by a participant, beneficiary, or fiduciary" of a pension or health plan governed by ERISA to gain redress for violations of "this subchapter or the terms of the plan"). We have no warrant to expand Congress' jurisdictional grant "by judicial decree." See Kokkonen v. Guardian Life Ins. Co. of America, 511 U. S. 375, 377 (1994).
55[697] Jackson Transit, Empire points out, referred to decisions "demonstrat[ing] that . . . private parties in appropriate cases may sue in federal court to enforce contractual rights created by federal statutes." 457 U. S., at 22. See Brief for Petitioner 15. This case, however, involves no right created by federal statute. As just reiterated, while the OPM—BCBSA master contract provides for reimbursement, FEHBA's text itself contains no provision addressing the reimbursement or subrogation rights of carriers.
56Nor do we read 5 U. S. C. § 8902(m)(1), FEHBA's preemption prescription, see supra, at 685-686, as a jurisdiction-conferring provision. That choice-of-law prescription is unusual in that it renders preemptive contract terms in health insurance plans, not provisions enacted by Congress. See 396 F. 3d, at 143-145; id., at 151 (Sack, J., concurring). A prescription of that unusual order warrants cautious interpretation.
57Section 8902(m)(1) is a puzzling measure, open to more than one construction, and no prior decision seems to us precisely on point. Reading the reimbursement clause in the master OPM—BCBSA contract as a condition or limitation on "benefits" received by a federal employee, the clause could be ranked among "[contract] terms . . . relat[ing] to . . . coverage or benefits" and "payments with respect to benefits," thus falling within § 8902(m)(1)'s compass. See Brief for United States as Amicus Curiae 20; Reply Brief 8-9. On the other hand, a claim for reimbursement ordinarily arises long after "coverage" and "benefits" questions have been resolved, and corresponding "payments with respect to benefits" have been made to care providers or the insured. With that consideration in view, § 8902(m)(1)'s words may be read to refer to contract terms relating to the beneficiary's entitlement (or lack thereof) to Plan payment for certain healthcare services he or she has received, and not to terms relating to the carrier's postpayments right to reimbursement. See Brief for Julia Cruz as Amicus Curiae 10, 11.
58[698] To decide this case, we need not choose between those plausible constructions. If contract-based reimbursement claims are not covered by FEHBA's preemption provision, then federal jurisdiction clearly does not exist. But even if FEHBA's preemption provision reaches contract-based reimbursement claims, that provision is not sufficiently broad to confer federal jurisdiction. If Congress intends a preemption instruction completely to displace ordinarily applicable state law, and to confer federal jurisdiction thereby, it may be expected to make that atypical intention clear. Cf. Columbus v. Ours Garage & Wrecker Service, Inc., 536 U. S. 424, 432-433 (2002) (citing Wisconsin Public Intervenor v. Mortier, 501 U. S. 597, 605 (1991)). Congress has not done so here.
59Section 8902(m)(1)'s text does not purport to render inoperative any and all state laws that in some way bear on federal employee-benefit plans. Cf. 29 U. S. C. § 1144(a) (portions of ERISA "supersede any and all State laws insofar as they may now or hereafter relate to any employee benefit plan"). And, as just observed, see supra, at 697, given that § 8902(m)(1) declares no federal law preemptive, but instead, terms of an OPM—BCBSA negotiated contract, a modest reading of the provision is in order. Furthermore, a reimbursement right of the kind Empire here asserts stems from a personal-injury recovery, and the claim underlying that recovery is plainly governed by state law. We are not prepared to say, based on the presentations made in this case, that under § 8902(m)(1), an OPM—BCBSA contract term would displace every condition state law places on that recovery.
60As earlier observed, the BCBSA Plan's statement of benefits links together the carrier's right to reimbursement from the insured and its right to subrogation. See supra, at 684-685. Empire's subrogation right allows the carrier, once it has paid an insured's medical expenses, to recover directly from a third party responsible for the insured's injury or [699] illness. See 16 G. Couch, Cyclopedia of Insurance Law § 61:1 (2d ed. 1982). Had Empire taken that course, no access to a federal forum could have been predicated on the OPM-BCBSA contract right. The tortfeasors' liability, whether to the insured or the insurer, would be governed not by an agreement to which the tortfeasors are strangers, but by state law, and § 8902(m)(1) would have no sway.
61In sum, the presentations before us fail to establish that § 8902(m)(1) leaves no room for any state law potentially bearing on federal employee-benefit plans in general, or carrier-reimbursement claims in particular. Accordingly, we extract from § 8902(m)(1) no prescription for federal-court jurisdiction.
62We turn finally to the argument that Empire's reimbursement claim, even if it does not qualify as a "cause of action created by federal law," nevertheless arises under federal law for § 1331 purposes, because federal law is "a necessary element of the [carrier's] claim for relief." Brief for United States as Amicus Curiae 25-26 (quoting Grable, 545 U. S., at 312, and Jones v. R. R. Donnelley & Sons Co., 541 U. S. 369, 376 (2004)). This case, we are satisfied, does not fit within the special and small category in which the United States would place it. We first describe Grable, a recent decision that the United States identifies as exemplary,[6] and then explain why this case does not resemble that one.
64Grable involved real property belonging to Grable & Sons Metal Products, Inc. (Grable), which the Internal Revenue Service (IRS) seized to satisfy a federal tax deficiency. 545 U. S., at 310. Grable received notice of the seizure by certified mail before the IRS sold the property to Darue Engineering & Manufacturing (Darue). Ibid. Five years later, [700] Grable sued Darue in state court to quiet title. Grable asserted that Darue's record title was invalid because the IRS had conveyed the seizure notice improperly. Id., at 311. The governing statute, 26 U. S. C. § 6335(a), provides that "notice in writing shall be given . . . to the owner of the property . . . or shall be left at his usual place of abode or business . . . ." Grable maintained that § 6335(a) required personal service, not service by certified mail. 545 U. S., at 311.
65Darue removed the case to federal court. Alleging that Grable's claim of title depended on the interpretation of a federal statutory provision, i. e., § 6335(a) of the Internal Revenue Code, Darue invoked federal-question jurisdiction under 28 U. S. C. § 1331. We affirmed lower court determinations that the removal was proper. "The meaning of the federal tax provision," we said, "is an important issue of federal law that sensibly belongs in a federal court." 545 U. S., at 315. Whether Grable received notice adequate under § 6335(a), we observed, was "an essential element of [Grable's] quiet title claim"; indeed, "it appear[ed] to be the only. . . issue contested in the case." Ibid.
66This case is poles apart from Grable. Cf. Brief for United States as Amicus Curiae 27. The dispute there centered on the action of a federal agency (IRS) and its compatibility with a federal statute, the question qualified as "substantial," and its resolution was both dispositive of the case and would be controlling in numerous other cases. See 545 U. S., at 313. Here, the reimbursement claim was triggered, not by the action of any federal department, agency, or service, but by the settlement of a personal-injury action launched in state court, see supra, at 687-688, and the bottom-line practical issue is the share of that settlement properly payable to Empire.
67Grable presented a nearly "pure issue of law," one "that could be settled once and for all and thereafter would govern numerous tax sale cases." Hart and Wechsler 65 (2005 Supp.). In contrast, Empire's reimbursement claim, McVeigh's [701] Vecounsel represented without contradiction, is fact-bound and situation-specific. McVeigh contends that there were overcharges or duplicative charges by care providers, and seeks to determine whether particular services were properly attributed to the injuries caused by the 1997 accident and not rendered for a reason unrelated to the accident. See Tr. of Oral Arg. 44, 53.
68The United States observes that a claim for reimbursement may also involve as an issue "[the] extent, if any, to which the reimbursement should take account of attorney's fees expended . . . to obtain the tort recovery." Brief as Amicus Curiae 29. Indeed it may. But it is hardly apparent why a proper "federal-state balance," see id., at 28, would place such a nonstatutory issue under the complete governance of federal law, to be declared in a federal forum. The state court in which the personal-injury suit was lodged is competent to apply federal law, to the extent it is relevant, and would seem best positioned to determine the lawyer's part in obtaining, and his or her fair share in, the tort recovery.
69The United States no doubt "has an overwhelming interest in attracting able workers to the federal workforce," and "in the health and welfare of the federal workers upon whom it relies to carry out its functions." Id., at 10. But those interests, we are persuaded, do not warrant turning into a discrete and costly "federal case" an insurer's contract-derived claim to be reimbursed from the proceeds of a federal worker's state-court-initiated tort litigation.
70In sum, Grable emphasized that it takes more than a federal element "to open the `arising under' door." 545 U. S., at 313. This case cannot be squeezed into the slim category Grable exemplifies.
71For the reasons stated, the judgment of the Court of Appeals for the Second Circuit is
73Affirmed.
74This case involves a dispute about the meaning of terms in a federal health insurance contract. The contract, between a federal agency and a private carrier, sets forth the details of a federal health insurance program created by federal statute and covering 8 million federal employees. In all this the Court cannot find a basis for federal jurisdiction. I believe I can. See Clearfield Trust Co. v. United States, 318 U. S. 363 (1943).
76There is little about this case that is not federal. The comprehensive federal health insurance program at issue is created by a federal statute, the Federal Employees Health Benefits Act of 1959 (FEHBA), 5 U. S. C. § 8901 et seq. (2000 ed. and Supp. III). This program provides insurance for Federal Government employees and their families. That insurance program today covers approximately 8 million federal employees, retirees, and dependents, at a total cost to the Government of about $22 billion a year. Brief for United States as Amicus Curiae 2.
79To implement the statute, the Office of Personnel Management (OPM), the relevant federal agency, enters into contracts with a handful of major insurance carriers. These agency/carrier contracts follow a standard agency form of about 38,000 words, and contain the details of the plan offered by the carrier. See § 8902(d) (2000 ed.) (requiring contract between carrier and agency to contain a detailed statement of the terms of the plan); see also Federal Employees Health Benefits Program Standard Contract (CR—2003) (2005), online at http://www.opm.gov/insure/carriers/samplecontract.doc (sample form agency/carrier contract) (as visited June 7, 2006, and available in Clerk of Court's case file). The contract lists, for example, the benefits provided to the employees who enroll. It provides a patient's bill of [703] rights. It makes clear that the Government, not the carrier, will receive the premiums and will pay the benefits. It specifies that the carrier will administer the program that the contract sets forth, for which the carrier will receive an adjustable fee. The contract also states, "By enrolling or accepting services under this contract, [enrollees] are obligated to all terms, conditions, and provisions of this contract." App. 90.
80As the statute requires, § 8907(b), the agency/carrier contract also provides that the carrier will send each enrolled employee a brochure that explains the terms of the plan, as set forth in the contract. The brochure explains that it "describes the benefits of the . . . [p]lan under [the carrier's] contract . . . with [the federal agency], as authorized by the [federal statute]." Id., at 158. The terms of the brochure are incorporated into the agency/carrier contract. Id., at 89. The carrier distributes the brochure with a seal attached to the front stating, "Authorized for distribution by the United States Office of Personnel Management Retirement and Insurance Service." Id., at 155.
81The program is largely funded by the Federal Government. More specifically, the Federal Government pays about 75% of the plan premiums; the enrollee pays the rest. § 8906(b). These premiums are deposited into a special fund in the United States Treasury. § 8909(a). The carrier typically withdraws money from the fund to pay for covered health care services, ibid.; however, the fund's money belongs, not to the carrier, but to the federal agency that administers the program. After benefits are paid, any surplus in the fund can be used at the agency's discretion to reduce premiums, to increase plan benefits, or to make a refund to the Government and enrollees. § 8909(b); 5 CFR § 890.503(c)(2) (2005). The carrier is not at risk. Rather, it earns a profit, not from any difference between plan premiums and the cost of benefits, but from a negotiated service charge that the federal agency pays directly.
82[704] Federal regulations provide that the federal agency will resolve disputes about an enrolled employee's coverage. § 890.105(a)(1); see also 5 U. S. C. § 8902(j) (requiring carrier to provide health benefit if OPM concludes that enrollee is entitled to the benefit under the contract). The agency's resolution is judicially reviewable under the Administrative Procedure Act in federal court. 5 CFR § 890.107 (2005).
83In sum, the statute is federal, the program it creates is federal, the program's beneficiaries are federal employees working throughout the country, the Federal Government pays all relevant costs, and the Federal Government receives all relevant payments. The private carrier's only role in this scheme is to administer the health benefits plan for the federal agency in exchange for a fixed service charge.
84The plan at issue here, the Blue Cross Blue Shield Service Benefit Plan, is the largest in the statutory program. The plan's details are contained in Blue Cross Blue Shield's contract with the federal agency and in the brochure, which binds the enrolled employee to that contract. In this case, the carrier seeks to require the enrolled employee's estate to abide by provisions that permit the carrier to obtain (and require the enrolled employee to pay) reimbursement from an enrollee for benefits provided if the enrollee recovers money from a third party (as compensation for the relevant injury or illness). The parties dispute the proper application of some of those provisions.
86First, the agency's contract with the carrier requires the carrier to "mak[e] a reasonable effort to seek recovery of amounts to which it is entitled to recover." App. 95. And the carrier must do so "under a single, nation-wide policy to ensure equitable and consistent treatment for all [enrollees] under this contract." Ibid. Any money recovered by the carrier goes into the statutory fund in the United States [705] Treasury, and may be spent for the benefit of the program at the discretion of the federal agency. See supra, at 703.
87Second, the agency/carrier contract and the brochure set forth the enrollee's obligation to reimburse the carrier under certain circumstances. The contract states, "The Carrier may . . . recover directly from the [enrollee] all amounts received by the [enrollee] by suit, settlement, or otherwise from any third party or its insurer . . . for benefits which have also been paid under this contract." App. 95. The agency/carrier contract also says that the "[c]arrier's subrogation rights, procedures and policies, including recovery rights, shall be in accordance with the provisions of the agreed-upon brochure text." Id., at 100. The relevant provisions in the brochure (which also appear in the appendix to the agency/carrier contract) tell the enrollee:
88"If another person or entity, through an act or omission, causes you to suffer an injury or illness, and if we pay benefits for that injury or illness, you must agree to the following:89
"All recoveries you obtain (whether by lawsuit, settlement, or otherwise), no matter how described or designated, must be used to reimburse us in full for benefits we paid. . . .90
"We will not reduce our share of any recovery unless we agree in writing to a reduction, . . . because you had to pay attorneys' fees." Id., at 165.91
The enrollee must abide by these requirements because, as explained above, the brochure tells the beneficiary that, by enrolling in the program, he or she is agreeing to the terms of the brochure, which in turn "describes the benefits of the [plan] under [the agency/carrier] contract." Id., at 158.
92I have explained the nature of the program and have set forth the terms of the agency/carrier contract in some detail [706] because, once understood, their federal nature brings this case well within the scope of the relevant federal jurisdictional statute, 28 U. S. C. § 1331, which provides jurisdiction for claims "arising under" federal law. For purposes of this statute, a claim arises under federal law if federal law creates the cause of action. Merrell Dow Pharmaceuticals Inc. v. Thompson, 478 U. S. 804, 808 (1986); see also American Well Works Co. v. Layne & Bowler Co., 241 U. S. 257, 260 (1916) (opinion of Holmes, J.) (A "suit arises under the law that creates the cause of action"). And this Court has explained that § 1331's "statutory grant of `jurisdiction will support claims founded upon federal common law as well as those of a statutory origin.'" National Farmers Union Ins. Cos. v. Crow Tribe, 471 U. S. 845, 850 (1985); see also Illinois v. Milwaukee, 406 U. S. 91 (1972); 19 C. Wright, A. Miller, & E. Cooper, Federal Practice and Procedure § 4514, p. 455 (2d ed. 1996) ("A case `arising under' federal common law presents a federal question and as such is within the original subject-matter jurisdiction of the federal courts"). In other words, "[f]ederal common law as articulated in rules that are fashioned by court decisions are `laws' as that term is used in § 1331." National Farmers, supra, at 850.
95It seems clear to me that the petitioner's claim arises under federal common law. The dispute concerns the application of terms in a federal contract. This Court has consistently held that "obligations to and rights of the United States under its contracts are governed exclusively by federal law." Boyle v. United Technologies Corp., 487 U. S. 500, 504 (1988). This principle dates back at least as far as Clearfield Trust, 318 U. S., at 366, where the Court held that the "rights and duties of the United States on [federal] commercial paper," namely a federal employee's paycheck, "are governed by federal rather than local law." The Court reasoned that "[w]hen the United States disburses its funds or [707] pays its debts, it is exercising a constitutional function or power," a power "in no way dependent on the laws of Pennsylvania or of any other state." Ibid. Accordingly, "[i]n [the] absence of an applicable Act of Congress it is for the federal courts to fashion the governing rule of law." Id., at 367.
96This Court has applied this principle, the principle embodied in Clearfield Trust, to Government contracts of all sorts. See, e. g., West Virginia v. United States, 479 U. S. 305, 308-309 (1987) (contract regarding federal disaster relief efforts); United States v. Kimbell Foods, Inc., 440 U. S. 715, 726 (1979) (contractual liens arising from federal loan programs); United States v. Little Lake Misere Land Co., 412 U. S. 580, 592 (1973) (agreements to acquire land under federal conservation program); United States v. Seckinger, 397 U. S. 203, 209 (1970) (Government construction contracts); United States v. County of Allegheny, 322 U. S. 174, 183 (1944) (Government procurement contracts).
97In this case, the words that provide the right to recover are contained in the brochure, which in turn explains the provisions of the contract between the Government and the carrier, provisions that were written by a federal agency acting pursuant to a federal statute that creates a federal benefit program for federal employees. At bottom, then, the petitioner's claim is based on the interpretation of a federal contract, and as such should be governed by federal common law. And because the petitioner's claim is based on federal common law, the federal courts have jurisdiction over it pursuant to § 1331. The lower federal courts have similarly found § 1331 jurisdiction over suits between private parties based on Federal Government contracts. See, e. g., Downey v. State Farm Fire & Casualty Co., 266 F.3d 675, 680-681 (CA7 2001) (Easterbrook, J.) (National Flood Insurance Program contracts); Almond v. Capital Properties, Inc., 212 F.3d 20, 22-24 (CA1 2000) (Boudin, J.) (Federal Railroad Administration [708] contract); Price v. Pierce, 823 F. 2d 1114, 1119-1120 (CA7 1987) (Posner, J.) (Dept. of Housing and Urban Development contracts).
98What might one say to the contrary? First, I may have made too absolute a statement in claiming that disputes arising under federal common law are (for jurisdictional purposes) cases "arising under" federal law. After all, in every Supreme Court case I have cited (except National Farmers and Milwaukee, and not including the Courts of Appeals cases), the United States was a party, and that fact provides an independent basis for jurisdiction. See 28 U. S. C. §§ 1345, 1346(a)(2), 1491(a)(1). In those cases the decision to apply federal common law was, therefore, a "choice-of-law issue" only, ante, at 691, and the Court consequently did not need to address the application of the Clearfield Trust doctrine to § 1331 "arising under" jurisdiction.
100But I have found no case where a federal court concluded that federal common law governed a plaintiff's contract claim but nevertheless decided that the claim did not arise under federal law. I have found several lower court cases (cited supra, at 707 and this page) where courts asserted § 1331 jurisdiction solely on the basis of federal common law. And in Machinists v. Central Airlines, Inc., 372 U. S. 682, 693, n. 17 (1963), this Court cited the Clearfield Trust cases in finding § 1331 jurisdiction over the contract suit before it, noting that although those cases "did not involve federal jurisdiction as such," nevertheless "they are suggestive" on the issue of § 1331 jurisdiction over suits involving Federal Government contracts "since they hold federal law determinative of the merits of the claim."
101It is enough here, however, to assume that federal common law means federal jurisdiction where Congress so intends. Cf. Clearfield Trust, supra, at 367 ("In absence of an applicable Act of Congress it is for the federal courts to fashion the governing rule of law according to their own standards" (emphasis added)). If so, there are strong reasons for the [709] federal courts, following Clearfield Trust, to assume jurisdiction and apply federal common law to resolve this case.
102First, although the nominal plaintiff in this case is the carrier, the real party in interest is the United States. Any funds that the petitioner recovers here it must pay directly to the United States, by depositing those funds in the FEHBA United States Treasury account managed by the federal agency. The carrier simply administers the reimbursement proceeding for the United States, just as it administers the rest of the agency/carrier contract. Accordingly, this case, just like the Clearfield Trust cases, concerns the "rights of the United States under its contracts." Boyle, 487 U. S., at 504.
103Second, the health insurance system FEHBA establishes is a federal program. The Federal Government pays for the benefits, receives the premiums, and resolves disputes over claims for medical services. Given this role, the Federal Government's need for uniform interpretation of the contract is great. Given the spread of Government employees throughout the Nation and the unfairness of treating similar employees differently, the employees' need for uniform interpretation is equally great. That interest in uniformity calls for application of federal common law to disputes about the meaning of the words in the agency/carrier contract and brochure. See Clearfield Trust, 318 U. S., at 367 (applying federal common law because the "desirability of a uniform [federal] rule is plain"); see also Bank of America Nat. Trust & Sav. Assn. v. Parnell, 352 U. S. 29, 33, 34 (1956) ("[L]itigation with respect to Government paper. . . between private parties" may nevertheless "be governed by federal [common] law" where there is "the presence of a federal interest"). And that interest in uniformity also suggests that the doors of the federal courts should be open to decide such disputes.
104Third, as discussed above, the provisions at issue here are just a few scattered islands in a sea of federal contractual provisions, all of which federal courts will interpret and [710] apply (when reviewing the federal agency's resolution of disputes regarding benefits). Given this context, why would Congress have wanted the courts to treat those islands any differently? I can find no convincing answer.
105Regardless, the majority and the Court of Appeals believe they have come up with one possible indication of a contrary congressional intent. They believe that the statute's jurisdictional provision argues against federal jurisdiction where the United States is not formally a party. That provision gives the federal district courts "original jurisdiction, concurrent with the United States Court of Federal Claims, of a civil action or claim against the United States founded on this chapter." 5 U. S. C. § 8912. According to the majority, if Congress had wanted cases like this one to be brought in the federal courts, it would have extended § 8912 to cover them. Ante, at 696.
106That is not so. Congress' failure to write § 8912 to include suits between carriers and enrollees over plan provisions may reflect inadvertence. Or it may reflect a belief that § 1331 covered such cases regardless. Either way, § 8912 tells us nothing about Congress' intent in respect to § 1331 jurisdiction.
107But why then did Congress write § 8912 at all? After all, the cases there covered—contract claims against the Federal Government "founded on" the federal health insurance program—would also be governed by federal common law and (if my view is correct) would have fallen within the scope of § 1331. What need would there have been (if my view is correct) to write a special section, § 8912, expanding federal jurisdiction to encompass these claims?
108The answer, as the majority itself points out, ante, at 686, is that Congress did not write § 8912 to expand the jurisdiction of the federal courts. It wrote that section to transfer a category of suits (claims against the United States exceeding $10,000) from one federal court (the Court of Federal Claims) to others (the federal district courts).
109[711] In sum, given Clearfield Trust, supra, and its progeny, there is every reason to believe that federal common law governs disputes concerning the agency/carrier contract. And that is so even though "it would have been easy enough for Congress to say" that federal common law should govern these claims. See ante, at 696. After all, no such express statement of congressional intent was present in Clearfield Trust itself, or in any of the cases relying on Clearfield Trust for the authority to apply federal common law to interpret Government contracts. See, e. g., cases cited supra, at 707; see also Clearfield Trust, supra, at 367 ("In absence of an applicable Act of Congress it is for the federal courts to fashion the governing rule of law according to their own standards"). Accordingly, I would apply federal common law to resolve the petitioner's contract claim. And, as explained above, when the "governing rule of law" on which a claim is based is federal common law, then the federal courts have jurisdiction over that claim under § 1331.
110The Court adds that, in spite of the pervasively federal character of this dispute, state law should govern it because the petitioner has not demonstrated a "`significant conflict. . . between an identifiable federal policy or interest and the operation of state law.'" Ante, at 693. But as I have explained, see supra, at 708-709, the Federal Government has two such interests: (1) the uniform operation of a federal employee health insurance program, and (2) obtaining reimbursement under a uniform set of legal rules. These interests are undermined if the amount a federal employee has to reimburse the FEHBA United States Treasury fund in cases like this one varies from State to State in accordance with state contract law. We have in the past recognized that this sort of interest in uniformity is sufficient to warrant application of federal common law. See, e. g., Boyle, supra, at 508 ("[W]here the federal interest requires a uniform rule, [712] the entire body of state law applicable to the area conflicts and is replaced by federal rules"); Kimbell Foods, 440 U. S., at 728 ("Undoubtedly, federal programs that `by their nature are and must be uniform in character throughout the Nation' necessitate formulation of controlling federal rules"); Clearfield Trust, 318 U. S., at 367 (applying federal common law because "application of state law . . . would subject the rights and duties of the United States to exceptional uncertainty" and "would lead to great diversity in results by making identical transactions subject to the vagaries of the laws of the several states," and therefore "[t]he desirability of a uniform rule is plain").
112But even if the Court is correct that "`[t]he prudent course'" is "`to adopt the readymade body of state law as the federal rule of decision until Congress strikes a different accommodation,'" ante, at 691-692 (quoting Kimbell Foods, supra, at 740), there would still be federal jurisdiction over this case. That is because, as Clearfield Trust, Kimbell Foods, and other cases make clear, the decision to apply state law "as the federal rule of decision" is itself a matter of federal common law. See, e. g., Kimbell Foods, supra, at 728, n. 21 ("`Whether state law is to be incorporated as a matter of federal common law . . . involves the . . . problem of the relationship of a particular issue to a going federal program'" (emphasis added)); Clearfield Trust, supra, at 367 ("In our choice of the applicable federal rule we have occasionally selected state law" (emphasis added)); see also R. Fallon, D. Meltzer, & D. Shapiro, Hart and Wechsler's The Federal Courts and the Federal System 700 (5th ed. 2003) ("[T]he current approach, as reflected in [Kimbell Foods, supra], suggests that . . . while under Clearfield federal common law governs, in general it will incorporate state law as the rule of decision"); 19 C. Wright, A. Miller, & E. Cooper, Federal Practice and Procedure § 4518, at 572-573 ("In recent years, the Supreme Court has put increasing emphasis on the notion that when determining what should be the content [713] of federal common law, the law of the forum state should be adopted absent some good reason to displace it" (emphasis added; citing Kimbell Foods, supra, and Clearfield Trust, supra)).
113On this view, the Clearfield Trust inquiry involves two questions: (1) whether federal common law governs the plaintiff's claim; (2) if so, whether, as a matter of federal common law, the Court should adopt state law as the proper "`federal rule of decision,'" ante, at 692 (emphasis added). See, e. g., Kimbell Foods, supra, at 727 (deciding that "[f]ederal law therefore controls" the dispute but concluding that state law gives "content to this federal rule"); United States v. Little Lake Misere Land Co., 412 U. S., at 593-594 (The "first step of the Clearfield analysis" is to decide whether "`the courts of the United States may formulate a rule of decision,'" and the "next step in our analysis is to determine whether" the federal rule of decision should "`borro[w]' state law"); see also Friendly, In Praise of Erie—and of the New Federal Common Law, 39 N. Y. U. L. Rev. 383, 410 (1964) ("Clearfield decided not one issue but two. The first . . . is that the right of the United States to recover for conversion of a Government check is a federal right, so that the courts of the United States may formulate a rule of decision. The second . . . is whether, having this opportunity, the federal courts should adopt a uniform nation-wide rule or should follow state law" (footnote omitted)). Therefore, even if the Court is correct that state law applies to claims involving the interpretation of some provisions of this contract, the decision whether and when to apply state law should be made by the federal courts under federal common law. Accordingly, for jurisdictional purposes those claims must still arise under federal law, for federal common law determines the rule of decision.
114Finally, the footnote in Boyle cited by the Court did not purport to overrule Clearfield Trust on this point. See Boyle, 487 U. S., at 507, n. 3 ("If the distinction between displacement [714] of state law and displacement of federal law's incorporation of state law ever makes a practical difference, it at least does not do so in the present case").
115With respect, I dissent.
116[1] Clinton A. Krislov and Michael R. Karnuth filed a brief for Julia Cruz, as representative of Jose S. Cruz, as amicus curiae urging affirmance.
117[2]The statement of benefits further provides:
118119"You must tell us promptly if you have a claim against another party for a condition that we have paid or may pay benefits for, and you must tell us about any recoveries you obtain, whether in or out of court. We may seek a lien on the proceeds of your claim in order to reimburse ourselves to the full amount of benefits we have paid or will pay.
"We may request that you assign to us (1) your right to bring an action or (2) your right to the proceeds of a claim for your illness or injury. We may delay processing of your claims until you provide the assignment.
"Note: We will pay the costs of any covered services you receive that are in excess of any recoveries made." App. 165.
[3] At oral argument, counsel for respondent McVeigh represented that "most of the [reimbursement claims] are not of th[is] magnitude"; "[m]ost of the cases involve [amounts like] $5,500 and $6,500." Tr. of Oral Arg. 52.
120[4] The United States, in accord with the dissent in this regard, see post, at 707, several times cites United States v. County of Allegheny, 322 U. S. 174 (1944), see, e. g., Brief as Amicus Curiae 10, 15, 26, maintaining that the construction of a federal contract "necessarily present[s] questions of `federal law not controlled by the law of any State,'" id., at 26 (quoting 322 U. S., at 183). Allegheny does not stretch as widely as the United States suggests. That case concerned whether certain property belonged to the United States and, if so, whether the incidence of a state tax was on the United States or on a Government contractor. See id., at 181-183, 186-189. Neither the United States nor any United States agency is a party to this case, and the auxiliary matter here involved scarcely resembles the controversy in Allegheny.
121[5] The dissent nowhere suggests that uniform, court-declared federal law would govern the carrier's subrogation claim against the tortfeasor. Nor does the dissent explain why the two linked provisions—reimbursement and subrogation—should be decoupled.
122[6] As the Court in Grable observed, 545 U. S., at 312, the classic example of federal-question jurisdiction predicated on the centrality of a federal issue is Smith v. Kansas City Title & Trust Co., 255 U. S. 180 (1921).